Sunteți pe pagina 1din 286

1

Anatomy:
1-Muscle involved in snoring? uvula

2-Which nerve if paralyzed pt looks downward and out ward? Oculomotor nerve

3-What are muscles of the palate?

Tensor veli palatine, levator veli palatine,musculais uvulais,palatogolssal and palate


pharyngeal

4-special sensory to ant2/3 of the tongue?

Chorda tympani of facial never by the lingual nerve of mandible

5-parasympathatic to lacrimal gland?

Facial nerve via the greater pertrosal never to the zygomaticofacial through the pterygoid
gongilia

6-muscles of facial expression.?

Frontalis,buccinators,zygomatico major, zygomatico minor,orbicularis oculi.orbicularis


oris,risorus,levator labii superioris,levator labii superiorus alaquenasii,mentalis ,depressor
labii inferioris,depressor anguli oris

7- The reflex in gagging patients is caused by?

A. Trigeminal nerve

B. Glossopharyngeal

C. Facial nerve

D. Recurrent laryngeal

Answer is b

8- Salivary gland that release the least mucous secretion:

labial glands,

palatal glands,

parotids,

submandibular,

submaxilar

answer is the parotid gland

9-Cranial nerve affected in a man with carcinoma in parotid salivary gland?

Facial nerve

1
2

10-muscle that opens Eustachian tube?

Tensor velli palatine

Function of tensor tympani and inervation?

Damps down the the vibration and inervated with mandibular nerve

11-Inervation of tongue muscles, digastric muscle (anterior and posterior belly), mylohiod
muscle?

Tongue muscle is the hypoglossal

Digastric muscl the anterior belly by the mandibulare nerve and posterior belly by facial

Mylohyoid by the mylohyoid nerve branch of the mandibular nerve

12-insertion of masseter muscle?

The lateral surface of the ramus

13-The portion of the medial pterigoid that arises from Maxillary Tuberosity is considered:

Dee

Superficial,

Bulk,

Upper

Answer is superfacial head

14-Patient 45 yr old, female, unable to close left eye. Dx? Facial palsy, Bell’s palsy, other
options

Bell’s pulsy

15-Which nerve supplies the muscles of facial expression?

Facial nerve

16- Nerve emerging from the incisive foramen? Nasopalatine

17- Diagram of mucles showing geniohyoid, styloglossus, stylohyoid

18- Diagram showing transverse facial artery, superficial temporal artery & facial artery

19- Diagram showing muscles of facial expression. Orbicularis oris, depressor anguli oris,
levator labii superioris alaque nasi

20- Diagram where hyoid bone was marked

21- Really awful and unclear diagram of extrinsic tongue muscles to identify 4 of them, it
looked like if a child had drawn it!

2
3

22-Only one question in the whole paper on cranial nerves –which was the one that
innervates facial muscles:.

Facial nerve

23- Diagram of arteries to identify 3 of them: I put external carotid, facial transversal and
ascending pharyngeal but not sure they are correct just to give you an idea that you need to
be familiar with name and location of the most important arteries of head and neck.

24- Where does the duct of the parotid gland open?

On the chech mucosa opposite to the maxillary second molar

25-Muscle involved in closing the oropharyngeal isthmus?

Palatoglossal

26-Nerve supplying skin over forehead?

opthalmic branch of 5

27-Type of saliva secreted by the glands ?

Parotid is serous

Submandibular is mixed

Sublingual is mucous

28-Branch of superficial temporal artery---transverse facial,branches to external


ear,zygomatic,middle temporal n two terminal ….posterior n anterior(all of them

29- Muscle attached to the auditory tube-?

Tensor tympani, tensor velli palatine ,levator velli palatine and salpingiopharyngeal

31-Eye muscles and their nerve supply ?

Superior oblique is by the trochlear nerve

Lateral rectus by the abducent

Oculomotor supplies the rest

32-Nerve passing through optic canal? Optic nerve

33-Questions on muscles in the floor of the mouth-hyoglossus,geniohyoid,mylohyoid(all 3)

34-Questions on tongue musculature, their nerve supply and movements?

Nerve supply by the hypoglossal except the palatoglossal by the pharyngeal plexus

Intrenstic muscles :superior longitudenial shorten the tongue and moves the tip upward

3
4

Inferior longitudinal shortens the tongue and moves the tip downward

Vertical muscles flattens and boardens the tongue

Transverse muscle marrows and elongates the tongue

Extrenstic muscle :genioglossal it protrudes and depresses the tongue

Palatoglossal depresses the soft palate and pulls the tongue upwars

Hypoglossal depresses and retrudes the tongue

Styloglossal evelavtes the tongue and retracts the tongue

35-. Lingual muscles extrinsic and intrinsic ones shown and you have to select their names

36-Nerve passing through nasopalatine canal (picture) ?  Nasopalatine nerve

36- Pictures of arteries and facial muscles,tongue muscles

37-Diagram of hyoid bone

38-Cranial nerves

39- Innervations of parotid gland?

Parasympathetic is by the lesser petrosal branch on the glossopharyngeal which synapse in


the otic gongilia and supplies the parotid gland through the auriculotemporal nerve branch
of the mandibulare nerve

Sympathetic by the carotid plexus

40- Innervations of tongue, anterior and posterior diagastric? repeated

41-Muscles of mastication, their origin, insertion etc.?

Temporalis which orginate from the inferior temporal line and inserts in thecoronoid
process

Masseter from the inferior boarder of the zygomatic bone and insertion in the lateral
surface of the ramus

Medial pterygoid the deep head is the medial surface of the lateral pteryoid and the
superfacial head is the maxillary tuberosity inserction is the posterior medial surface of the
mandible

Lateral pterygoid the superior head is the infratemporal fossa and the inferior head Is the
lateral surface of the lateral ptyrigoid muscle insertion is the neck of the condyle

42-Facial nerve.

43-Lingual nerve

4
5

44-Parasympathetic supply of salivary glands ..

Parotid is the glossopharyngeal (lesser peterosal ) and auriculotemporal of the mandile


through otic ganglia

Sublingual and submandibular is the the chorda tympani of the facial nerve which synapse
through submandibular gongilia and supplies through the lingual nerve

45-Cranial nerves just do the signs when they get damaged, especially, facial nerve,
hypoglossal nerve,?

46-Muscles used in the process of swallowing.?

These are all the muscle involved in swallowing :

intrinstic muscle of the tongue,supra hyoid muscle,palatoglossus and stylogolssal,orbicularis


oris,levator and tensor veli palatine,platopharyngeal sphnictor,digastrics
stylohyoid,palatopharygeous,stylopharengeous ,slinopharyngeous,aryepiglotis and oblique
arytenoids and pharyngeal constrictor

but I think the main muscles involved are the Levator velli palatine,tensor velli palatine
,palaglossal and palatopharyngeal

47-Do the nerve supply completely of face, teeth, tongue

48- Lingual nerve

49-Nerve supply to muscles

50-Nerve lesions abdusent,hypoglossal nerve

Abducent nerve ,Patient is unable to look laterally.

Hypoglossal nerve the tongue deviates to the affected side

Glossopharengeal the soft palate deviates to the unaffected site

51-Extracranial cause for facial palsy?

Trauma , infection ,neoplasim, melkerson Rosenthal syndrome, bellsy pulsy , misplaced local
anasthesia and connective tissue diseases

52-What type of lesion occurs in the following?

i)a patient with deviation of the corner of his mouth when he smiles but wrinkling of his
forehead is normal

ii)a patient with bell's palsy

iii)a patient with deviation of the corner of his mouth when he smiles and cannot wrinkle his
forehead-

i-superior nuclear lesion

5
6

ii-inferior nuclear lesion

iii-inferior nuclear lesion

53- Origin & insertion of all muscles of mastication.

54-Which muscle is responsible for moving food from the buccal sulcus in between the
teeth during mastication? Buccinator muscle

55- Which nerve is affected if a patient is unable to gaze laterally to the left

a)right abducens (b) left abducens (c)right trochlear (d)left trochlear

answer is b

56-Which nerve is affected if the tongue deviates to the right when protruded

a)right hypoglossal (b )Ieft hypoglossal (c )right glossopharyngeal (d) left glossopharyngeal

answer is a

57-Parasympathetic nerve supply to the salivary glands is by

a)Vagus and glossopharyngeal nerves

b)faciaJ and glossopharyngeal

c)facial and lingual

answer is b

58- BASE OF THE SKULL WITH ARROW ON THE CAROTED CANAL - NAME IT , WHAT PASS
THROW IT

Internal carotid artery

59- Which salivary gland gives the major contribution at rest? Submandibular

60-Which salivary gland gives the major contribution at function? parotid

61-.Cranial nerve defects- different options with different clinical presentations about 8
questions with

different optionsf options. CN II, eN III, v, VII, IX, X)

62- Loss of taste sensation is attributed to which nerve?

Chorda tymoani branch of the facial nerve

63-questions about land marks ofOPG. Hard palate, ID nerve

canal, zygomatic matress, .

64- What muscles are attached to condyles,coronoid process.max tuberosity,ramus of


mandible

6
7

Condyle is the lateral pterygoid muscle

Coronoid is the temporalis and the deep head of masseter muscle

Maxillary tuberiosity is the superfacial head of the medial pterygoid muscle

Ramus of the mandible is the masseter muscle

65- anatomy of parotid and floor of mouth with differnt muscels to mark in diagram

66-(FIG 25, MASTER] PAGE 40) NAME THE STRUCTURES (BUCCENATOR, MEDIAL PTERYGOID,
...)

67-. Picture of different muscles given asked to identify buccinator, geniohyoid, mylohyoid,
lateral pterygoid.

68- foramens shown ovale, internal' carotid canal, spinosum structures passing through
them.

Ovale :mandibular nerve,lesser petrosal

Spinosum :middle meningile artery

Nervouspinosum

Carotid canal :internal carotid artey

69- Which nerve makes pt to look upwards and outwards occulomotor

70- diagram of the muscles of tongue to identify, plus some sprahyoid muscles:
palatoglosuss , hyoglosuss, stylohyoid

71- Blood supply of Masseter muscle.?

Massetric branch of the maxillary artery

72- The nerve supplying the TMJ is,

A-auriculotemporal nerve

B. Nerve to masseter

C. Facial nerve

Answer is a

73- which arch forms the digastrics muscle and its invervation?

anterior belly from the first arch and innervated by the mandibular and posterior belly from
the second arch innervated by the facial

74- block given to anaesthetise buccal side in third molar extraction option are:

7
8

a-nasopalatine,b-inferior alveolar,c-long buccal

answer is c

75-which one of the following is an extracranial cause of facial palsy:

a-stroke,b-melkerson-rosenthal syndrome,c-bells pulsy

answer is b and c

76-ramsy hunt syndrome is a complication of:

a-herpes simplex infection,b-herpes zoster infection,c-coxachie virus infection

answer is b

77- Medial pterygoid muscle has:


a)anterior-had
B)superior-head
C)upper-head
D) superficial head
answer is d medial pterygoid has a superfacial and deep head,lateral pterygoid has a
superioer and inferior head

78-What portion of masseter leavers mandibular and pulls anteriorly?


Superfacial part

79- Which one is remnant of the Meckel's cartilage


A)temporomandibular-ligament
B)sphenomandibular-ligament
C) stylomandibular ligament
Answer is B

Immunology
1. Which immunoglobin exists as both a monomer and diamer?
Answer: IgA
2. Which immunoglobulin associated to MALT (mucosa associated lymphatic tissue?)
Answer: Predominantly IgA (IgA & IgM
Mucosa-associated lymphoid tissue (MALT) is scattered along mucosal linings, measuring
roughly 400 m2. It is the most extensive component of human lymphoid tissue. These
surfaces protect the body from an enormous quantity and variety of antigens. The tonsils,
Peyer patches within the small intestine, and the vermiform appendix are examples of
mucosa-associated lymphoid tissue (MALT).
Mucosa-associated lymphoid tissue (MALT) includes gut-associated lymphoid tissue (GALT),
bronchial/tracheal-associated lymphoid tissue (BALT), nose-associated lymphoid tissue
(NALT), and vulvovaginal-associated lymphoid tissue (VALT). Additional mucosa-associated

8
9

lymphoid tissue (MALT) exists within the accessory organs of the digestive tract,
predominantly the parotid gland.

The germinal center is key to the generation of a normal immune response. The location of
mucosa-associated lymphoid tissue (MALT) is key to its function. Stimulation of B
lymphocytes leads to the production of immunoglobulin A (IgA) and IgM within the Peyer
patches, preventing adherence of bacteria and viruses to the epithelium, thus blocking
entry to the subepithelial layers of the intestine.
The direct secretion of secretory IgA onto mucosal epithelia represents the major effector
mechanism of mucosa-associated lymphoid tissue (MALT). Major accumulations of
lymphoid tissue are found in the lamina propria of the intestine. M cells in the intestinal
epithelium overlying Peyer patches allow transport of antigens to the lymphoid tissue
beneath it.
The complex interplay among antigens, cells, and cytokines results in a very efficient
immune response. The efficiency of mucosa-associated lymphoid tissue (MALT) also
depends on the adequate function of IgA. Individuals with selective IgA deficiency are prone
to infections along mucosal surfaces in the respiratory, gastrointestinal, and genitourinary
tracts. Adequate function of IgA depends on the production and acquisition of a joining (J)
chain. This glycoprotein is produced by plasma cells and is important in the formation of IgA
dimers and IgM pentamers. It has been shown that in children who have recurrent
tonsillitis, B lymphocytes in tonsillar crypts do not produce the J chain. The J chain is key in
permitting secretory IgA and IgM to function as the first line of defense in mucosal
epithelium.

3. What type of hypersensitivity reaction causes amalgam restoration?


Answer: Type IV

4. KIWI CAUSES ANAPHYLAXIS & URTICARIA ?


ANSWER: TYPE I HYPERSENSITIVITY

5. W HICH IMMUNOGLOBIN IS THE FIRST TO BE FOUND WHEN THE CHILD IS BORN?


ANSWER: IGM( FIRST TO BE PRODUCED BY THE NEONATE AND PRODUCED IN PRIMARY IMMUNE
RESPONSE , 5-10% INSERUM)
IGG CROSSES PLACNETA AND PROTECTS FETUS, PRODUCED IN SECONDARY IMMUNE RESPONSE . M OST
ABUNDANT IMMUNOGLOBULIN 80% IN SERUM .
IGA MAJOR SECRETRY IMMUNOGLOBULIN (10 – 15%)

6. Which immunoglobin exists as both a monomer and diamer?


Answer: IgA

7. Which immunoglobin will increase in gingival inflammation?


Answer: IgG

8. W HAT IS THE % OF TRANSMISSION OF THESE VIRUSES?


A -I S IT HBV
B-HCV

9
10

C-HIV
A -30%,B-3%,C-0.3%
9-WHICH IS THE MOST STABLE VIRUS OUTSIDE THE BODY?
THE HEPATITIS A VIRUS, OR HAV, IS RELATIVELY HARDY.IN GOOD CONDITIONS, IT CAN SURVIVE OUTSIDE
THE BODY FOR MONTHS . H CAN SURVIVE CERTAIN ACIDS AND SOME HEAT . F OR A PERIOD OF TIME AND
UNDER CERTAIN CONDITIONS , HAV CAN SURVIVE IN SEA WATER , DRIED FECES AND LIVE OYSTERS.

 HEPATITIS B VIRUS
THE HEPATITIS B VIRUS, OR HBV, CAN STILL BE INFECTIOUS FOR UP TO A WEEK OUTSIDE THE BODY.

HOW LONG DOES THE HEPATITIS C VIRUS SURVIVE OUTSIDE THE BODY?
THE HEPATITIS C VIRUS CAN SURVIVE OUTSIDE THE BODY AT ROOM TEMPERATURE , ON ENVIRONMENTAL
SURFACES , FOR AT LEAST 16 HOURS BUT NO LONGER THAN 4 DAYS .

10-H EPATITIS D VIRUS?


THIS VIRUS NEEDS HELP FROM THE HEPATITIS B VIRUS TO BE INFECTIOUS , SO IT ONLY INFECTS SOMEONE
WITH HEPATITIS B.

11-H EPATITIS E VIRUS?


THIS VIRUS IS SPREAD SIMILAR TO THE HEPATITIS A VIRUS AND CAUSES ACUTE DISEASE SIMILAR TO THE
OTHERS.

10-3 IN 1000 GET HIV AFTER NEEDLE STICK INJURY


4 IN 1000 GET HEBC   AFTER NEEDLE STICK INJURY
THE RISK IS MORE THAN THAT IN HEP B BECAUSE WE ARE IMMUNIZED THAT FROM ODELL BOOK 2010.
** HIV VIRUSE HAS THE LEAST ABILITY TO SURVIVE OUTSIDE THE BODY AS IT CANNOT SURVIVE OUTSIDE
THE BODY FOR MORE THAN A MAXIMUM OF 30 TO 60 SECONDS

11. H B E ANTIGEN MEANS:


A . HIGH RISK INFECTION
B. RECENT INFECTION
C. IMMUNE TO INFECTION
ANSWER: A

12. IGG AND C3 ARE SEEN IN:


A . LICHEN PLANUS
B. PHEMPHIGOID
C. ERYTHEMA MULTIFORM
D . PHEMPHIGUS VULGARIS
ANSWER: D

13. YOU SUSPECT THE PATIENT SUFFERS HEPATITIS B SYMPTOMS , AFTER INESTIGATION, THE TEST SHOW
HBE AG ANTIBODIES, WHAT DOES THAT INDICATE ?
A - THE PATIENT IS GETTING BETTER
B- THE PATIENT IS IN A TRANSMITTING STATE OF THE DISEASE
ANSWER:B

10
11

14. IS MYASTHENIA GRAVIS AND GRAVE'S DISEASE TYPE 2 OR TYPE 5 HYPERSENSITIVITY REACTION?
ANSWER: BOTH, MEDIATED BY IGM AND IGG

15. W HICH IMMUNOGLOBULIN INCREASES IN GINGIVAL INFLAMMATION ?


ANSWER: IGG

16. IG PRESENT IN PERIODONTITIS?


ANSWER: IGG ESPECIALLY IGG2

17. W HICH IMMUNOGLOBULIN IS DIMERIC ?


ANSWER: SECROTORY IGA

18. WHICH ANTIBODY IS FOUND IN BACTERIAL AND VIRAL INFECTION?


ANSWER: IGG AND IGM

19-LEVEL OF IGA?
A) SALIVA >SERUM>BREASTMILK >TEARS
B) SERUM> SALIVA > BREASTMILK >TEARS
C) SALIVA > BREASTMILK >TEARS> SERUM
ANSWER: B
ALTERNATE ANSWER : SERUM>TEARS>SALIVA>BREAST MILK     OR IF TO GIVE IN FIGURES 
SERUM--40-400 MG/DL,TEARS--60-85 MG/DL,  SALIVA--6.2-14.5 MG/DL,  BREAST
MILK--APPROX 0.708 MG /DL      LOOK IN BLUE BOOK PAGE 452                             

20. Which antibody is used for dental caries vaccine?is it IgA or IgG?
Answer: IgA INHIBITS THE ADHERENCE of microbes and neutralizes toxins and viruses.

21. Abundant immunoglobulin?


Answer:
IgG is the most abundant in body followed by IgA
Immunoglobulins, quantitative serum
IgA 68 - 378 mg/dL
IgG 768 - 1632 mg/dL
IgM 60 - 263 mg/dL
IgE 10 - 180 IU/L

22. M OST ABUNDANT IMMUNOGLOBULIN IN SALIVA DURING INFECTIONS?


ANSWER: IGA

23. W HICH IMMUNOGLOBIN WILL INCREASE IN GINGIVAL INFLAMMATION? IGG

24. Which immunoglobulin increase in periodontitis? IgG

25. Type of Hepatitis that doesn’t become chronic ?


Answer: Hep A and E
26. M OST RELIABLE TEST TO INDICATE THE PRESENCE OF ACTIVE HEPATITIS -

11
12

A. HBS ANTIGEN
B. HBC ANTIGEN
C. HB E ANTIGEN
D . ANTIBODY TO HBS ANTIGEN
Anwers is A
27 – type of hypersensivity in rhuematoid arithritis ? Type 3 hypersinsivity immune complex

28.which condition that phagocyte kill bacteria is more difficult (tbc etc.)

29. which Ig will increase in gingival inflammation? IgG

30. Which hepatitis doesnt become chronic? Hepatitis A and E

31-Vaccination that does not include deactivated micro-organism?


Deactivated vaccination mean the use of live microbes such as Typhoid,BCG,MMR and
vericella ,yellow fever.

32-. Which immunoglobin is present after a bacterial and viral infection? IgM and IgG

33-. Which immunoglobulin is pentameric? IgM

34- What cells do you see in parasitic infection? Eosinophils

35- Immunoglobulin in mucosa ? IgA

36- Which cell release histamine? Mast cells

37-which type of hypersensitivity reaction  is seen in tuberculous lesion?


a) type1
b)type2
c)type3
d)type4
answer..d

38-WHICH OF THE FOLLOWING INFLAMMATORY CELLS IN PERIODONTITIS TRANSFORM INTO PLASMA


CELLS UPON SECONDARY EXPOSURE TO AN ANTIGEN ?
A . NEUTROPHILS
B. T-LYMPHOCYTES
C-B-LYMPHOCYTE
D . M ACROPHAGES
E .M AST C ELLS
ANSWER ... C...

39- THE PRESENCE OF ANTIBODIES AGAINST WHICH ANTIGENS OF THE FOLLOWING INDICATES IMMUNITY
AGAINST H EPATITIS B?

12
13

HBE ANTIGEN
HBC ANTIGEN
ANSWER.. HBS ANTIGEN

40-Which test is the most reliable test to indicate the presence of active hepatitis
Answer is HbsAg
Hbs(surface)antigen - Hepatitis B surface antigen is a marker of infectivity. Its presence
indicates either acute or chronic HBV infection.
Hbc(core)antigen - It may be used in prevaccination testing to determine previous exposure
to HBV infection.
Hbe antigen - Hepatitis B “e” antigen is a marker of a high degree of HBV infectivity, and it
correlates with a high level of HBV replication. It is primarily used to help determine the
clinical management of patients with chronic HBV infection.
Antibody to HBs antigen - Antibody to hepatitis B surface antigen is a marker of immunity.
Its presence indicates an immune response to HBV infection, an immune response to
vaccination, or the presence of passively acquired antibody. (It is also known as HBsAb, but
this abbreviation is best avoided since it is often confused with abbreviations such as
HBsAg.)

41- what are the types of different Hepatitis ?


Hepatitis A Virus

The hepatitis A virus, or HAV, is relatively hardy. In good conditions, it can survive outside
the body for months. HAV can survive certain acids and some heat. For a period of time and
under certain conditions, HAV can survive in sea water, dried feces and live oysters.
Hepatitis D Virus
This virus needs help from the hepatitis B virus to be infectious, so it only infects someone
with hepatitis B.
Hepatitis E Virus
This virus is spread similar to the hepatitis A virus and causes acute disease similar to the
others.
Hepatitis B Virus
The hepatitis B virus, or HBV, can still be infectious for up to a week outside the body.

42-How long does the hepatitis C virus survive outside the body?
The hepatitis C virus can survive outside the body at room temperature, on environmental
surfaces, for at least 16 hours but no longer than 4 days.

Wikipedia: hypersensitivity
Hypersensitivity
Classification & external
resources

13
14

Hypersensitivity refers to undesirable (damaging, discomfort-producing and sometimes


fatal) reactions produced by the normal immune system. Hypersensitivity reactions require
a pre-sensitized (immune) state of the host. It is a four-group classification .
Type 1 - immediate (or atopic, or anaphylactic)

Type 1 hypersensitivity is an allergic reaction provoked by reexposure to a specific type of


antigen referred to as an allergen. [2] Exposure may be by ingestion, inhalation, injection, or
direct contact. The difference between a normal immune response and a type I
hypersensitive response is that plasma cells secrete IgE. This class of antibodies binds to Fc
receptors on the surface of tissue mast cells and blood basophils. Mast cells and basophils
coated by IgE are "sensitized." Later exposure to the same allergen, cross-links the bound
IgE on sensitized cells resulting in degranulation and the secretion of pharmacologically
active mediators such as histamine, leukotriene, and prostaglandin that act on the
surrounding tissues. The principal effects of these products are vasodilation and smooth-
muscle contraction.
The reaction may be either local or systemic. Symptoms vary from mild irritation to sudden
death from anaphylactic shock. Treatment usually involves epinephrine, antihistamines, and
corticosteroids.
Some examples:
Allergic asthma
Allergic conjunctivitis
Allergic rhinitis ("hay fever")
Anaphylaxis
Angioedema
Atopic dermatitis (eczema)
Urticaria (hives)
Eosinophilia
Penicillin
cephalosporin
Type 2 - antibody-dependent
In type 2 hypersensitivity, the antibodies produced by the immune response bind to
antigens on the patient's own cell surfaces. The antigens recognized in this way may either
be intrinsic("self" antigen, innately part of the patient's cells) or extrinsic (absorbed onto the
cells during exposure to some foreign antigen, possibly as part of infection with a pathogen).
These cells are recognised by macrophages or dendritic cells which act as antigen presenting
cells, this causes a B cell response where antibodies are produced against the foreign
antigen. An example here is the reaction to penicillin where the drug can bind to red blood
cells causing them to be recognised as different, B cell proliferation will take place and
antibodies to the drug are produced. IgG and IgM antibodies bind to these antigens to form
complexes that activate the classical pathway of complement activation for eliminating cells
presenting foreign antigens (which are usually, but not in this case, pathogens). That is,
mediators of acute inflammation are generated at the site and membrane attack complexes
cause cell lysis and death. The reaction takes hours to a day.
Another form of type 2 hypersensitivity is called Antibody Dependent Cell Mediated
Cytotoxicity (ADCC). Here, cells exhibiting the foreign antigen are tagged with antibodies
(IgG or IgM). These tagged cells are then recognised by Natural Killer (NK) cells and

14
15

macrophages (recognised via IgG bound to the cell surface receptor, CD16 (FcγRIII)), which
in turn kill these tagged cells.
Some examples:
Autoimmune hemolytic anemia
Goodpasture's syndrome
Erythroblastosis Fetalis
Pemphigus
Pernicious anemia (if autoimmune)
Immune thrombocytopenia
Transfusion reactions
Hashimoto's thyroiditis
Graves' disease (see type V below)
Myasthenia gravis (see type V below)
Rheumatic fever
Hemolytic disease of the newborn
Type 3 - immune complex
In type 3 hypersensitivity, soluble immune complexes (aggregations of antigens and IgG and
IgM antibodies) form in the blood and are deposited in various tissues (typically the skin,
kidney and joints) where they may trigger an immune response according to the classical
pathway of complement activation (see above). There are two stages relating to the
development of the complexes, firstly the complex forms when IgG and IgM antibodies are
bound to an antigen, after this, the complexes can form larger ones which can be cleared by
the body. It is at the first stage of this formation where clearance is not possible and the
antigen-antibody complex will spread and deposit as stated above. The reaction takes hours
to days to develop.
Some clinical examples:
Immune complex glomerulonephritis
Rheumatoid arthritis
Serum sickness
Subacute bacterial endocarditis
Symptoms of malaria
Systemic lupus erythematosus
Arthus reaction
Farmer's Lung (Arthus-type reaction)
Type 4 - cell-mediated (Delayed-Type Hypersensitivity, DTH)

Type 4 hypersensitivity is often called delayed type as the reaction takes two to three days
to develop. Unlike the other types, it is not antibody mediated but rather is a type of cell-
mediated response.
CD8+ cytotoxic T cells and CD4+ helper T cells recognise antigen in a complex with either
type 1 or 2 major histocompatibility complex. The antigen-presenting cells in this case are
macrophages which secrete IL-12, which stimulates the proliferation of further CD4+ T cells.
CD4+ T cells secrete IL-2 and interferon gamma, further inducing the release of other Type 1
cytokines, thus mediating the immune response. Activated CD8+ T cells destroy target cells
on contact while activated macrophages produce hydrolytic enzymes and, on presentation
with certain intracellular pathogens, transform into multinucleated giant cells.
Some clinical examples:

15
16

Contact dermatitis (poison ivy rash, for example)


Temporal arteritis
Symptoms of leprosy
Symptoms of tuberculosis
Transplant rejection
Coeliac disease
Metals
Type 5 - stimulatory
This is an additional type that is sometimes (often in Britain) used as a distinction from Type
2.[3]
Instead of binding to cell surface components, the antibodies recognize and bind to the cell
surface receptors, which either prevents the intended ligand binding with the receptor or
mimics the effects of the ligand, thus impairing cell signalling.
Some clinical examples:
Graves' disease
Myasthenia gravis
Type Alternative names Often mentioned Mediators
disorders
I Allergy (immediate) Atopy IgE
Anaphylaxis
Asthma

II Cytotoxic, antibody-dependent Autoimmune IgM or IgG


hemolytic anemia (Complement)
Thrombocytopenia
Erythroblastosis
fetalis
Goodpasture's
syndrome
Graves' disease
*see type V
explanation below
Myasthenia Gravis
*see type V
explanation below

III Immune complex disease Serum sickness  IgG


Arthus reaction  (Complement)
Systemic lupus
erythematosus
(SLE)

IV http://en.wikipedia.org/wiki/Type_IV_hypersensitivity Delayed-
Contact dermatitis T-cells
[2] [3]
type hypersensitivity (DTH), cell-mediated immune Mantoux test
memory response, antibody-independen Chronic transplant
rejection
Multiple sclerosis [4]

16
17

V http://en.wikipedia.org/wiki/Type_V_hypersensitivity Autoimmune Graves' disease IgM or IgG


disease, receptor mediated (see below) Myasthenia Gravis (Complement)

44- WHICH IMMUNOGLOBULIN IS PRESENT AFTER A BACTERIAL AND VIRAL INFECTIONS ?


IGG IS PRESENT AFTER BACTERIAL N VIRAL INFECTIONS
IGM IN EARLY INFECTIONS

45-Most abundant immunoglobulin in saliva during infections?


According to wikipedia its IgA, easy way to remember is most of the body excreted fluid are
IgA eg: tears, blood

46-which ig will increase in periodontitis ? IgG

47- which ig will increase in gingival inflammation? IgG

48- eosinophilis increase in which hypersensitivity ? IMMEDIATE TYPE 1


49- Ig present in mumps? IgM...IN EARLY AND IgG IN LATE

50- MOST COMMON VACCINE WITH INACTIVATED VIRUS? INFLUENZA


An inactivated vaccine (or killed vaccine) consists of virus particles which are grown in
culture and then killed using a method such as heat or formaldehyde.
Examples
Types include:
viral: polio vaccine (Salk vaccine) and influenza vaccine[1]
bacterial: typhoid vaccine, cholera vaccine, plague vaccine, and pertussis vaccine[1]
Inactivated vaccines are contrasted with Attenuated vaccines, or "live" vaccines. Examples
of "live" (i.e. attenuated) vaccines include:
Viral: measles vaccine, mumps vaccine, rubella vaccine, chicken pox vaccine, yellow fever
vaccine,[1] and nasal-spray flu vaccine (including the seasonal flu nasal spray and the 2009
H1N1 flu nasal spray). Rabies vaccines are now available in two different attenuated forms,
one for use in humans, and one for animal usage.
Bacterial: BCG vaccine,[1] typhoid vaccine

51-amalgam lichenoid reaction..which type of hypersensitivity-? type 4 delayed

52-Ig in anaphylaxisis? IgE


53-Lichinoid reaction what type of hypersensivity? Type 4

54-Abundant Ig in saliva? IgA

55-Which Ig will increase during gingival inflammation? IgG

56-Kiwi sensivity is? Type 1

17
18

57-Latix sensivity?
Mainly type 4 rarely type 1 check scully

58-contact dermatitis? type 4

59-mast cell are important in defence against?


a-forgien particles
b-worm infection

answer –b

60-what type of hypersensivity is ABO in bl tranfussion? Type 2

61-Which of these is not APC?


Macrophages
erythrocytes
bcells

Answer ..erythrocyte

62-What is the precursor of platelets? Megakaryocytes

63-Mmr is live or inactivated vaccine?? Live

64-J-chain is seen in which Ig? IgA.

65-hypersensitivity type 3 present in ?

Answer is sle and ra,serum sickness

66-In delayed hypersensitivity reaction the patch test should be checked after 48 hours

67-What type of hypersensitivity do you find in anaphylaxis? Type 1

68-What will you see in a patient with penicillin allergy? Skin rashes

69-Immunoglobin in reaction to perfume? IgE

70-WHAT TYPE OF HYPERSENTIVITY IS SEEN IN ORAL ALLERGY SYNDROME


TYPE 1
TYPE 3
TYPE 4
ANSWER IS TYPE ONE REFERENCE IS SCULLY

71-.Half-life of IgG is

18
19

a)21 days
b)10 days
c)15 days
d)3 days
answer is a

72-hypersensitivty in gingivitis and lechoid reaction? Both type 4

73-type of hypesinsitivity in recurrent auphthous ulcer? Type 4

74-organic specific auto antibody in which type? Type 4 or 5

Embryology and oral histology


1-bone which ossifies purely intramembraneous?

maxillae, nasal, ,lacrimal , frontal, partiel and vomer

2-structure formed purely from meckels cartilage?

malleous , incus , mandible ,sphenomandibular and sphenomalleous ligement

3.Which one is not developmental?

a-cross striations,
b-perikymata,
c-hunter-schreger,
e-Incremental lines of von ebner
answer is C

4-What is the first dentine?

Answer is the mantle dentine

5-theory of pain most accepted in tooth?

Hydrodynamic theory of dentine sensitivity

6-cells which form bone? osteoblast

7-diagram of tooth development to mark(most probably early bell stage )and their
developmental origin

8-What is the type of secretion of salivary glands mucous, serous?

Parotid is serous,submandibular is mixed and sublingual is mocuos

9-Development of parotid gland? 4-6 weeks

19
20

10-Development of tooth, all stages be very thorough questions were asked in a very
tricky manner and also do the the weeks properly .which stage when? Atleast 5 to 6
questions

11-Calcification of primary and secondary teeth, their time of development and their
eruption be very thorough.

12- There were questions on cementum, enamel, and dentin mainly composition?

Enamel :96% of inorganic hydroxyapetiate ,tyrosin rich amelogenine protein,trace of


enamiline and water form 4%

Denine:70% of hydroxyapatite,20% of plasma protein,glycoprotein,phosphoprotein,collagen


type 1 and proteoglycan and 10% of water

Cementum:50% hydroxyapatite and organic formed of 95% type 1 collagen and 5% type 3
collagene

13- Which cells are more in pdligament? The fibroblast

14- Calcification and eruption dates of primary and permanent dentition

15- Bud stage.cap stage.bell stage

16- Development of the parotid gland begins in utero at what month? 4-6 weeks page 305
tencate

17- Bud stage of tooth development begins at? 8 weeks of utero

18- Dates of calcification and eruption of various permanent teeth were asked?

19-Bud stage of upper 1st permanent molar begins at? 24 week

20-Which dental hard tissue is 45% mineralized and resorbs slower than bone

a-Enamel

b-dentine

c-cementum

answer is c

21- What does the transcluscent/sclerotic zone in dentine represent? Intratublar dentine…
check

22- oral histo about enamal and pulp development

23- which is most important function of saliva? lubrication

20
21

24- Type of cell that resorbs bone.

a-osteoblast,

b- ostoclast,

c- odontoblast,

answer is b

25-Type of cell that forms bone? osteoblast.

26-. Origin of

A- cementum, 1- dental follicle,

B-PDL 2- stellate reticulum,

C- pulp, 3- dental papilla

D- enamel, 4- HERS

E- root, 5- IEE

a-1,b-1,c-3,d-5,e-4

27-.Fusion time for anterior fontanelle. options:

a- 6 months,

b-12 months,

c-24 months

d-18 month

the anteriofrontanelles fuse at the age of 18 months after birth

The posterior frontanelles and sphenoid fuse 3 months after birth

The mastoid frontanelless fuse at the age on one year

28-. Width of periodontal ligament, fibres and type of collagen?

11-16 years old its 0.21mm,32-52 is 0.18mm,and 52-70 is 0.15mm,type of fibers is


collagen ,oxytalan and eluanine,type of collagene fibers include type1 55% and type 3 45%.

29- Which one is responsible for buffering capacity in saliva? HCO3

30- Histology – structures of erupting tooth

21
22

31- Periodontal ligament of adolescent in comparison to adult?

a-Similar,

b-thinner,

c- wider

answer is c

32- Collagen in periodontal ligament: predominant ? Type 1 55%

33- Histological cut of tongue

34- Type of collagen most common in PDL? Type 1

35- Type of collagen second most common in PDL? Collagen type 3

36-where is the PDL the thinnest?

In the middle third of the root

37- PDL in adolescents as compared to adults? It is wider

38- The direction of most of the n PDL fibres is? oblique

39- PDL fibres are attached to the? cementum

40- Picture of an embryo marked with maxillary process & lateral nasal process

41- Tissue originated from ectoderm:

a- dental lamina

b- dental papilla

c-dental pulp

d- cementocytes layer

e- dental follicle

answer is a

42- Odontoblasts differentiate from:

a-Internal enamel epthelium,

b-Stellate reticulum

c- dental papilla,

22
23

d- dental follicle

answer is c

43- Origin of internal auditory meatus and middle ear

a- 1st, 2nd branchial arch

b- 3rd branchial arch,

c-1st, 2nd pouch

d- 3rd branchial pouch

answer is c

44- Type of receptors in the PDL of a dentate adult?

Mechanoreceptor or neoreceptor

45- What produces alkaline phosphatase that contributes to enamel formation?

a-OEE

b- IEE

c- Stellate Reticulum

d- Stratum Intermedium

answer is d

46- Where is the narrowest part of the pulp:

A. At the radiographic apex

B. At the dentino-enamel junction

C. At the orifices

D.At the dentocementum junction

ANSWER IS A

47- In cavity preparation 1mm below DEJ what is seen:

A. More dentinal tubules, some intertubular and peritubular

B. Some dentinal tubules, more intertubular and less peritubular

23
24

C. More peritubular, some intertubular and dentinal tubular

D. Equal amount of dentinal tubules, intertubular and peritubular

-B because it is under the cavity preparation there will be deposition of tretary


dentine which is an irregular dentine and contain few dentinal tubules.rapid produced
tretary dentine produces very few dentinal tubules which is termed as osteodentine

48-How does sympathetic and parasymathatic nerve supply affect the salivation?
parasympathatic works at rest and saliva is hypotonic

Sympathetic works during stimulation ,quantity increases and saliva is less


hypotonic more HCO3 and NACL

49-what are the bones that develope intramembraneous? Maxillae


,nasal,lacrimal,frontal,paretial,vomer

50-what are the bone that develop intramabraneous and endochondral?

mandible,temporal ,occipital and sphenoidal

51- what are the bone that develop endochondral?? hyoid,inferior nasal conchae and
ethmoidal

52-.Failure of closure of vertebral arches? Spina bifida

53- Which cells are usually not seen in a healthy pulp? The b lymphocytes

54- Predominant fibres in periodontal ligament:? Oblique

55- Width of periodontal ligament? Between 0.21-0.15mm

56-In periodontal ligament where are the fibres thinner? Middle part of the root

57-What it is passive eruption ?

passive eruption is the apparent lengthening of the crown due to the loss of attachment, or
recession of the gingiva.

58- External acoustic meatus is derived from?

a)1.pharyngeal-pouches
B)1.pharyngeal-clefts
C) 2.pharyngeal arches

answer is b

24
25

PHYSIOLOGY:
1-Mediators of coagulation what happens on the first stage?

The process starts with contraction of the blood vessels which brings the ednothilial ends of
the opposing parts of the injured site to stick together.this is ollowed by the activation of
platelet and aggrecation to form a platelet plug. Then the formation of the blood clot.

2-the percentage of deoxygenated blood coming of of lungs?

Deoxygenated blood coming from the ling is around 10-5% ,deoxygenated blood going into
the lungs is 20-40%

3- Stroke volume and respiratory volumes.

Stroke volume =70ml/min

Respiratory volumes include the following;

Tidal volume=500ml

Residual volume=1200ml

Expiratory reserved volume=1200ml

Inspiratory reserved volume =3000ml

Functional residual capacity=2400ml

Vital capacity=4700ml

Total lung volume=5900ml

4- Hormones produced by pituitary, kidney and hypothalamus

5- Adrenal cortex hormones?

Glucocorticoid(cortiso;)act in stress response and immune development

Androgen .....stimulate female sex drive

Mineralcortocoid(aldosteron).....regulates the blood pressure and secretion of the water by


the kidney

6- what is Noradrenaline ?

It is a catecholamine produced by chromaffin cells of the adrenal medulla thats works in


stress and increases blood pressure

7- Which of the following options given is a typical exocrine gland:

25
26

  a-sweating gland

b-pitutary

c-liver

answer is a

8-which of these is important determinant of vessel resistance?


a-length
b-radius
c-viscocity of blood
answer is b

9-Which is the acid produced in the stomach? HCL

10-Pituitary gland, hormone of middle part? Melanocyte stimulating hormone

11-Tidal volume – definitions?

It is the amount of air inhaled and exhaled at rest ...500ml

12-Hormone produced by the islet of Langerhans by the beta cells? Insulin

13- Hormone produced in the adrenal cortex & which increases in stress ?

Glucocorticoid (cortisole)

14- What hormone increases blood glucose and potassium?

Glucagon,

Cortisol,

growth hormone

Answr is cortisol

15-what are the hormones sectered from the the pancrease langerhans cells ?

Beta cells....insulin

Alpha cells...glucagon

Delta cells....somatostatin

26
27

16- The average respiratory rate for adults and for children?

adults respirotary rate =12 breath/minute


Children 6-12 is =12-20
Toddlers3-6 is= 20-30

17- Valve between right atrium and right ventricle ? Tricuspid valve

18-“All-or-nothing” phenomenon? action potential

19- Potential at the SA node; options-

a-saltatory,

b-pacemaker,

answer is b

20- Substance controlling Ca2+ metabolism? Parathyroid hormone

21- Definition of lung volumes?

vital capacity:it is the maximum amout of air that can be exhaled with effort after a
maximum inhalation(IRV+ERV+TV)=4700ml

inspiratory reserve volume:it is the maximum amount of air that can be inhaled in excess to
tidal volume=3000ml

expiratory reserve volume:is the maximum of air exhaled under effort in excess to tidal
volume=1200ml

22- Which vessels:

Dilate during vasoconstiction?

Play a role in thermoregulation?

Carry filtered fluid?

Is more elastic?

Carry more volume of blood

Have more smooth muscle

Small arterioles,

terminal arterioles, venules,

27
28

arteriovenous shunts,

capillaries,

lymphatic vessels

arteries

veins

answer is vasodilation is the small arteriols

thermoregulation is the capillaries

carry filtered fluid is the lymphatic vessels

most elastic is arteries

carry most blood is veins

with most smooth muscle is the arteries

23 which clotting factor is associated with the following conditions?

a-xmas disease

b-von williebrands disease

c-haemoghilic A

d-dissemenated intravascular coagulopathy

e-vitamin k deficiency

f-haemophilia c

answer a-3,b-6,c-1,d-5,e-4,f-2

24-What is the type of neurotransmitter responsible for closing the precapillary sphinctor?

a-acetylcholine

b-serotonin

c-nor-adrenaline

d-substance p

answer is c

28
29

25-EFFECT OF EXERCISE ON SYSTOLIC PRESSURE

~~~~~~~~~~~~~~~~~ON DIASTOLIC PRESSURE

~~~~~~~~~~~~~~~~~ON TOTAL PERIPHERAL RESISTANCE

On cardiac output

Systolic increases ,diasltolic remains unchanged in normal conditions increase in patient


with coronary diseases and decrease in athletics and total peripheral resistance is
decreased,increase in the cardiac output.....

26- Chronotropic,ionotropic definitions and what does adrenaline do?

27- Different values of FBC were given and type of anaemia was asked

28- Glucose levels during fasting and random

29- corticosterone?

Hormone secreted from the adrenal cortex during stress and used for metabolism of
fat,protein and carbohydrate

30- If gland on one side are stimulated what will the response be like?

a-iplister

b-bilateral

Answer is ipilester means the other gland will not be stimulated

31-. After what percentage of blood loss should transfusion be given?

Transfusion should be given if blood loos is between 20% pink book is the referecen

32- jugular venous pressure is best described as(the choices may not be accurate, they were
something like this:)

a-Pulse pressure

b-10 mmHg more than ventricular Pressure


c-10mmHg less than ventricular pressure

d-20mmHg less than ventricular pressure

29
30

e-20mmHg more than ventricular pressure

answer is c JVP is 6-8 cm H20 while VVP is 6-12 cm H2O.

33-.Pulse is better got from which artery?

Carotid artery at the side of the neck and radial artey at the wrist

34- Clotting factors and formation of clot ?

Prothrombin converts toThrombin

Fibrinogen, converts toFibrin,

Factor XIII which stabelises the fibrin stabilising

35-O2 pressure in the alveoli ? 105 mmHg

36-what is the neurotransmitter in vagus nerve?

Acetylcholine,,,,vagus is onlt parasympathatic

37- blood ph =

option were:6.8 ,

7,

7.4 ,

7.6 ,

amswer is 7.4

38- the immunoglobulin in saliva ? IgA

39-.seated healthy man 70kg what is the blood presure /

Options

a- 120/80
b- 140/100
c- 120/100

30
31

d- 140/110
e- 120/60

answer is a

40- Sodium and calcium channels?

Calicium channel are long lasting channel

Sodium channels are rapid channel

K channels are slower than Na channels

41- Action of glucagon ?

1)increases glycogen break down into glucose(glycogenolysis)

2) increases gluconeogenesis

3) increased sysnthesis of ketone and ultimately raises plasma glucose levels

Answer is all of the above.

42- Stroke volume how do you measure it?

Stroke volume =End diastolic volume –end systolic volume

43 What hormone increases blood glucose and what decreases it? Insulin decreases blood
glucose and glucagon increases it

44- .Machine used to measure lung functions? spirometer

45- Normal value of glucose in fasting

Answer is 4-6

46- What ion is factor 4 clotting factor? calcium

47-what is Glycogen?what sugars are monosacchride and which are disaccharide?

Glycogen is a polysacchride molecule that store glucose

Monosaccharide;glucose ,fructose and galactose

31
32

Disacchrides(lactose=glucose+galactose ,maltose= glucose+glucose,


sucrose=fructose+glucose)

Polysacchrides (starch,cellulose,glycogen)

48- Which glands secrete hormones into ducts?

Exocrine glands..Exocrine glands are glands that secrete their products (excluding hormones
and other chemical messengers) into ducts (duct glands) which lead directly into the
external environment. Exocrine glands are named apocrine gland, holocrine gland, or
merocrine gland based on how their product is secreted.

Apocrine glands - a portion of the plasma membrane buds off the cell, containing the
secretion,an example is fat droplet secretion by mammary gland.

Holocrine glands - the entire cell disintegrates to secrete its substance,an example is
sebaceous glands for skin and nose.

Merocrine glands - cells secrete their substances by exocytosis an example is pancreatic


acinar cells.

Typical exocrine glands include sweat glands, salivary glands, mammary glands, stomach,
liver, pancreas

49- what is the Membrane potentials, resting membrane potential, action potential,
refractory, generation and propagation of action potential?

Membrane potential:it is the difference in polarity across the membrane.

Resting potential;steady transpotential of a cell that does not produce an electrical signal
measures around -70mv .

Action potential;all or none depolarization of the membrane

Refractory ;Na voltage channels already opened or in the inactive phase.

Generation :opening of the Na voltage channels

Propagation;opening and closing of sodium channels along the membrane

50- Which gas is inhaled from cupped hands? Co2

51- In hyper-ventilation, which gas has high concentration? O2

52- Factor VIII is reduced in ? Haemophilia A and vWD.

53-  Which cell is responsible for carrying oxygen & nutrients? Red blood cell

54- iron combines irreversibly with what? oxygen

32
33

55- Anatomy of the heart ?

The heart contains two atrioventicular valves the tricuspid between the right atria and
ventricular ,the bicuspid between the rleft atria and ventricle. It also contains two semilunar
valves between the aortic valve between the aorta and left ventricle and the pulmonary
valve between the pulmonary artery and the right ventricle

56- Which acid is involved in gastric reflux disease? HCL

57- Which enzyme acts on fats? lipase

58- Which enzymes acts on proteins in the GIT? Pepsine from the stomach,trypsine from the
pancrease and chymotrypsine from the small intestine

59- What is the amount of gas in the lungs at the end of tidal volume?

Functional residual capacity…amount of air in the lung after a normal tidal volume
expiratory 2400ml

60- cells in the alveoli at the site of gases exchange?Alveolar type 1

61- Type of epithelium lining the upper respiratory track? pseudostratified columnar ciliated
epithilum with gobelts cells

62- Gas released during fat and carbohydrate metabolism? Co2

63- Functions of different components of saliva: enzymes, mucins, proteins,.what is the role
of NaCl in saliva?role of HCO3? What makes saliva, isotonic, hypotonic?

Enzyme in saliva include:amylase for carbohydrate metabolism

Lysosome antibacterial properities

Lactoferrin;antibacterial chelates with iron

Sialoperoxidase:antibacterial

Mucin ;MG1,MG2

Protein:proline rich protein,agglutinins,IgA,

Blue book page 366

Role of NaCl is to taste salt

Role of HCO3 is buffering

64- What stimulates salivary production in dogs that doesn’t stimulate it in humans?

65- What does peripheral arterial pressure measures? 100mmHg for O2 and 40mmHg for
CO2

33
34

66-Heart rate in a healthy young male individual during long period of exercise? 108-153

67- Male suffered trauma and lost almost 1 lt of blood. What is the reference blood volume
per kilogram:

70ml,

120ml,

150ml,

220ml

Answer is Reference blood volume per kilogram in humans is 77ml ( so I guess d and sh'd b
70ml !).Chk blood volume in wiki ( tabular column

68- Coagulation factor that creates a “complex” with tissue factor to degradate prothrombin

Tissue factor and factor 7

69- Questions about Beta and Alpha cells

Secretion of insulin and amylase

Alpha cells secret glucogen

70- Questions about “chief cells and partiel cells in the stomach?

Cheif cell of the gastric gland in the stomach secrets pepsinogen

Parietal cells secrect hydrochloric acid and intrinstic factor

Enteroendocrine(gastric cell) secrets gastrin

Mucous cell secret mucous

71- Muscles responsible for expiration during exercise? intercostal muscles,abdominal


muscles.

Cells associated with viral infection and bacterial infection ?


Viral t-lymphocyte
Bacterial neutrophil

72- Direct precursor of plasma cell,macrophages?

Plasma cell is b-lymphocyte cells

34
35

Macrophages are from monocytes

73- What is the rate of respiration per minute?

For healthy adult it is between 12-20 breath per minute

Adults during steneous exercise 35-45 breaths/minute

Athletes at peak 60-70breath /minute

New borns 44 breath /minute

Infants 40-60breath/minute

Preschool chidren 20-30breath/minute

Older children 16-25breaths/ minute

74- Production and mechanism of action of ADH

Impulses are send to the brain and htpothalumus to secrect ADT and store it in the posterior
pituitary gland and released from there in response to increased blood
concentration,decreased blood volume,increased blood pressure and angiotensin 2.
Reference youtube the khan acadenmy

75- factors deficient in haemophilia A ,B

Haemophilia a-factor 8

Haemophilia b-factor 9

76- Questions about acidosis and alkalosis ,alpha and beta receptors, chronotropic
,ionotropic in heart(like which is rhythm),epinephrine and acetyl choline

Heart muscle is an ionotrophic

77- .Lots of histology-striated muscle....cardiac and skeletal


http://www.lab.anhb.uwa.edu.au/mb140/corepages/muscle/muscle.htm#CARDIAC

78. Infective Endocarditis affects which valve? Mitral valve

79- About fluid loss and effect on venous return

35
36

Due to blood loss there will be a decrease in the stroke volume ,cardiac out put and mean
atrial pressure ….this is correct but not to normal values by the detection of baroreceptors
reflux, an increase of the peripheral restistance and heart rate due to the sympathetic reflex
in an attempt to correct the venous return,, this is followed by fluid movementfrom the
interstaial fluid to the blood due to the effect of increase vasoconstriction and decrease the
hydrostatic capillary pressure . blood loss is corrected in moderate haemorrhage around 12-
24 hours

80- neutrophilia is seen in?

Is seen mainly in bacterial infection,under stress or due corticosteroid treatment such as


prednisone

81- partial oxidation of haemoglobin with combustion products

82- Percentage of blood that can be lost without causing hypovolemia-

options-

a-5%,

b-10%

c-20%

d- 40%,

e-50%

answer is a cos hypovolemic shock happens when there is blood loss between 10-15%
(blue book page 497)

83- Question on which ion is released when there is a low gastric ph? HCO3 and h

84- What is the kind of epithelium lining exocrine ducts? Stratified sequamous epithelium
(grandular epithilum)

85- Vagus is sympathetic or para for Heart? parasympathetic

86- What part pituitary partially controls the adrenal ?

The anterior pituitary gland by the secrection of adrenocorticotropin hormone which


regulates the secretion of hormones from the adrenal cortex.

87- Slowest voltage gated channel?

Potassium channels check answer some say its Ca

88- What increase with age? Systole pressure

36
37

89- What affects cardiac output? Stroke volume and the heart rate

90-Which vessel plays a role in thermal regulation


Venues
Capillaries
Arteries
Av shunts

Answer is capillaries

91- What is the commonest cause of Cushing’s disease and Addison disease

a-Adrenal disease

b-Pituitary adenoma

c- decrease in the ATCH due to surgical removal of part of the pituitary gland

answer to cushing disease is b due to increase in the ATCH secretion mainly due to pituitary
gland adenoma ,Addison disease is c

92- what is the increae in pco2 and increased H?


Respiratory alkalosis
Respiratory acidosis
Metabolic alkalosis
Metabolic acidosis

Answer is respiratory acidosis

93-what increases pulse rate during exercise?


Systolic pressure
Dystolic pressure
Cardiac out put

94- Which part of the nervous system causes increased salivary secretion ?
Sympathetic
Parasympathetic
Both

Both increases but in parasympathetic more

95- hormone which increases Na and water reabsorption? Mineralcorticoid(aldosteron)

96- primary function of saliva? Lubrication

97- RBC increased in? Polycythemia

37
38

98-.xerostomia is seen in? Sygorjen syndrome,after radiotherapy.

99- What is the type of secretion of salivary glands mucous, serous e?c Submandibular is
mixed,

parotid is serous

sublingual mucous ,

other minor salivery glands is mixed

100- where is resting saliva is mostly from? Which gland give more than 2/3 of saliva when
stimulated?

Answer is Submandibular gland and second answer is parotid gland

101- which one is haemoglobinopathy? both thalassemia and sickle cell were in options

102- catecholamine produced by adrenal medulla thats works in stress and increases B.P?
Noradrenaline, adrenaline.

103- types of connections between cells?


Gap junction:between cardiac mucsle to allow the transsion of the action
potential(connexin)
Desomosomes:between skin cell to allow elasticity(cadherins protein)
Tight junction between epithelial cells

Desmosomes :- consist of a disk shaped region between two adjacent cells where
apposed plasma membranes are separated by about 20 nm and have a dense accumulation
of protein at the cytoplasmic surface of each membrane and in the space between two
membranes.Function to hold adjacent cells together in areas that are subject to
considerable stretching such as in skin.

Tight Junctions:- formed when extracellular surfaces of two adjacent plasma membranes
are joined so there is no extracellular space between them. Unlike the desmosome which is
limited to a disk shaped area of the membrane, the tight junction occurs in a band around
the entire circumference of the cell.Most epithelial cells are joined by tight junctions e.g
epithelial cells covering the inner aspect of intestinal tract.Only ions and water can move
with ease.

Gap junctions :- Consists of protein channels linking cytosols of adjacent cells.In the region
of gap junction,the two opposing plasma membrane come within 2-4nm of each
other,which allows specific proteins from the two membranes to join forming small protein
lined channels linking the two cells.Small diameter (1.5 nm) limits movements of different

38
39

substances to small molecules and ions such as Na and K. Muscle cells of the heart and
smooth muscle cells.Also co ordinate the activity of adjacent cells by allowing chemical
messengers to move from one cell to the other.

104- Which is the most abundant Extracellular cation? Na

105- Jugular venous pressure indicates what pressure? Central venous pressure

106- what is the extracellularly and intracellularly ions what ions responsible for action
potential and what channels does lignociane block? How much is the potential of Excitatory
and Inhibitory Synaptic?neurotransmitters liberation process is by?

Extracellularly:Na (150),

Cl (110)

K (5)

Intracellularly Na (15),

Cl (10)

K (150).

Na and K responsible for action potential. Lignocaine block the voltage gated Na channels
and prevent them from depolarization

The Excitatory synape is -70mv

Inhibitory synapse is +70mv

Neurotransmitter released by Exocytosis

107-compostiosion of Thick muscle fiber thin muscle fiber ?

Thick muscle composed of myocin and thin muscle made of actin,troponin and tropomyocin

108- what is a A-band?

A thick filament area in the middle of each sacomare between two z lines they are
arranged orederly parallar to produce a dark band

109-what is the H zone ?

It is a light area in the centre of the A band it corresponds the end of two opposing thin
filament.

39
40

110-what is the I band?

It is a light band that lies at the end of each sarcomere for the end of band A, contains those
thine filaments that are not overlapped by thick filament they are joined by a z line.

111-what is the M line?

It is a dark line in the middle of H band represent a protein that links the center parts of
thick filaments together.

112- Function of intermediate zone of adrenal?

It secret glucocorticosteriod (cortisole) during stress and to mantain the metabolic process
of charbohydrate ,fat and protein

113-what are the band A made of? Myosin

114-what protein in the thin filament of a muscle combines with the Ca++ from the
sacroplasmic reticulume?

The calcium combines with the troponine and due to that binding the tropomycin filament
is removed to expose the underlying actin protein which bind with the myosin in the thick
filaments

115-what protein the muscle fiber have subcomponent to it?

Troponin protein has three types C I T,C binds to Ca to bring conformation change in
tropomycin ,T hinds to tropomycin to hold tropomycin-troponin in place, and I binds to the
actin thin filament to keep the troponin-tropomycin in place.

116- what is the nuetrophil family? Granulocyte

117- during what infection is the nuetrophil present the most?

a-Bacterial,

b-viruses,

c- fungi,

40
41

answer is a

118- In which blood cell is the nucleus lost in the early stages? Erythrocytes

119- Precursor cells of platelets? Megakaryocytes

120- What ion acts as a second messenger? Calcium ion

121- Which anion is present in bone?

Phosphate and bicarbonate and both are negatively charged.

122- Hemoglobin combines irreversibly with what?

Carbon monoxide binds allosterically with HB and modifies it.Thats why in CO toxicity
person goes all pink

Do the O2 dissociation curve then you will not have problem answering the questions! as
there were questions related to it.

123-.What part of the nervous system is responsible for fight or flight? Sympathetic

124-What part of the nervous system causes increased salivary secretion?

Both sympathetic and parasympathetic increase the secretion of saliva , however the amout
secereted under parasympathetic influence is more than the sympathetic (remember when
you panic your mouth goes dry)

124-What part of the nervous system increases heart rate? Sympathetic

125- Blood cells-types of anemia?

Type of blood cell are the erythrocyte

Platelets,white blood cells which include the monocyte,


granulocyte(neutorphil,eiosinophils,basophilis)monocytes and lymphocytes

126-HB range in males and females ?

Females 11.5-16.5g/dL
Males 13-18G/dl

41
42

New born 17-22


One week old 15-20
One month old 11-15
Older children 11-13
Men after mid age 12.9-14.9
Women 11.7-13.8

127-What are the types of anemia?


First we should always remember these figures:
The mean cellvolume which ranges between(76-96Fl)
Haematocrit is 40-54% for males and 37-47% for males
According the these numbers anemia can be divided into the following:

A-microcyte hypochromatic anemia M.C.V is less than 80Fl and haematocrit is less
than 27 this type of anemia include iron deficiency anemia and thalassemisa

B-normocytic normochronic anemia;M.C.V is between 80-96fl and haematocrit is more than


27pg. This include blood loss,haemolytic anemia,aplastic anemia,G6PD anemia and sickle
cell anemia

C-macrocytic normochromic anemia:M.C.V is more than 95 fl and the haemtocrit is more


than 27pg this includes B12 difceincy ,prencious anemia,folate deficiency and megaloblastic
anemia

128- Lack of what vitamin causes sub acute degeneration of the spinal cord?

a. Vit b 12-

b. Thiamine

c. Vit d

answer is a

129-Which of these is a important determinant of resistance in vessels ?


Length
Radius
Viscosity of blood

Answer is radius vander pager 363

103-Which sends impulses faster?


Myelinated
Unmyelinated

Answer is mylenated

42
43

104-Saliva in relation to plasma is


Isotonic
Hypotonic
Hypertonic

Answer is hypotonic reerence in slideshare

105-Which of d following nerve fibres r responsible for pulp all pain transmission?
A beta & c nerve fibres
A delta & c nerve fibres
A beta & A delta nerve fibres

Anwser A delta nerve fibers and c fibers


Types of nerve fibres
A- alpha - proprioception
A-Beta - touch
A-Delta - pain ( myelinated)
C - pain ( non myelinated)

106-Whichof this is not a neurotransmitter?


ATP
Glycine
Enkephalins
Histamine

Answer is enkephalins

108-Muscle spindle receptors r seen in


Jaw elevating muscles
Jaw depressing muscles

Microbiology
1-Which is not a component of gram +ve cell wall:

a-peptidigylcan

b-lipopolysaccharide

c-n acetyl muramic acid

d-trichroic acid

b-cos lipopolysaccharide is an exotoxinwhich is present in gram

2- ELISA test used commonly for the diagnosis of which disease?

43
44

Because the ELISA can be performed to evaluate either the presence of antigen or the
presence of antibody in a sample, it is a useful tool for determining serum antibody
concentrations (such as with the HIV test[8] or West Nile Virus). It has also found applications
in the food industry in detecting potential food allergens such
[9]
as milk, peanuts, walnuts, almonds, and eggs.  ELISA can also be used in toxicology as a
rapid presumptive screen for certain classes of drugs.

ELISA (Enzyme-Linked Immunosorbent Assay) plate.

The ELISA was the first screening test widely used for HIV because of its high sensitivity. In
an ELISA, a person's serum is diluted 400-fold and applied to a plate to which HIV antigens
are attached. If antibodies to HIV are present in the serum, they may bind to these HIV
antigens. The plate is then washed to remove all other components of the serum. A specially
prepared "secondary antibody" — an antibody that binds to other antibodies — is then
applied to the plate, followed by another wash. This secondary antibody is chemically linked
in advance to an enzyme.
Thus, the plate will contain enzyme in proportion to the amount of secondary antibody
bound to the plate. A substrate for the enzyme is applied, and catalysis by the enzyme leads
to a change in color or fluorescence. ELISA results are reported as a number; the most
controversial aspect of this test is determining the "cut-off" point between a positive and a
negative result.
A cut-off point may be determined by comparing it with a known standard. If an ELISA test is
used for drug screening at workplace, a cut-off concentration, 50 ng/mL, for example, is
established, and a sample that contains the standard concentration of analyte will be
prepared. Unknowns that generate a signal that is stronger than the known sample are
"positive." Those that generate weaker signal are "negative."
Doctor Dennis E Bidwell and Alister Voller created the ELISA test to detect various kind of
diseases, such as Malaria, Chagas' disease, and Johne disease. [10] ELISA tests also are used as
in in vitro diagnostics in medical laboratories. The other uses of ELISA include:

 detection of mycobacterial antibodies in tuberculosis.


 detection of rotavirus in feces.
 detection of hepatitis B markers in the serum.
 detection of enterotoxin of E. coli in feces.
Percentage of population having type 1 hypersensitivity – 20 – 30%

947. Microorganisms

Streptococcus mutans group Several species are recognized within this group, including S.
mutans and S. sobrinus. Aerobic. Synthesizes dextrans. Colony density rises to >50% in
presence of high dietary sucrose. Able to produce acid from most sugars. Most important
organisms in the aetiology of caries.

44
45

Streptococcus oralis group includes S. sanguis, S. mitis, and S. oralis. Account for up to 50%
of streptococci in plaque. Heavily implicated in 50% of cases of infective endocarditis.

Streptococcus salivarius group Accounts for about half the streptococci in saliva.
Inconsistent producer of dextran.

S. intermedius, S. angiosus, S. constellatus (formerly S. milleri group) Common isolates from


abscesses in the mouth and at distant sites.

Lactobacillus Secondary colonizer in caries. Very acidogenic. Often found in dentine caries.

Porphyromonas gingivalis Obligate anaerobe associated with chronic periodontitis and


aggressive periodontitis.

Prevotella intermedia Found in chronic periodontitis, localized aggressive periodontitis,


(juvenile periodontitis), necrotizing periodontal disease, and areas of severe gingival
inflammation without attachment loss.

Prevotella nigrescens New, possibly more virulent.

Fusobacterium Obligate anaerobes. Originally thought to be principal pathogens in


necrotizing periodontal disease. Remain a significant periodontal pathogen.

Borrelia vincenti (refringens) Large oral spirochaete; probably only a co-pathogen.

Actinobacillus actinomycetemcomitans Microaerophilic, capnophilic, Gram -ve rod.


Particular pathogen in juvenile periodontitis and rapidly progressive periodontitis.

Actinomyces israelii Filamentous organism; major cause of actinomycosis. A persistent rare


infection which occurs predominantly in the mouth and jaws and the female reproductive
tract. Implicated in root caries.

Candida albicans Yeast-like fungus, famous as an opportunistic oral pathogen; probably


carried as a commensal by most people.

Spirochaetes Obligate anaerobes implicated in periodontal disease; present in most adult


mouths. Borrelia, Treponema, and Leptospira belong to this family.

QUESTIONS ABOUT WHEATEAR IT 'S DNA OR RNA VIRUS

1- HEPATITIS A
2- HEPATITIS B
3- HEPATITIS C
4- HIV
HEPATITIS B IS DNA REST ARE RNA

45
46

1- RNA NON - ENVELOPED  


2- DNA DOUBLE SHELL STRUCTURE / ENVELOPED  
3- RNA ENVELOPED
4- RNA OR RETROVIRUS  

. BACTERIA THAT CAUSE URINARY TRACT INFECTION?


E COLI
N GONORRHOEA  
TREPONEMA PALLADIUM  
CHLAMYDIA 

BACTERIA THAT MOST COMMONLY CAUSE INFECTION IN DIGESTIVE SYSTEM IN UK?


CAMPHYLOBACTER

. THE COMPONENT OF BACTERIA THAT ATTACHES TO TOLL PROCESS:


A . PROP IONIC ACID
B. LIPOPOLYSACHHARIDE
ANSWER: B

150.QUESTION OF CANDIDIASIS WHICH IS CAUSED IN PARTICULAR GROUP /RACES(DONT REMEMBER THE


EXACT ETHNIC GROUP )
A .C.TROPICALIS
B.C.ALBICANS

157. COMMONLY USED TREATMENT OF CANDIDIASIS:


A . FLUCANOZOLE
B. MICANAZOLE
C. NYSTATIN
ANSWER: B

158- WHICH VIRUS IS NOT A HERPES VIRUS?


A -VARCELLA ZOSTER
B -EPESTEEN BARR VIRUS
C -HERPANGINIA
E -CYTOMEGALOVIRUS

ANSWER IS C ,BECAUSE ITS A COXSAKIE VIRUS


REMINDER .. HERPES VIRUSES ARE ENVELOPED DOUBLE STRAND DNA

46
47

Oral medicine
1-

Answer is E

47
48

2-

48
49

49
50

5- the highest chance of malignant transformation?


A-erythroplakia
b-speckled leukoplakis
c-oral submucosal fibrosis

6--REDUCED SALIVARY FLOW REASON ?


A-XEROSTOMIA
B-MUMPS
C-SALIVARY CALCULUS
ANSWER IS A

7- WHICH OF THE FOLLOWING CONTAIN CLEAR FLUID FILLED BLISTER ?


A-PEMPHIGUS
B-MUCOUS MEMBRANE PEMPHIGOID
C-COELIAC DISEASE ...
ANSWER IS A..... PEMIPHIGUS BLISTER CONTAIN CLEAR FLUID ,MUCOUS MEMBRANE PEMOPHGIOD IS
MIXED WITH BLOOD ,CELIAC DISEASES DOESN’ T HAVE BLISTERS

8-where does the mandible deviate in condylar hyperplasia,condylar hypoplasia and


ankyloses?
Hyperplasia deviated to the collateral side
Hypoplasia to the ipsilateral side
Ankyloses to the contralateral side in unilateral if bilateral no effect

9-Which leukoplakia is most likely to turn malignant?


a-erossive
b-ulceratived

50
51

c-speckled

answer is c cause erythroluekoplakia is a speckled leukoplakia

10-herpis labialis is caused by ?

Answer is ,,,,Herpes simplex virus 2

11-which disease is characterized by cobblestone mucosa?


Answer is crohn’s disease and orofacial granulomatosis
Sign of oral manifestation o crohn’s disease includes; lip and cheek swelling ,buccal mucosal
tags,full width gingivitis,cobblestone,angular chelitis ,RAU and vertical issur of the lip

12-which leasion has increased risk of malignancy?


Answer is erythroplakia

13- manifestation of congenital syphilis?


Hucheston incisors,mulberry molars,frontal bossing and flat nasal eminence

14-type of pain in trigeminal neurolagia?

Sever paroxysmal pain lasting for seconds in the branches of one or more of the trigeminal
nerve specially the maxillary and mandibular branch and only 5% affecting the ophthalmic.
Pain is described as an electrical shock, stabbing, lacerating or piercing which may be
associated with a trigger zone either intra or extra orally. Patient might avoid shaving or
washing that area to avoid provoking the pain.

15- causes of angular chelitis?

Anaemia
Skin infections (fungal)
Candidial infection.....candida albican
Bacterial infection...staphylococcus aures and beta haemolytic streptococcus
Nutritional deficiency such as iron ,folate and vitamine B12
Truma
Inadequate in the vertical dimension of denture.
Immunocmpressed patient such as HIV

16-what is amelogensis imperfect?


Is a group of conditions that are either autosomal dominant inheritance, recessive
inheitance or X-linked. It is characterised by enamel hypoplasia, enamel hypo mineralized
,hypomaturation

17- in which salivary gland tumor the malignant spread around the facial nerve?
a-acinic cell adenoma

51
52

b-mucoepidermoid carcinoma (can occur at any age).


c-adenoid cystic cancinoma

answer is c

18- patient with butterly rash?

Systemic lupus erythromatoues or chronic discoid lupus erythromateous

19-patient with gingival enlargement?

20-karposis sarcoma is associated with what virus?

Associated with human herpes virus 8 HHV8

21-BECHET disease?
Recurrent aphthous ulcer
Genital ulcer
Uvietis

22-causes of aphthous ulcer?


Stress
Nutritional deficiency such as iron,vitamine B1,B6 and B12
Trauma
Systemic diseases such as,crohn’s disease,ulerativie clositis and behcet disease
Endocrine ..during the luteal phase of menstrual cycle for females
Allergies
Infection
Smoking ...is a negative effect of cessation of smoking

23- virus causing shingle and treatment?


Virus is herpes zoster affecting the sensory gongilia leading to vesicular eruption affecting
the skin supplied by the nerve.
Treatment systemic high dose of systemic aciclover 800mg*5 a day for 7 days.

25-desquemative gingivitis is a character of?


Mucous membrane pemiphigoid and pemiphigus valgaris. Also in Lichen planus.

26-What is frey syndrome?


Is a skin gustatory sweating and flushing when parotid gland is activated to secret saliva due
to an injury to the parotid gland after truma or surgery leading to the cross interaction
between the sympathetic and parasympathetic innervations

27-what is sacrodiosis?

52
53

Is a granulomatous multisystemic disorder affecting mid aged females?.Serum angiotensen


converting enzyme elevated. Features including the following;
Lungs...hilar lymphopathy
Skin....erythma nodosum
Eyes...uveitis
Oral...orofacial granulomatosis,gingival hyperplasia,painless red nodules

28-percentage of submucous fibrosis turning into malignant?


1%

29-what is gardener syndrome?


It is an autosomal dominant trait charactirized by the ollowing;
Hard tissue tumor inclused,bony exotoses,compound odontome and supernumery teeth
Soft tissue involve ..sebceous cyst,submucousal fibroma and polyposis of the small intestine
which invariably changes to malignancy.

30-features of intra-oral luekemia?


Bleeding, petechial haemorrhagic,mucosal ulceration,mucosal pollar and swelling of
gingival.

31-features of epithilal dysplasia?

Loss of polarity
Hyperplasia of basal cell layer
Dropped of rete pegs
Premature keratinisation
Pleomorphic
Nuclear hyperchromatism
Increased nuclear-cytoplasmic ratio
Disoreded maturation
abnormal or increased mitoses
reduction in the intercellular adhesion

32- patient with a lesion for four years first action to be done is?
Referral for biopsy

33-type of lichen planus which convert to malignant?


Erosive type

34-koilonichya is associated ??
Iron deficiency anemia ,angular chelitis in Brow Kelly-paterson syndrome (plummer Vinson
syndrome) associated with postcricoid web

35-what is the survival rate of cancer in stage one?


>85 % and in stage 4 it is 10% over 5years.

53
54

36-which o the following salivary gland tumour is most likely to occur bilaterally?
a-pleomorphic adenoma
b-mucoepidermoid carcinoma
c-adenoid cystic carcinoma
d-acinic cell carcinoma

answer is d..... is the second most epithlial malignant tumour affecting the salivary gland
after mucoepidermoid cancinoma and occurs often bilaterally.....andeolymphoma occurs
bilaterally only in 10%.

37-most common type of benign odontogenic neoplasm tumour intra-orally?


Ameloblastoma

38-what most common type of benign salivary gland tumour?


Pleomorphic adenoma

39-what organism is associated with hairy leukoplakia?


EPESTEEN BARR VIRUS

40-what syndrome is associated without connective tissue involvement?


SICCA (primary sjorgen syndrome)

54
55

41-a woman with a hyper plastic gingiva red to purple. On biopsy giant cell were found and
increase ANCA?
1-SLE
2-kawasaki
3-wengers granulomatosis

Answer is 3...
SLE has no gaint cells
Kawasaki.. is a disease affecting Japanese and far east children under the age of 5 associated
with strawberry tongue.
Wingers granulomatosis;is agranulomatous disease of nasal tract,profileration of the
gingival tissue, associated with mucosal ulceration, histology show presence of giant cell and
antineutrophil cytoplasmic antibodies (ANCA)

42-what is true about pyogenic granuloma?


a-classically contains giant cells
b-always forms on gingival mucosa

43-35yr old patient with removable white patches on the buccal mucosa and non removable
white patches on the tongue..
a-lekoplakia,
b-osmf,
c-HIV

answer is (i think )HIV as they develop candidial leukoplakis on the buccal mucosa and hairy
luekoplakia on the later aspect of the tongue, luekoplakia in not removable and oral
submucosal fibrosis can not be removed too as the white is due to the fibrosis of the
mucosa

44- EMQ about salivary gland tumours: Old man with bilateral swelling of parotid gland?

ANSWER IS sarcodiosis...phot is on page388 cawson’s

45--kaposis sarcoma occur in oral cavity –


a-tongue
b-palate
c-cheek
4-buccal gingiva

answer...b...is found in the junction between soft and hard palate

46- SCC MOST COMMONEST SITE IN ORAL CAVITY


A -VENTRAL ASPECT OF TONGUE
B-LIP
C-DORSUM OF TONGUE ...

55
56

ANSWER IS VENTRAL OF THE TONGUE REFERENCE IS PAGE 281 CAWSON’S

47-minor, major and herpetic form ulcer site presentation in oral cavity

Minor:labial andbuccal mucosa and tongue

Major;labial and buccal mucosa ,tongue, palate and pharynx

Herpetic form:labial and buccal mucosa,soft palate and floor of the mouth.

48-man with sudden growing tumor on left side of his face , what test would you perform to
confirm

biopsy
saliva
facial nerve

56
57

57
58

49- man with slow growing mass on his cheek with facial palsy? Is it adenomatoid
odontogenic tumour??

I think its mucoepidermoid carcinoma cos it grows slowly and is associated with facial nerve
paralysis but check answer

50-MOST POSSIBLE MALIGNANT TRANSFORMATION


A- MINOR SALIVARY GLANDS
B-PLEOMORPHIC ADENOMA
ANSWER IS A,,,,PAGE 466 CHURCHILL

51- clinical scenario about shingles

52-uncontrolled diabetes patient with bilateral swelling of the parotid gland.

Sialosis

53-ulcers are not common in which type of viral infection?

a-coxsackie virus
b-epestien barr
c-rubella
d-herpes

answer is c....all a,b and d cause oral ulcer apart from rubella which causes German measles
and oral manifestation is petechial red spots on the mucosa reference is pink book page
398,399 and scully page 485and 491

54- Pt with herpetic lesion on the lips what is present

Answer ..cold sore or labialis

55- Treatment for mucocele?

Excised with the underlying gland to prevent recurrence

56- Histology of pemphigus/pemphigoid?


T- lymphocyte CD4 and raised Cd8......check answer

57- Which lesion is more malignant?


a- Lichen planus
b- pemiphigiod
c- Submucosal fibrosis

58
59

Answer is c.... the other more malignant lesions are erosive lichen planus,erythroplakia is
the most common, and speckeld leukoplakia cos it is a mixture of erythroplakia and
leukoplakia...check the answer

58-what is the Chron’s disease main feature?

Cobblestone

59-what is the main feature of ulcerative closistis?

Ulcers indistinguishable from apthae are seen. Artritis,erythema nososuma nd uveitis also
occurs.

60-microorganism causes syphilis?

Answer treponemia palladenum

61-most common site infected with sequamous cell carcinoma due to exposure to sun?

Lower lip

62-what are the coeliac disease main oral features?

Angular chelitis,glossitis, and oral dysthesia,5% may be presented with RAU due to anemia

Reerence is page 167 scully

63-what are the diseases caused by EBV?

Grandular fever/infectious mononycleosis/kissing disease,


burkitts lymphoma
hairy leukoplakia

64-what are the clinical features of primary herptic stomatitis?

Widespread stomatitis, unstable mucosa and vesicle rapture to form painful ulcers, halitosis,
coated tongue and general malaise. Enlarged tender cervical lymph node

65-what is the drug used for the treatment of trigeminal neurolagia?


Carbamazepine/gabapentin/

66- What are the common features of mucous membrane pemiophigoid?


Thick wall bullae filled with blood sometimes, filled with blood and other fluid found on the
posterior part of the palate. These rapture leaving a painful ulcer which heals with scaring.
Affects oral mucosa, genital, conjunctiva and less commonly skin involvement .

59
60

67- what is the condition?

Answer is mucosal cobblestone ,,,crohn’s diease ,sarcodiosis or oral granulomatosis

68-what is the condition this child is affected with and type of organism?

60
61

Bilateral swelling of the parotid gland lesion, child could have mumps which is caused by
paramyxovirus ...treatment involve bed rest, fluid , analgesia and antipyretic.

69-survival rate in a person with tumour and TNM is T1 N0M0?


Survival rate is 85%

70-an elderly lady with history of ulcer in the mouth and scaring in the eye?
a-pemiphgus vlgaris
b-bullae pemiphigoid
c-mucous membranous pemiphigoid

answer is c...pemiphgus valgaris doesn’t heal by scaring ,bulla pemiphigoid has no oral lesion MMP
on the other hand has oral lesion involvement ,skin lesion absent and affect the conjunctiva and heals
with scaring.

71-picture of a child with gingivitis and history o malaise and fever?


Primary Herpatic gingivostomatitis

72-a patient with molar rash extending across her nasal bridge what would she have?
SLE

73- Most common site of intra-oral malignant melanoma ?


Palate and the maxillary alveolar ridge rarely found on the tongue

74-most important thing to rule out in parotid gland tumor?


Facial nerve involelment

75-what is the diagnostic test for diagnosis of infectious monoculosis?


Monospot test and Paul Bunnell..(IgM+VE AND IgG-VE)

61
62

76-Common features of shingle?


It is caused by herpes zoster virus that reactivation incubation period 30 days
It begins in the mouth as vesicles that appear unilaterally following the path of the trigeminal nerve
especially the mandiblar branch.
Later vesicles on the skin appear which turn into scabs later on
Diagnosis is by POLYMERASE CHAIN REACTION and IMMUNOGLOBULIN IgG +ve
Treatment by steroid and high dose aciclovir 800mg*5 for 7 days

77-LICHEN PLANUS ?
Is a bilateral white lesion
Affects females more than males3:2
Common site is the buccal mucosa
Acanthosis of the epithelial layer
t-lymphocte proliferation and liquidation of the basal cell layer this leads to hyperkeratosis
associated with chronic cells ...T-lymphocytes
types include reticular,atropic,papular,erosive ,desquamated gingivitis,plaque and bolus
rarely transforms in malignancy and if so mainly the erosive or atropic type

78-common features of pleomorphic adenoma?


Benign tumour affecting 75% the parotid gland
Common site intra-orally junction between soft and hard palate
Slow growing mass,rubbery and firm
Contain a under developed capsule
Treatment by surgical removal...complections may influde frey syndrome
Recurrent because of difficulties in surgery due to the presence of facial nerve and underdeveloped
capsule that causes escapes of some parts of the tumour.

79- Sarcoidosis?

80- what is the percentage o premalignant lichen planus and leukoplakia that transforms
into malignancy?

Lichen planus...0%

Leukoplakia....5% in pink book it is 10% now check it

Candidial leukoplakia in the floor of the mouth more than 10-40%

Erythoplakia over 40%

81-What kind of lichen planus is more likely to turn malignant?

62
63

a. Bullous

b. Erosive

c. Papillary

anser is b

82- Picture of the buccal mucosa with a white line at the level of the teeth- what causes it

a. Lichen planus

b. Lichenoid reaction

c. Traumatic keratosis

i think the answer is c

83-histological differences between pemiphigiod and pemphigus?

Pemiphigius is intraepthilial ,intercellular IgG and C3 ,heals without scaring ,skin lesion
involved

Pemiphigoid is supraepithilial, linear IgG and C3 at the basement membrane zone,heals with
scaring ,skin lesion is uncommon

84-drug used or lichen planus treatment

If asymptomatic no treatment is required

If symptoms are present then a wide range of treatment none is universal starting from:

1-anti-septic mouth wash such as chlorohexidine gluconate and denzydamine hydrochlorid.

2-corticosteriods topical such as( triamcinolone,betamethasone,belcometason)

Intralesion include triamcinolone

Systemic prednisolone and azaithoprine

85-features of epithelial dysplasia include

a-hypo chromatosis

b-atypical mitosis

answer is b

63
64

86- Questions on mucocele, where is it usually found

a. Upper lip Retention

b. Lower lip Extravasation

c. Floor of the mouth

d. Palate

answer is ;lower lip retention cyst,lower lip extravasation ,floor of the mouth ranula(is found
on the buccal mucosa )

87-how would you treat the mucocel?

Excision with underlying duct or mucocel

For the ranula by marsupialisation because incision will lead to recurrence and enoculation
is difficult because of thin walls

88- A PCS OF 30 YEAR MAN WITH A LIP LESION.HE HAD COME BACK FROM A HOLDAY AND
HIS PARTNER HAD THE SAME LESION EARLIE ON.OPT WERE ;

a-HERPES SIMPLEX

b-,TUBERCULOSIS,

c-ERYTHEMA MULTIFORM

Answer is A, one of the causes of labialis is sunlight

89- Patient presents with ulcer which he’s had for 4 weeks. What will you do?

a. Refer to specialist

b. Perform incisional biopsy

answer is referral to a specialist

90- Picture of the tongue with an ulcer on the lateral border- what drug causes it

a. Ace inhibitors

b. Calcium channel blockers

64
65

c. Potassium sparing drugs

answer is C or a,,nicrondial is a putassum blocker which causes ulceration of the oral


mucosa. Calicium blocker such as nifidipine causes gingival hyperplasia and angiotensine
converter enzyme inhibitor causes dry mouth,taste ulteration and lechinoid reaction

91-A PATIENT PRESENTED WITH ASYMPTOMATIC , SMOOTH , CIRCUMSCRIBED RED AREA INMIDLINE
ANTERIOR TO CIRCUMVALLATE PAPILLAE ON THE DORSUM OF TONGUE WITHMICROSCOPIC EVIDENCE OF
EPITHELIAL HYPERPLASIA . THE MOST PROBABLE DIAGNOSISOF THE CONDITION IS :
A .GEOGRAPHIC TONGUE
B.HAIRY TONGUE
C.MEDIAN RHOMBOID GLOSSITIS
D .LINGUAL THYROID
ANSWER IS C

92-in the hairy tongue ,there is hypertrophy of?


a-fungi papillae
b-filiform papillae
c-foliate papillae
d-circumvallete papillae
answer is b

93-M OST COMMON TYPE OF CLEFT IN MALES IS :


A . UNILATERAL CLEFT LIP ALONE
B . UNILATERAL CLEFT PALATE ALONE
C . UNILATERAL LIP AND PALATE
D .B ILATERAL CLEFT LIP
A NSWER IS A

94-A PATIENT PRESENTS WITH SMALL YELLOW SPOTS , PRESENT BILATERALLY ON BUCCALMUCOSA
OPPOSITE TO POSTERIOR TEETH WITHOUT ANY OTHER ASSOCIATED COMPLAINT .M OST PROBABLE
DIAGNOSIS OF THE CONDITION IS :
A .K OPLIK ’ S SPOT
B .F ORDYCE ’ S GRANULES
C .M ELANOTIC MACULED .
D .W HITE SPONGE NEVUS
A NSWER IS B

95-B LUE SCLERA IS SEEN IN –


A ) M ARFAN ’ S SYNDROME
B ) O STEOGENESIS IMPERFECTA

65
66

C) O STEOPETROSIS
D ) A LL OF THESE
A NSWER IS B

96-In Behcets disease which HLA subtype supports diagnosis


a. HLA B 51
b.HLA 51
c.HLA A 51
d. HLA T 51
answer is A it is of no clinical value in diagnosis but HLA B51 is a predictor of occular lession
which can lead to blindness

97-N AME THE CONDITION :


A N AUTOSOMAL DOMINANT CONDITION OF NAIL DYSTROPHY , LEUKOPLAKIA AND PIGMENTED SKIN
SEEN COMMONLY IN CHILDHOOD .
D YSTERAKOSIS CONGENITA

98-The most commonest tumors found in salivary glands of children?

Lymphangeomais and heamangomaous

99-A child presents with sore throat, fever, headaches, generalized lymphadenopathy,
malaise and a macula- papular rash. Oral manifestations of oral ulceration and petechial
harmorraghes. What is the possible diagnosis?

Grandular fever

100-which immune cells mediate the autoimmunse reaction in Lichen planes

ANSWER id t-lymphocyte CD4 and CD8

101-T O WHICH VIRUS FAMILY DOES VARICELLA ZOSTER BELONG ?


A) COXSACKIE
B) HERPES
C) HEPADNAVIRUS
D) PAROVIRUS

A NSWER IS B

102-MOST COMMON TUMOR OF SALIVARY GLAND IS


A) PAPILLARY CYSTADENOMA LYMPHOMATOSUM
B) ADENOID CYSTIC CARCINOMA
C) PLEOMORPHIC ADENOMA
D)OXYPHILLIC CELL ADENOMA

ANSWER IS PLEOMORPHIC ADENOMA

103-which of the following medication is the most likely cause of dry mouth?

66
67

a-aspirin

b-atenolol

c-bendroflumethazide

d-captopril

ANSWER IS C

104- TARGET LESIONS ARE ASSOCIATED WITH WHICH ONE OF THE FOLL CONDITIONS ?
A . STEVENS JOHNSON SYNDROME
B. PHEMPHIGUS
C.PHEMPHIGOID
D .SYPHILIS

Answer is a,associated with eryrhemia multiform which has target cell spread on the palm
,leg,neck and face .steven Johnson syndrome is a sever condition

105-1. WHICH ONE OF THE FOLLOWING CONDITIONS IS LEAST LIKELY TO BE ASSOCIATED WITH AN
INCREASED RISK OF ORAL CANCER
1. ACTINIC CHEILOSIS
2. E RYTHROPLASIA
3.LEUKODEMA
4.LEUKOPLAKIA
5.PLUMMER VINSON SYNDROM

ANSWER IS 3

106-A 55 Y OLD FEMALE PRESENTS WITH A NON-HEALING ULCER ON THE LATERAL BORDER OF TONGUE .
THE ULCER HAS NOT RESPONDED TO LOCAL MEASURES . YOU SUSPECT A SQUAMOUS CELL CARCINOMA .
WHICH TYPE OF INVESTIGATION WOULD YOU PERFORM FOR HER?
A . EXCISIONAL BIOPSY
B. EXFOLIATIVE CITOLOGY
C. IMMUNOFLUORESCENCE
D . INCISIONAL BIOPSY
E .NEEDLE ASPIRATION
ANSWER IS D

107-A 45 Y OLD, FEMALE PRESENTS WITH A 6 MONTHS HISTORY OF RECURRENT MEAL TIME SWELLING
AND PAIN IN HER RIGHT SUBMANDIBULAR REGION . C LINICAL EXAMINATION DOS NOT REVEAL ANY
DENTAL ABNORMALITY . WHAT IS THE MOST LIKELY CAUSE OF HER SYMPTOMS?
A . LYMPHADENITIS
B. SIALOMETAPLASIA – BENIGN ULCERATIVE LESION. DUE TO ISCHEMIC NECROSIS OF THE SUBLINGUAL
GLANDS . PALATE FALLS OFF.
C. SIALOLITIASIS - OBSTRUCTION IN THE SALIVARY GLAND .
D . S IALORRHEA - EXCESSIVE SALIVATION .
E. XEROSTOMIA

67
68

ANSWER IS C

108-A linear white patch seen in the buccal mucosa of geriatric patients?

109-

Answer is c

110-ACCORDING TO NICE GUIDELINES, WHICH OF THESE PATIENTS WOULD YOU REFER URGENTLY FOR
POSSIBLE ORAL CANCER?
A - DIFFICULTY SWALLOWING FOR A WEEK
B- A SMOOTH WHITE PATCH THAT PERSISTS FOR MORE THAN 3 WEEKS .
C- UNEXPLAINED TOOTH MOBILITY
D - RECURRENT ULCERS
ANSWER IS C

111-WHAT FEATURE OF A WHITE PATCH WOULD MAKE YOU LEAST SUSPICIOUS OF ORAL CANCER?
A - APPEARANCE CHANGED RECENTLY
B- ULCERATED
C- NODULAR
D - SOFT WHEN PALPATED
ANSWER IS D

68
69

112-IN THE UK, ORAL CANCER MOST COMMONLY OCCURS IN THE


TONGUE
LIPS
PALATE
CHEEK
ANSWER IS THE TONGUE

113-

Answer is osteopenia bone loss that is not sevre as in osteoporosis.

114-WHICH TECHNIQUE IS NOT USED FOR ORAL CANCER SCREENING?


CHEMILUMINESCENCE
AUTOFLUORESCENCE
ULTRASOUND
TOLUIDINE BLUE USED TO IDENTIFY IN CARCINOMA IN SITU .
ANSWER IS ULTRASOUND

115-WHICH OF THESE VIRUSES IS COMMONLY ASSOCIATED WITH ORAL CANCER?


COXSACKIE B VIRUS
HERPES VIRUS
EPSTEIN-BARR VIRUS
HUMAN PAPILOMA VIRUS
ANSWER IS HUMAN PAPILLOMA VIRUS

115-which of the following is the most likely to turn malignant?

69
70

a-intradermal nevus

b-junctional navous

c-lichen planus

d-papillpma lechin planus

116--what is newton's classification and regarding which disease?


Newtons classification classifies the denture stomatitis into its severity.
1- pinpoint hyperaemia 2- diffuse erythema 3-granular papillary

117-raynaud’s phenomenia usually occurs at?

a-systemic sclerosis
b-lichen planus
c-leukemia
d-sjogren's syndrome
answer is a

118-ground glass appearance is seen in?

a-RAS
b-hyperparathyroidism
c-darier;s syndrome
d-gardner's syndrome
answer is b

119-during oral examination of a 57 years old man with a white keratotic patch that covers
the entire palate is noted .some red spot seen in this patch what is the most probable
explanation is that the patient is a ?
a-pipe smoker
b-snuff smoker
c-cigar
d-tobacco chewer
answer is a

120-SIALADENOSIS COMMONLY INVOLVES


A] P AROTID GLAND
B] ACCESSORY SALIVARY GLAND
C] SUBLINGUAL GLAND
D ]SUBMANDIBULAR GLAND
ANSWER IS A

121- ADENOID CYSTIC CARCINOMA IS KNOWN TO COMMONLY INVOLVE


A ] PAROTID GLAND
B] SUBMANDIBULAR SALIVARY GLAND

70
71

C] MINOR SALIVARY GLANDS


D ] SUBLINGUAL SALIVARY GLAND
ANSWER IS C

122-what condition is associated with an increased risk of malignant change?


a-hairy leukoplakia
b-denture stomatitis
c-erossive lichen planus
d-sublingual keratosis
e-geographic tongue
answer I think is c because lechin planus converts to malignant in 1% whereas sublingual
keratosis is 10% the rest are benign

123-
What is the condition?
Ectodermal dyplasia

124-
Identify the condition?
Congenital syphilis

125-in which salivary gland calculi is more common?

Parotid
Submandibular
Sublingual

answer is submandibular

71
72

125- AMONG THE FOLLOWING MUSCLE WHICH IS LEAST AFFECTED IN MYOFACIAL PAIN DYSFUNCTION
SYNDROME ?
A -LATERAL PTERYGOID
B-MASSETER
C-TEMPORALIS
D -MEDIAL PTERYGOID

Answer I think is a

126-which o the following features are not related to myofacial dysfunction syndrome?
a-eitology is related to psycho-physiology
b-females are more commonly affected than males
c-muscle tenderness more common around tyhe angle of the mandibule
d-treatment of this syndrome is by conservative measures
answer is c

127- A PATIENT COMPLAINS OF NUMBNESS OF LOWER LIP WITHOUT ANY PREVIOUS HISTORY OF TOOTH
EXTRACTION OR TRAUMA . THE CONDITION MAY BE RELATED TO
A -METASTATIC NEOPLASM
B- INFECTION
C-DISEASE OF CNS
D -NEUROFIBROMA
...

128-most common type of malignancy of the bone?


a-osteosarcoma
b-ewing sarcoma
c-metastatic carcinoma
d-multiple myloma

129- METASTATIC DISEASE TO ORAL CAVITY MOST FREQUENTLY OCCURS IN


A - POSTERIOR MAXILLA
B-TONGUE
C-POSTERIOR MANDIBLE
D -FLOOR OF THE MOUTH

130- - ACCESSORY SALIVARY GLANDS OF WHICH OF THE FOLLOWING INTRAORAL SITES ARE MOST
FREQUENTLY AFFECTED BY SALIVARY GLAND TUMORS ?
A -SOFT PALATE
B-HARD PALATE
C-CHEEK
D -UPPER LIP

131- -W HICH OF THE FOLLOWING IS THE REACTIVE LESION OF GINGIVA THAT MAY SHOW BONE
FORMATION MICROSCOPICALLY AND RADIOGRAPHICALLY ?
A - IRRITATION FIBROMA

72
73

B- OSTEOMA
C-PERIPHERAL OSSIFYING FIBROMA
D -PYOGENIC GRANULOMA

132- -THE MAIN HISTOLOGIC DIFFERENCE BETWEEN CARCINOMA AND CARCINOMA IN SITU IS THAT OF
A -HYPERCHROMATISM
B-NUCLEAR ATYPIA
C-INVASION
D -ALTERED NUCLEAR CYTOPLASMIC RATIO
ANSWER IS C

133squamous cell carcinoma of the tongue metastasis to the?


a-brain
b-cervical lymph node
c-liver
d-bone
answer is b

134-what is the treatment for pemphigoid when there is no response to corticosteroid??


Dapsone

135-what is xeroderma pigmentosa??


is a genetic disorder of repair of DNA / DNA, in which the body's normal ability to remove
the damage caused by ultraviolet (UV) radiation is poor. This can lead to multiple basal cell
carcinomas/scc. Children are known as moon babies.

136-Cytomegalovirus is
A) HHV 2
B) HHV 3
C) HHV 4
D) HHV5
Answer is d

General Medicine
1-a lady with 6 month history of mitral valve stenosis and high arched palate diagnosis is:
a-down syndrome
b-marfans syndrome
c-cleideocranial dystosis

answer is B...reference is that Marfan’s syndrome is a hereditary disorder of connective


tissue characterized by elongated extremities , subluxation of the lens, dilatation of the
ascending aorta and pigeon breast ,ABRIHAM LINCOIN had this syndrome.

73
74

2-a man has diet contolled type diabetes,he consumes 30 glasses of alcohol and 40
cigarettes per day.what is the reson of dry mouth?
a-Sjorgen
b-Dehydration
c-candidasis
d-drug induces

answer is B...reference is that in the question there is no reference of syjorgen


syndrome,candidasis is a result of dry mouth,patient in diabetic on diet control so no
medication is given so D is not the answer ,too much alcohol causes dehydration so answer
is b.

3-picture of an adult patient with large hand and face prominent supraorbital ridges and
prognathsim?

Answer is acromygaly due to over production of growth hormone by the pitutarty gland.

4-what are the characteristic features of cushing disease?


Answer,,,,,moon face,truncal obesity,weight gain,personality change,hypertension ,thin skin
and cautenous stria.

5- a patient with RH fever will you prescribe antibiotics for prophylaxis?


No

6-hypoglycaemia is ;
a-atype of consciousness
b-low insulin level
c-low blood pressure
d-low blood sugar

answer is d

7-what are the glucose level during fasting and random blood glucose test?
Fasting is less than 6mmol/L in diabetic patient it is over 7
Random is less than 8mmol/L in diabetic patient it is equal or more than11.1mmol/L

8-what is treatment of angaina?


Aspirin which prevent the platelet activity by preventing the synthesis of thromboxane A2
aspirin is an acetylate platelet cyclo –oxigenase.
Coplidgel which a glycoprotein 2b and 3a receptors which prevent the adhesion of
fibrinogen with the platelets
Lowering lipid drugs
Ca channel blocker........nifidipine
Potassium channel blocker....nicorandil
Glycerol trinetrate (GTN) decreases the peripheral resistance and decreases the oxygen
demand from the heart.

74
75

Post angioplasty and coronary artery bypass graft

Reference is scully page 109

10-What type of test would you ask a patient with suspected diabetes to do?
Fasting test,random test and oralglucose testing tolerance

11-question about RH fever?


Fever ,arthraligia margratory, erythema margiratory,subcautenous nodules, cerebral
involvement leading to involuntary spasmodic movement such as Sydenham chorea or ST
vitus dance,lung involvement,cardiac complication
Treatment by the use of pencillin and prophylaxis is by phenoxymethyl pencillin (pencillin v
orally) or sulphidoamide if allergic to pencillin
It is a type II hypersensitivity

12- what are the causes of cushing disease?


Is the increase in glucocorticoid due to hyperplasia of the adrenal gland secondary to an
increase in the secretion of ATCH by the pitutarty gland from the basophil adenomas.
Patient should be given corticosteroid cover preoperatively 25-50 mg IV of hydrocortisone

13- crushing pain in the chest radiating to the arm and relieved by sublingual GTN?
ANGINA PECTORIS

14-crushing pain in the chest radiating to the arm accompanied by vomiting?


Myocardial infraction

15-treatment of basilar artery anuerysim?


Restore the deteriorating respiration and decrease the intracranial pressure by surgical
clipping or endovascular coiling.,,,,refrence wikipidia

16-a women with hiatus hernia and loss tooth surface what is the treatment option?
a-medicine
b-dietary advice
c-occlusal splint

splint to prevent further erosion of the teeth

17-which condition would elective dental treatment be postponded for 6 months


a-myocardial infraction
b-infective endocarditis
c-cardiac failure
d-angaina

answer is a reference s, angina elective treatment deferred for 3 months, infective


endocarditis treatment should take place to prevent further infection ,cardiac failure
treatment takes place when the condition is controlled and finally myocardial infarction is
the condition where treatment should be deferred for 6 months.

75
76

18-what is the age of the onset of type one diabetes?


Between the age of 12-30 but can occur at any other age.

19-what is used for the treatment of type 1 diabetes?


a-insulin
b-diet control
c-hypogycaemia drugs
d-drugs and diet.

I think the answer is a (not sure confussed with whether insulin is considered drugs)

20-what substance is increased in diabetic coma?


There are two types of coma the hypoglycema coma there will be an increase in insulin and
in the hyperglycemia there is an incease glucose Ketones(acetoactate, beta
hydroxyburylate,acetone)(only type 1)

21-investigation or coeliac disease,crohn’s disease.IBS


Coeliac disease.....FBC,HAEMATINIC,STOOL EXAMINATION,PRESENCE OF EXCESS FAT IN THE
STOOL,LACTOSE INTOLERANCE and D XYLOSE TEST,ENDOSCOPIC BIOPSY FOR JEJUNAL
MUCOSA.

Crohn’s disease:.....blood count, serum potassium, zinc and albumin.Erythrocyte


sedemiation rate ESR and C reaction protein CRP,plain film and contrast radio graph,
ultrasound,CT and endoscopy with biopsy of mucosa

22-patient with weight gain and intolerance to cold?


Hypothyroidism

23-a patient with angina in the past six months what will you do?
a-continue treatment
b-refer
c-not treat
answer is a

24-type1 diabetic patient weight of patient and age?


Patient is usually underweight and age is under 25

25-type 2 diabetic patient weight and age?


patient is usually obese and age is over 45 howerver, matutary onset of diabetes in young
MODY patients are usually under 25 and slim

26-what would you give a hyperglycaemic patient on collapse or dizzeness?


Oxygen ……

76
77

( If sure it is hyperglycaemia then give an intravenous infusion to treat dehydration and give
patient IV insulin 20 unit stat)emergency drug kit contain no insulin

27-finger clubbing is seen in all except?


a-folate tetratology
b-lung cancer
c-chrons syndrome
d-angina

Answer is angina

28- What’s the most frequent site of endocarditic?


a-tricuspid
b-mitral
c-aortic
d-bicuspid

answer is B ,,,tricuspid is more infected with patient on Iv drugs however the mitral valve is
more prevalent in drug and none drug users ...reference is Wikipedia.

29- Infective endocarditis causative micro organism in the uk?


Streptococcus sangus
Streptococcus mutanus

30-who is at most risk of hypoglycaemia during dental treatment?


a-Adult with type 2 dieabetes on diet control
b-insulin dependent who had their meal and medication
c-young patient type one diabetes who missed their mealtime

answer is c

30- what is the common site of crohn’s disease?


Ilium and caecum

31-what are the clinical features of Paterson-brown-Kelly syndrome?


Post cricoid dysphagia,upper oesophageal web and iron deficiency anemia

32-picture of herpatiform ulcers and the ventral of the tongue what kind of GIT infection?
Celiac disease

33-oral manifestation of anginotensin converting enzyme inhibitor?


Such as captopril,Ramipril, Lechnoid reaction,angiooedenoma and dry mouth,burning
sensation ulceration and loss o taste

34-what are the high substance found in diabetes coma?


Ketones and glucose in hyper and insulin in hypo

77
78

35- M OST COMMON CONGENITAL HEART DISEASE ?


A -A ORTIC STENOSIS
B -PDA
C -M ITRAL VALVE PROLAPSE
D -VSD
Answer is c scully page 101

36- what are the immune cells in multiple myloma?

monocyte
lymphocyte

according to scully i think it is plasma cell page 227

37-adrenal suppression is less when steroid is given ?


1-in divided doses on the same day
2-alternate days

Answer is 2 page 150 scully

38 a man has diet contolled type diabetes,he consumes 30 glasses of alcohol and 40
cigarettes per day.what is the reson of dry mouth?
a-Sjorgen
b-Dehydration
c-candidasis
d-drug induces –

answer is b

39- what are the oral manifestations with these antihytpertensive drugs ?
Beta blockers
Calcium channel blocker
ACE inhibitors

Beta blocker...lichenoid reaction, dry mouth , parasthesia


Calcium channel......gingival hyperplasia
ACE inhibitor......dry mouth,burning sensation ,taste ulteration and lichenoid reation

39--what test u do for patient with hypochromic microcyte anemia with normal folate level?
thalassemia

39-A man comes to surgery after 2weeks of extraction, has an artificial mitral valve, has
fever & feeling frail, on examination- increased heart rate. What is the condition? Answer is
infective endocarditis

78
79

40-what are the hematological tests required or anemia?


Blood test FBC, blood film and shilling test for vitamin B12 deficiency,MCV

41-which is the safer mode of giving LA to haemophilia


Block
Intramuscular
Infiltration
Intrligamentary

Answer is intraligamentary

42-reed Stenberg cells are seen in

Hodgkins
Non-hodgkins
Burkitts lymphoma
Answer is hodgkins

43-A African male with microcytic anaemia, normal ferritin levels, what other investigation
do v have do ?

HbF and HbA2 and serum total iron binding capacity TIBC to rule out thalassemia…check
this

44-INR

Normal? 1.0
Suitable for dentoalveolar procedure? Less than 4

45-most common type of acute leukemia in adults?


Acute non-lymphoblastic leukemia...acute myeloblastic leukemia...affects adults

46-most common type of leukemia in adults?


Chronic lymphocytic leukemia......affect over 40 years old

47-most common type of leukemia in children?


Acute lymphoblastic leukemia children between 2-4 years old

Note :
 Acute lymphoblastic leukemia-peak incidence 2-4yrs but can affect any age
grp.most common childhood leukemia.tx :cytotoxic drugs,bm transplantation if
chemo fails
 Adult acute lymphoblastic leukemia - worse pognonsis than chilhood rest all the
same as above
 Acute non lymphoblastic (myeloblastic) leukemia - most common acute leukemia
of adults.esp causes gingival enlargement

79
80

 Chronic lymphocytic leukemia - most common type of leukemia. men particularly


affected. asymptomatic patients may not need Rx.symptomatic- with radio and
chemo therapy. prognosis better than acute.
 Chronic myeloid leukmia - proliferation of myeloid cells in bone
marrow,peripheral blood and tissues.have philepedia chromosome. >40 yrs age
grp affected.lymphadenopathy is rare.

47-Microcytic anaemia, ferritin levels normal what exam to check probable cause? Name of
exam requested to check thalassemia???
microcytic anaemia due to iron def, if TIBC increases it is thalassmeia, if that decreases liver
disease.
MCV,TIBC,UIBC,HB and blood smear
Thalessmia histological features include anisolocytosis poikilocytosis target cells,basophilic
strippling

48- Which test will you perform in an afro-caribbean if the tests for anemia are normal?
Sickle cell test

49-what condition is associated with raynaud phenomenia?


Cryoglobulinemia.. and scleroderma

50-what is mikulicz syndrome ?


It is a autoinmmiune condition usually associated with SS-1,involving both lacrimal and
salivary gland enlargement it is usually symmetrical with mononuclear cell infilteration can
be also seen in sarcadiosis and lymphoporliferation.

51- anaemia is seen in all except?


Hypothyroid
Cirrhosis
Cyanotic heart disease
Peptic ulcer

Answer is cyanotic heart disease is associated with polycythemia

52-which of these is under the influence of pituitary gland?


Diabetes type 1
Diabetes type 2
Diabetes insipidus

Diabetes inspidus due to the increased production of ADH

53-bleeding is delayed in?


Coagulation defects
Platelet defect

Answer is coagulation defect ,platelet defect in associated with immediate bleeding scully
page 181

80
81

54-which antibiotic is safer in patient taking warfarin?

Amoxicillin
Ampicillin
Tetracycline
Penicillin v
Answer is pencillin v

55-half life of factor 8 replacement ?

12h
18h
16h
24h
Answer is 12h ,heparin half life time is 4-6 hours

56-What percentage of factor 8 replacement should b given for extraction & maxillofacial
surgery ?

75-100% for maxillofacial surgery


50-75% for surgical extraction

57-heparin inhitis which haemostasis?


Primary
Secondary
Tertiary
Answer is secondary

58-haemathroses is a characteristic feature of what condition?


1-xmas disease
2-haemophila A
3-VWD
4-heamophilia c

Answer is haemophilia A

59-the most common inherted bleeding disorder is?

Haemophila A
Haemophila B
VWD
IDC

Answer is vWD.

60-NORMAL RANGE OF BLEEDING TIME


1-6 minutes

81
82

61-in VWD factor 8


increases
decreases
normal
Answer it decreases

Bruxism is a common form of parafunctional activity of which the patient may or may not be
aware. It may be important in the development of a treatment plan to determine whether
the patient is an active bruxist.
The principle clinical sign of active bruxism is:
A. Head and / or neck pain
B. Excessive tooth wear
C. Temporomandibular joint clicking
D. Sensitive teeth
E. Cheek ridging and tongue scalloping
Answer is e

If a patient develops osteosarcoma at an old age.what bone disease should u suspect? Paget
disease

Pepper pot skull is seen in which condition? Multiple myeloma,,, pepper pot skull is a skull
with punched out lesions in the bone multpe radiolucencies

62-alcohol intake in diabetic patient cause?

Hypoglycaemia
Hyperglycaemia
Answer is hypoglycaemia

63-which of these measures is efficeincy to record the intrinsic pathway?

APTT
PT
INR
Answer is APTT

64-Which of the following is not involved in common pathway of d coagulation system?


Factor10
Factor4
Factor9
Factor 2

Answer is factor 9

82
83

65-In which of these bleeding time is prolonged


HaemophiliaA
Von willibrands

Answer is VWD.

66-Which test is most important for detecting haemophilia


PT
BT
APTT
INR

Answer is APTT it will be prolonged

67-what do these serum sample result indicated?


a-patient with HbeAg 1-nurse can’t work in the surgery
b-patient with HbeAB 2-serum in the liver patient is a carrier
c-patient with HbsAb 3-high carrier
d-patient with HbsAg 4-low carrier
e-patient with HbcAg 5-carrier or vaccination

answers,a-3,b-4,c-5,d-1,e-2

68- what is a dupuytren sign?

Answer is is a hand sign involving the 5 and fourth finger associated with alcoholic hepatitis

69-which of the following statement is correct regarding seriological makers of viral


hepatitis are correct?
a-presence of HbsAg alone implies that the patient had previpus infection but is at low risk
withregard to transmitting the disease
b-presence of HbsAg alone implies that petient had previous infection but is at high risk with
regard to transmitting infection
c-HbcAg if present in serum means the patient is the patient is highly infective
d-HbeAg if present in serum means the patient is highly infective
e-the antibiody of HbsAg will only be present if patient is highly infective.

Answers are a,d

70- What kind of Ag should be detected to check if the infected dentist with hepatitis B
should be discontinued from practice?

If HBeAg is +ve
Or HBeAg is -ve but have greater than 1000 Hbv viral particles per ml of blood

71-a patient with no positive history came along for scaling the moment you pick up your
scaler you punch your finger, what should you do

83
84

a-complete procedure as nothing has happened


b-check patients blood for HBsAb
c-check patients blood for HBsAg
d-check dentist blood for HBsAb and HIV antigen
e-check dentist blood for HBsAg
f-dentist should go and take HBsAb

answer is a
72-what are the percentages of inoculation injuries in the dental clinic?
HCV
HBC
HIV
Answer is ;HCV 3%,HBV 30%, HIV 0.3%

73-patient came to you surgery and said some people want to kill him what is the diagnosis?
paranoid

74-patient came to the surgery wants to extract his teeth because someone implanted a
device to spy on them? Schezo

75-patient could not brush his teeth his wife died recently? Depression

76- A 60 year old patient attends your surgery complaining of a sore mouth. He has Type II
diabetes well controlled by diet and metformin. On examination white patches which
cannot be removed are present on his buccal mucosa.

What is the most likely diagnosis?

A. Frictional keratosis

B. Leukoplakia

C. Lichen Planus

D. White sponge naevus

E. Candidosis

ANSWER IS C

77- A 60 year old female attends your surgery complaining of soreness affecting her
gingivae. No other area of her oral mucosa is affected but she has noticed an itchy rash on
the flexor surface of her forearms. She is fit and well and is not taking any medication.
Scattered purple/red papules each about 4mm in greatest dimension are present on the
flexor surface of her forearms and on intraoral examination a desquamative gingivitis is
present.

Based on the above findings what is your diagnosis?

A. Mucous membrane pemphigoid

84
85

B. Lichen planus

C. Pemphigus vulgaris

D. Erythema multiforme

E. Lichenoid drug reaction

Answer is B

78- A 48 year old woman complains of a sore area on the right buccal mucosa adjacent to a
restored tooth. The lesion has a lichenoid appearance and this is confirmed
histopathologically following a biopsy.

Which of the following restorative materials is most frequently associated with lichenoid
changes.?

A. Gold

B. Amalgam

C. Porcelain

D. composite

E. Glass ionomer cement

Answer is gold

79- A 60 year old female attends your surgery complaining of soreness affecting her
gingivae. No other area of her oral mucosa is affected but she complains of occasional
bleeding and crusting inside her nose and itchiness affecting one of her eyes. She is fit and
well and is not taking any medication. On examination intra-orally a desquamative gingivitis
is present. Her left eye looks inflamed and there is some evidence of scarring giving rise to
symblephron formation between the conjunctiva lining her lower eyelid and that covering
the surface of her eye itself.

Based on the above findings what is your diagnosis?

A. Mucous membrane pemphigoid

B. Lichen planus

C. Pemphigus vulgaris

D. Erythema multiforme

E. Lichenoid drug reaction

Anser is A

85
86

80- A 60 year old female attends your surgery complaining of soreness affecting her
gingivae. No other area of her oral mucosa is affected, she has no skin lesions and other
mucosal surfaces do not appear to be affected. She is fit and well and is not taking any
medication. On examination intra-orally a desquamative gingivitis is present. Your
differential diagnosis lies between lichen planus and mucous membrane pemphigoid. You
carry out an incision biopsy and send the fresh tissue to the laboratory without putting it
into formalin.
What procedure, performed on sections of fresh frozen tissue, do you expect the
histopathology department to perform in addition to conventional staining with
haemotoxylin and eosin?
A. Indirect immunofluorescence

B. Direct immunofluorescence

C. Papanicolaou (PAP) staining

D. Periodic acid Schiff (PAS) staining

Answer is B

81-If a patient is unable to look down & medically which nerve is affected
3rd
4th
6th
Answer is 4th

82-Convergent squint is due to which nerve damage


3rd
2nd
4th
6th
Answer is 6th

83-All viruses are associated with Bell's palsy except


CMV
Coxsackie
EBV
Zoster
Anwer is coxsakie

84-what type of cells in found in TB?


Granulomas Langerhans macrophages

85-which microorganism is not killed under sterilization?


Prions

86-what is characterized by swan fingers? Rheumatoid Arthritis

86
87

87-what is the characteristic feature of Behcet disease? Uvietis,aphtous ulcer and


genital ulcers

88-treatment of asthma?
Beta agonist (salbutamol):used for routine control of asthma
Antimascarinic branchodilater(ipratropium):used for asthma with bronchitis
Mast cell stabilizer(nedocromil):used in children
Recobmant humanized monocolonal IgE (omalizumab):allergic asthma
Coticoteriods (belcometason,betametason):used when asthma symptoms are on daily
basis

89-osteoartheritis is characterized by the following:


Hederben nodules,swelling of joints and buochard nodules ,no inflammation signs
Degeneration of the cartilages in weight baring joints and deposition of bone instead

90-clinical characterastics of TB?


Weight loss,fever ,cough and haemoptysis

91 Clinical presentation of
Asthma
Chronic bronchitis
Tumour in the upper lobe of the lungs
Codeine use
Pethidine use
Carbamazepine use
Options are : inspiratory wheeze, expiratory wheeze, Stridor, daily mucus production,
dry airway, dry cough,consiptation
Athma-expiratory wheezing,
Chronic bronchitis-thick mucus production
Codeine-consiptation
Tumor in the upper lobe –stridor
Pethidine-dry cough
Carbamazepine-dry airway

91-pain in intrephalangyeal? RH

92-best medication for acute asthmatic attack?


Beta 2 agonist salbutamol

93-clinical feature of the following dieases?


Hydrochondrosis, clausterphobia, obsessive compulsory disorder,mania,
schizopherinia
Hydrochondrosis:mainly is a disease of olderly patients as they live alone ,excessive
preoccupation with fear of disease or strong believe of having a disease, those
patients usually associated with oral dystheasia syndrome and facial pain with no
organic explanation to it.

87
88

Obsessive compulsory disorders;is the fourth most common mental disorder were
patients repeat checking on things ,patients are intelligent, stubborn affect checking
on dirt,cleanisness many of them work in the health sector
Clausterphobia;unable to tolerate being in confined small spaces
Mania;patient in euphoric mood,no need to sleep,excessive talking and racing thought
Schizophrenia:due to dopamine over activity,hallucination delusion and thought
disorder

94-what diseases are haemogobulinopathy? sickle cell and thalassemia

95-Dysphagia is associated with which one of the following conditions


A)oesophageal-carcinoma
B)hyperthyroidism
C) hypothyroidism
d)artirial fibrilation
answer is a

96-a patient complaines of a severe chest pain while in the dental chair whih of the
following is regarded as an appropriate manegment?
a-put the patient flat
b-give the patient oxygen
c-put the patient in recovery postion
d-adminseter gloucose tablet
answer is b

97-which of the following statment regard down syndrome is incorrect?


a-it iscaused by trisomy 21
b-the incidince increases with the age of the mother
c-the down patient suffers from microglossia
d-down syndrome suffer from cardiac anomilies
answer is c

98-which one of the following statment regarding cardiac fibrillation is correct?


a-it is uncommon in elderly patients
b-it is treated with digoxin
c-it does not cause fainting in elderly patient
d-there is a p wave in the ECG
answer is b

100- which of the following are NOT the orofacial side effects of
Antihypertensive drugs?
A)Xerostomia
B)lichenoid reaction
C)erythema multiforme...
D)andioadenoma

88
89

e)all of the above


f)none of the above
answer is f

101- In a patient with thalassemia peripheral blood film for red blood
morphology shows
a-hypochromic-microcytic
b-normochromic-normocytic-cells
c-macrocytosis
d-sickle cells
e-spherocyte
answer is a

102- -A pt of chronic diarrhoea is having angular stomatitis and glossitis.the


most likely cause of these signs is deficiency of
a-folic-acid
b-iron
c-proteins
d-thiamine
e-pyridoxine

Answer is b

103-the most common facial abnormality in gardener's syndrome ?


Adenomatous polyposis associated with multiple osteomas skin fibromas
,epidermal cyst and pigmented ocular fundus.
Ostemoa is typically multiple and may arrange along the alveolar rigde of the
mandibuler jaw.it is also associted with dental abnormalities such as multiple
impacton other than the wisdom tooth and supernumary teeth

104-bronchiectasis is most common in which lobe ?


Right middle lobe

105-Which is more commonly involved with rheumatic heart disease


A)tricuspid-valve
B)mitral-valve
C)both...
D) neither
Answer is b

106-Of the various serum enzymes, which is the first to be elevated after an
acute myocardial infarction
A)creatine-kinase
B)lactic-dehydrogenase
C)aspartate-aminotransferase
D) troponin t

89
90

D is the answer reference is scully page 110

107-which of the following normally contains > 10% body iron?


A)transferrin
B)heart
C)neutrophils
D) macrophages
Answer is d, transferrin contian less than 0.1 o the body iron .rest is stored in
the liver or phagocytosed by the macrophages after the break down of the
RBC to keep the iron concentraction low in the body fluid.
Reference is wikipidia

108-sever hemophilia is typically associated with what percentage of factor


VIII in the plasma
A)less-than-1%
B)2-5%
C)6-10%
D) 50%
Answer is a scully page 188

109- IN HEMOPHILIA A
A) PROTHROMBIN TIME (PT) IS NORMAL , PLASMA THROMBOPLASTINE TIME (APTT) IS
NORMAL
B) PT IS NORMAL , APTT IS PROLONGED
C) PT IS PROLONGED ,A PTT IS NORMAL
D) PT IS PROLONGED , APTT IS PROLONGED
A NSWER IS B SCULLY PAGE 188

110-fatty change in the liver is due to the accumulation of ?


a- cholesterol
b-VLDL
c-LDL
d-triglycerides
answer is d from wikipidia

111- ASPIRIN SENSITIVE ASTHMA IS ASSOCIATED WITH ?


A - EXTRINSIC ASTHMA
B - URTICARIA
C - NASAL POLYPS
D - OBESITY
answer is c scully page 418 , patient allergy to aspirin have a tridal astham
which is asthma,nasal polyps and aspirin sensivity

112-most common clinical sign of pulmonary embolism ?


Plumonary infract and haemoptysis reference is scully page 201.it is also
associated with gradual pulomanry hypertension and right sided heart failure

90
91

113-Safest antibiotic and analgesics for diabetic patients ?


Amoxicillin, Ibuprufen and acetaminophen

114-Mycoplasma pneumoniae infection can lead to:


AHashimoto's thyroiditis.
BAgranulocytosis.
CCold hemagglutinin disease.
D Thrombocytopenic purpura
Answer is c

115-maximum life span of transfused RBC ?


Normal Adult RBC 120 days
Neonatal RBC 80 days
Transfused RBC 60 days

116- sleep apnoea is defined as a temporary pause in breathing during sleep


lasting at least how many seconds ?
10 seconds

117- normal plasma contains how many units of factor VIII per millilitre??
A)3
B)2
C)1
D)5
Answer is c

118- what is the function of von ville brand factor??


A)platelet adhesion to damaged endothelium
B)protects factor VIII from proteolytic degradation
C)both
Answer is c

119- warfarin acts by ??


A)prevent metabolism of vit K
B)enhance metabolism of vit K
C)decrease platelet aggregation
D)antifibrinolytic activity
Answer is a

120-systemic arterial hypertension leads to


A) left ventricular hyperthrophy
B) an increased incidence of infective endocarditis
C) both
D)niether
Answer a

91
92

121- right heart failure cause each of following except


A)splenomegaly
B)pulmonary-edema
C)ankle-edema
D)distended neck edema
Anser is b

122-Which one of the following is not an advantage of low molecular heparin


compared to i fractionated heparin
A) it can be given once or twice daily
B) there is a reduced risk of osteoporosis
C) it only has a half life of 30 min so is easily reversed
D) it does not need monitoring by blood test
Answer is c

123-What finding is not typical of type 2 diabetes


A)glicosuria
B)ketonemia
C)polyuria
D) high fasting glucose level
Answer is b

124-Which can not be transmitted by a needle stick during a patient care


A)hepatitis-A
B)hepatitis-B
C)hepatitis-C
D)HIV
answer is A

125- the most common site of ectopic pheochromocytoma ?


The medulla of the adrenal gland

126- Mantoux test :


A) Is carried out by intradermal inoculation of live attenuated bacilli.
B) Positivity indicates prior exposure of the individual to M. Tuberculosis.
C) Positivity indicates good immunity against TB
D) Is a measure of individuals humoral immunity
Answer is B
127-The cells most frequently found in a Granuloma are:
A: Mast cells
B: Giant cells
C: Lymphocytes
D: Neutrophils
E:RBC
Answer is b

128- Causes of folate Deficiency?

92
93

DIETARY DEFICIENCY (EG, ALCOHOLISM , DIETARY FADS)


MALABSORPTION (EG , COELIAC DISEASE , TROPICAL SPRUE, CONGENITAL
SPECIFIC MALABSORPTION , JEJUNAL RESECTION , INFLAMMATORY BOWEL DISEASE )
POOR INTAKE
OLD AGE
POOR SOCIAL CONDITIONS
MALNUTRITION
ALCOHOL EXCESS (ALSO CAUSES IMPAIRED UTILISATION)
POOR INTAKE DUE TO ANOREXIA
FOOD FADS

128-Mottled enamel is produced by:


a)Flourine
b)Acids
c)Tuberculosis
Answer is a

129- Characteristic features of Noonan's syndrome?


The three most common characteristics of Noonan syndrome are:
unusual facial features
short stature
congenital heart disease (heart defects present at birth)

130- The lesion that Characteristically occurs on the alveolar ridges of implants?
a)White spongy nevus
b)Fordyce granules
c)Bohn's nodules
d)Congenital lymphoma
Answer is c

131-Gohn’s focus is usually:


A)Subpleural
B)Peribronchial
C)Apical
D) In lyphnode
Answer is a

132- A 2year old girl was withdrawn from a day care center for
excessive irritability. On examination, she has multiple small
superficial ulcers covered with a grey membrane and surrounded
by a thin erythem
atous ring.The ulcerations heal spontaneously
over the next 5 days. Which of the following is the most likely
diagnosis?
a.Aphthous-stomatitis
b.Candidiasis

93
94

c.Gingivitis
d.Pyogenic granuloma
Answer is a

134-It is an inflammatory bowel disease affecting part or the whole of the large
intestine, most frequently the lower colon and rectum.
A-Irritable bowel syndrome
B.Coeliac disease
C. Ulcertative colitis
D.Diverticular disease

ANSWER is C

135-A spiral bacterium that lives in the stomach and duodenum which is the most
common contributory cause to peptic ulcers?
HELICOBACTER PYLORI

136-convulsive episodes occur when there is a severe deficiency of ?


a- riboflavin
b- thiamine
c-pyroidoxine
d-folic acid
answer is pyroidoxine

138-which one of the following drug is to be avoided in a patient of diabetes mellitus


with hypertension ?
a- nifedipine
b- methyldopa
c- captopril
d-propranolol
answer d

139- The pain of trigeminal neuralgia(true/false)


A) is indistinguishable from post- herpetic neuralgia
B) is indistinguishable from the trigeminal pain in some cases of multiple sclerosis in
younger people
C) is associated with flushing and nasal congestion on the affected side
D) consists of lightning- like stabs of pain of short duration
B,d is true

140-trigeminal neuralgia(T/F)
A)affects elderly adults
B)can spread to the opposite side
C)responds better to anticonvulsants than analgesic
D) is not precipitated by mastication

94
95

B,d is true

141-Hyperthyroid patient (T/F)


A)should usually be sedated before dental treatment
B)are subject to cardiac arrhythmias
C)hould be given Alfa- blockers to achieve rapid control of the signs and symptoms
D) are likely to be anxious, irritable, and difficult to manage
A,b,d are true

142- A 27 year old female complaining of oral pigmentation is considered possibly to


have Addison's disease. The following features would support this disease(T/F)
A) hypotension
B) weakness and lassitude
C) nausea and anorexia
D) vitiligo
Answerss are all true

143- Addison's disease typically


A) causes hyperpigmentation,low blood sodium levels and high blood pressure
B) is an autoimmune disease in majority
C)may develop hypotensive collapse during dental treatment
D) should carry a blue warning card
B,c,d is true

144- Patient with breathlessness and sleeps with 4 pillows , associated medical
condition is??
A)COPD
B)right heart failure
C)left heart failure
Answer is c

145- which communicable disease should be informed to clinician??


Hepatitis B

146- hand signs in mitral valve problems??


Splinter haemorrohage

147- hand sign in Rheumatoid arthritis?


Ulnar deviation is the pathognomonic feature of rheumatoid arthritis

148- Hand sign in inflammation of hand joints?


herberden nodes

150- In type 1 diabetes (insulin-dependent diabetes mellitus), the target of the


autoimmune attack is:
A All of the cells in the islets of Langerhans.
B The b-cells in the islets of Langerhans.

95
96

C The somatostatin-producing cells in the islets of Langerhans.


D The glucagon-producing cells in the islets of Langerhans.
E Cells throughout the body which have an insulin receptor

Answer is b

151- The inflammatory infiltrate in autoimmune thyroiditis comprises mostly:


APhagocytic cells.
BPolymorphonuclear leukocytes.
CLymphocytes.
D Mast cells.
E EIOSINOPHILIS
Answer is C

152-)Charcteristic sign of Asthma??


Expiratory wheeziness,dysponia,coughing , chest tightness ,nasal polyp
,tachycardia,use of accessory muscle to increase their ventilatory effort

Q5)what are the clinical features and of lung silicosis??


Lung silcosis is an occupational lung diseases affecting the miners,sandlasting and
potters the source is the inhalation of silica dust clinical features include dysponea
,coughing,fatigue,weight lost rapid breathing and chest pain.characteristic features
include pulmonary fibrosis leading to cor pulmonel,tuberculosis,

154-Hypo osmolar condition is seen in what condition?


Diabetes inspidus
155-Most difficulty in brushing teeth is seen in which disease ??
Rheumatoid artheritis

156-65 year old lady with forgetfulness & difficulty in doing daily tasks , what is the
condition??
Alzahmeir disease

What is the minimum platelet count required in a patient undergoing extraction??


Granulocyte(nuetrophilis) less than 500 and the t-tympocytes is less than 200

157-In which condition intra articular bleeding is minimal??


Haemophila A

158-What is the minimum platelet count required in a patient undergoing


extraction??
50*10(9)/L This is for normal extraction ,minimum amout of platelet for major syrgery
is 75*10(9)L and for normal La is 30*10(9)L

159-What is scleroderma?? how it effects dental treatment??


Scleroderma is an autoimmune disease caused by the excess profileration of collegen

96
97

by fibroblast.it is charicterised by prevascular macrophage infilteration ,endothilial


damage and increased collagen and extracellular matrix are the main features. Clinical
features are charectirized by CREST
C;calcinosis accumilation of calicium in the hands,fingers and face
R;raynoud syndrome
E:esophageal dysfunction
S:Sclerodactyly
T:telangistasia
Due to stiffness of skin constiction of oral orifice can cause limitation of the mouth
opening.involvement of peri-articular of the TMJ together with microstomia makes
dental care difficult.some patient may be associated with xerostomia,difficulties in self
care modifivation of toothbrush handle.
Treatment is by pencillemia which cause ulteration of taste and lechnoid reaction and
use of nefidipine cause gingival hyperplasia

160-A patient who is under weight, very conscious with calorific value of diet?? what
is the condition??
Anroxia nervosa

161-what is the age of onset of Type 1 diabetes?


Age of 12 years old

162-Philadelphia chromosome is present in which type of Leukemia??


Chronic myloid leukemia

163- Recurrent oral ulcers , Uveitis, and genital ulcers are seen in which condition??
Behcet disease

164-Patient with fatigue and excessive bleeding , what is the condition??


Luekemia

164-what are the dental characteristics in patient with bulilmia??


erosion of the palatal ,lingual and occlusal surface of the teeth due to recurrent self-
induced vomiting, oral ulceration or abrasion particularly on the soft palate may be
caused by finger or other objects used to induce vomiting

165-which one disease is most accurately described by this statement. There is


bleeding in mucous membranes, the platelet count is normal , factor VIII level may be
moderately reduced and partial thromboplastin time may be normal or prolonged.
A)haemophilia-A
B)haemophilia-B
C) Von Willebrand disease
D)immune thrombocytopenic purpura
answer is c

166- Dysphasia, sore throat, pharyngeal pain which occurs due to elongated styloid

97
98

process exerting pressure on pharyngeal wall is known as


A) troter's syndrome
B) Eagle' s syndrome
C) glosopharyngeal neuralgia
D) orofacial dyskinesia
Answer is b

167-in which of the following condition, serum alkaline phosphate se levels are
increased
A) Paget' s disease
B) osteogenesis imperfecta
C) osteoporosis
D) cherubism
Answer is a

168- A patient with petechiae on oral mucosa with gingival bleeding . Blood
examination shows platelet count of 30.00 mm3 with increase in bleeding time and
clot retraction time, RBC, TLC, are normal. Most probably diagnosis is
A)haemophilia
B)infectious-mononucleosis
C)thrombocytopenic-purpura
D) anemia
Answer is c

169-Philadelphia chromosome is characteristically seen in


A) acute lymphocytic leukaemia
B) acute monocytic leukaemia
C) chronic lymphocytic leukaemia
D) chronic myeloid leukaemia
Answer is d

170-a patientWith spontaneous necrotizing ulcers of oral cavity. On laboratory


examination, RBC count was normal but WBC count was 2.000 with lymphocytes 65%,
neutrophils 5%, monocytes 28%, eosinophils 2% and basophils 0%. The most likely
diagnosis is
A) infectious mononucleosis
B) agranulocytosis
C) cyclic neutropenia
D) leukaemia

Answer is b
Agranulocytosis, also known as agranulosis or granulopenia, is an acute condition
involving a severe and dangerous leukopenia (lowered white blood cell count), most
commonly of neutrophils causing a neutropenia in the circulating blood.[1][2] It is a
severe lack of one major class of infection-fighting white blood cells. People with this
condition are at very high risk of serious infections due to their suppressed

98
99

172-Craniofacial dysostodis with syndactyly is also known as


A) Crouzon 's syndromes
B) aperture syndrome
C) Trecher Collin's syndrome
D) evanceschetti syndrome
Answer is b

173-a 12 year old boy has a history of severe sore throat followed by migratory
arthralgia and swollen joints of the extremities.This history is suggestive of
A.gout.
B.osteoarthritis.
C.Still's-disease.
D.rheumatic-fever.
E. rheumatoid arthritis
Answer is d

174-rheumatoid arthritis is......type hypersensitivity reaction


a.type-1
b.type-2
c.type-3
d.type 4
Answer is type 3

175-Which of the following tissue is most sensitive to radiation induced carcinogenesis

A)thyroid
B)liver
C)salivary-gland
D) brain
Answer is a

176- Majority of lymphomas developed in patient of Sjögren's syndrome are of:


A) Hodgkin's lymphoma
B) Brukitt's lymphoma
C) non- Hodgkin lymphoma of B cell origin
D) non- Hodgkin's lymphoma of T cell origin
Answer is c
Lymphomas associated with sjorgen syndrome are b-cell in origin and musoca-
assciated type lympoma

177- Multiple lesions seen in patient suffering from Von Recklinghausen's disease of
skin are
A)neurofibromas
B)neurolemmomas
C)hemangiomas
D) griant cell tumour
Answer is a

99
100

Von recklinglinausen’s disease is a systemic diseases affecting the nerves with tumour
usually bengin developing causing compression on nerve tissue and surrounding
structure.

A patient comes to your clinic , before the start of treatment suddenly he complains of
severe chest pain radiating to left hand , you prescribe him GTN but the pain does not
reduce even after 3 mins , what may be the condition???
MI

179- Swelling in inter phalangeal joint is seen in which condition??


Rheumatic arthiritis

180- in medullary carcinoma thyroid tumor marker is ?


a- calcitonin
b- TSH
c-T3,T4 and TSH
d-alpha feto protein

answer is a

182-what is sarcadosis?
It is a multisystemic granulomatous disease mostly affecting the respiratory system.
young adult females.clinical features include lofgren syndrome which include;
Fever
Bilateral hilar lymphadionopathy
Arthralagia
Erythemia nodosum
Pulmonary infilteration and impared respiratory effeciency

183- what is difference between primary and secondary sjogren syndrome??


Primary sjorgen syndrome is not associated with CT diseases and xerostomia is more
than secondary sjorgen syndrome.

184- what is the difference between orofacial granulomatosis and crohns disease?
Orofacial granulomatosis has the similar oral biopsy features to crohns disease but no
detectable systemic disease findings. Orofacial granulomatosis could be a reaction to
some food or medicine .

185- In Asthma what happens to the airway??


hyper-reactivity causes reversible airway obstruction from smooth muscle
contraction(bronchospasm)mucosal oedema and mucus hypersecretion

186- A patient comes to your clinic , before the start of treatment suddenly he
complains of severe chest pain radiating to left hand , you prescribe him GTN but the
pain does not reduce even after 3 mins , what may be the condition???
Myocardial infraction

100
101

187- tension headache is due to the contraction of what muscles?


tension headache is a link ring or band arounf the head,frontalis ,occiptalis and
temporalis

188- what do you consider a seizure to be a status eplipticus ?


Major fit more than 5 min... Or if starts again

189- failure of the vertebral arches to fuse lead to what condition? Spinal bifida

190- what are the dental abnormalities associated with CLP?


malocclusion,hypodontia,enamel hypoplasia and supernumery teeth

191- brudzinski sign is positive in what disease?


meningitis
stroke
Parkinson disease
Answer is stroke
Reference scully page 328

192- a young patient with bilateral trigeminal neuralgia, prescription of


carbamazepine does not relief symptoms, what other conditions needs to be
considered?
Multiple sclerosis

193- what neurotransmitter secretion in decrease in parkinsons disease? dopamine

194- submucosa cleft is:


palate fuses but not mucosa
mucosa fuses but not palate
both do not fuse
Answer is mucoas fuses but not palate

195- treatment courses for TB patient?


Isoniazid , rifampicin, pyrazinamide, ethambutol for 2 months... Followed by
isoniazid& rifampicin for 4 months

196- viral meningitis is usually fatal ?true/false


Answer is false

197- development o the premaxilla is during what weeks of gestation? 4-7 weeks

198- what is the time scale of correction of CLP? 6-12 months

199- irreversible disease with destruction of the alveoli?


bronchitis
emphysema
Answer is emphysemia

101
102

200- chevostecks sign is positive in?


hyperparathyroidism
hypoparathyrodism
hyperthyroidism
hypothyrodism
Answer is hypo parathyrodism reference is scully 158

201- what is the antagonist for the following medication/


heparin
warfarin
Answer is protamine sulphate and vitamin K

202- a patient came for a molar extraction ,on extraction tooth appeared to be
immobilized, xray reviled hypercementosis,what do you think is the clinical diagnosis/
hyperparathyroidism
paget disease
fibrous dysplasia
Paget disease is associted with hypercementosis of the teeth….hyperparathyrodism is
associated too

203- parathyroid hormone is regulated by what ion? calicium

204- what is the correct answers


1-paget dieases mostly affects the
maxilla
mandible
2-osteoradionecrosis mostly affects the
spongy bone
Spongy bone
Compact bone
1-maxillae 2-compact bone

205- cotton wool appereance is of what condition? Paget disease

206- patient with bilateral swelling of parotid gland?


1-tuberculosis
2-sarcodiosis
3-osteonecrosis
Answer is sarcodosis

207- what bisphosphate drug is of high risk?


1-etidronate
2-clorodronate
3-zoledronate
4-tiludronate
Answer is 3

102
103

208-cystic fibrosis is a
autosomal dominant trait
autosomal recessive trait
Answer is autosomal recessive

209- Serum alkaline is increased in all except


Pagets
Osteopetrosis
Rickets
Sarcoidosis

Answer is oeteoprosis
210- what is the vaccination to prevent TB?
BGC
211- alkaline phosphate in paget disease are/
increased
decreased
normal
Increased

212- Rickets is seen in


Child
Adults
Both
Child

213- what is the type of cells found in TB?


Granulomatous Langerhans cells

214- What is associated with COPD


Respiratory alkalosis
Respiratory acidosis
Metabolic acidosis
Metabolic alkalosis

Answer is Respiratory acidosis


215- Which condition is associated with trigerminal neuralgia ?
Tension-headache
Migraine
Glossopharangeal-neuralgia
Migranous neuralgia
Answer is glossopharyngeal neurolagia

216-Most common seizure of childhood


Grand-mal
Petit-mal

103
104

Jacksonian
Answer is petit mal

217- The tongue in protrusion deviates to the affected side in


Hypoglossal n lesion
LMN lesion
Answer is hypoglossal

219- Which is d most common epilepsy


Grand mal
Petit mal
Partial seizures
Answer is partial

220- Which is most common in males


Cleft-lip
Cleft-palate
Cleft lip & cleft palate
Answer is cleft lip,females are associated with cleft palate

221- Greenish discolouration if teeth & dental hypo plasia r seen in


Congenital jaundice
Lung cancer
Primary biliary cirrhosis
Hepatitis
Answer is congenital jaundice

222- the most common type of authous ulcer in beceht disease?


Minor

223- Persistence if Hbs Ag beyond 13 weeks implies?


Acute infection
Non infection
Carrier state
Carrier state

224- which clotting factor is associated with the following conditions?


a-xmas disease ? factor 9
b-von williebrands disease? Factor 8
c-haemoghilic A ? factor 8
d-dissemenated intravascular coagulopathy ?

225- oral manifestations of coeliac disease?


Enamel-defects
Glossitis

104
105

Burning-mouth
Angular-stomatitis
Ulcers ...

226- A patient recently underwent Surgical resection for Crohn's disease, which
vitamin deficiency will he b suffering from?
Vit-A
Vit-C
Vit-B12
Vit D
Vitamin B12

227- Nitro glycerine is contraindicated in


MI
Angina
Arythmmia
Cardiomyopathy
Answer is Cardiomyopathy

228-What are the incubation periods of the following viral hepatitis:


1.HAV
2.HBV
3.HCV
4.HDV
5. HeV
Anawer hav 2-6 weeks
Hbv 2-6 months
Hcv 2-22 weeks
Hdv 3 weeks to 2 months
Hev 2-9 weeks

1-factor 8
2-factor 11
3-factor 9
4- factor 2,7,9,10
5-factor 1
6-factor 8 and platelet
229.Which of the following statements are true regarding the various types of
diabetes mellitus?
A) type2 is commonly caused by destruction of the pancreatic islet cells leading to
insulin insufficiency
B) type 2 diabetes is often associated with obesity
C) gestational diabetes is always controlled by diet alone
D) patient with type 1 diabetes are more likely to get ketosis than those with type 2
diabetes
True are B,C,D

105
106

230.for which of the following conditions might a patient be taking bisphosphonate


drugs?
A) bone metastasis from breast cancer
B) multiple myeloma
C) odontogenic myxoma
D) osteoporosis
TRUE IS A,B,D

231.pregnant woman
A) can present with an epulis
B) rarely get gingivitis
C) may become hypotensive when supine
D)can take aspirin safety
TRUE IS A,C

232. Which of the following measures are appropriate for managing a patient
experiencing an addisonian crisis?
A) place patient in horizontal position
B) give glucagon
C) give intravenous hydrocortisone
D)call for medical assistance
TRUE ARE A,C,D

233. He 9.5g/dl, WBC 5.3x10(9)/l, platelets 200x10(9)/l, RBC 4.7x10(9)/l, MCV 76 fl,
MCH 21.8pg.this blood film:
A) is consistent with anaemia
B) shows microcytic anaemia
C) shows macrocytic anaemia
D) is consistent with iron deficiency anaemia
True answers are A,B,D

234.patient with which of the following conditions may be on long term


anticoagulants?
A)atrial fibrillation
B) previous deep vein thrombosis
C) cardiac pacemaker
D) prosthetic heart valves
ANSWERS ARE A,B,D

235.a patient who weighs 60 kg and is 1 m 50 cm in height would have the following
body mass index( BMI)
A) 20
B) 24
C) 28
D)32
ANSWER IS B AND C

106
107

236.Regarding vitamin deficiency


A) vitamin A deficiency causes scurvy
B) vitamin C deficiency causes beri beri
C) vitamin D deficiency in children causes rickets
D) vitamin D deficiency in children causes delayed tooth eruption
ANSWER IS C,D

237. Which of the following are causes of finger clubbing?


A) bacterial endocarditis
B) bronchial carcinoma
C)congenital cya optic cardiac disease
D) angina
ANSWER IS A,B,C

238.features of bacterial endocarditis include


A) boutonnière's deformity
B) night sweats
C) history of recent dental extractions
D) splinter haemorrhages
ANSWER IS B,C,D

239-what is the choice of treatment for Palatal ulcers seen in neonates with cleft lip
and palate that appear to result from trauma from the tongue ???
A)surgery
B)palatal plate
C)occlusal splint
Answer is b

240-a diabetic patient is given oral glucose when the RBS is


5.5 mmol / l
3.2 mmol/ L
< 5mmol / l
> 2 and < 4
Answer is less than 5mmol/l

241-Small punched out osteolytic lesions involving the posterior mandible are typically
seen in ??
A)CML
B)AML
C)multiple myeloma
D)SECONDARY macroglobulemia
Answer is c

242-Hand signs
A) clubbing
B) Osler nodes

107
108

C) leuconychia
D)heberden nodes
For each of the following scenarios, choose the most appropriate hand sign from list
above.
I) a 25 year old man with infective endocarditis
II) 73 year old with squamous cell carcinoma of the bronchus
III) 25 year old with hypoalbuminaemia from nephrotic syndrome
IV)58 year old with generalised osteoarthritis
Answer is 1-b,2-a,3-c,4-d

243-Chest pain
A) angina
B) aortic aneurysm
C) myocardial infarction
D) oesophagitis
E) pulmonary embolism
For each of the following scenarios, choose the most appropriate diagnosis from list
above.
I) a 60 year old with acute sever chest pain radiating through to the back with a wide
mediastinum on chest x- Ray
II) a 75 year old smoker with central chest pain radiating to the jaw when he walks to
the shops. This pain resolves at rest
III) a 40 year old woman with right sides chest pain, worse on breathing in. There is
associated coughing and haemoptysis
IV) 53 year old man whit sudden onset chest pain, radiating to the left arm, associate
with sweating and nausea. The pain has been present for 2 hours.
Answer is 1-b,2-a,3-e,4-c

243-1.Endocrine disease
A) achondroplasia
B) acromegaly
C) Cushing syndrome
D) hypothyroidism
E) Addison's disease
For each of the following statements, choose the most appropriate diagnosis from the
list above.
I) a 76 year old woman presents to her GP complaining of weight gain, dry brittle hair
and lethargy
II) a 19 year old man presents to emergency department complaining of a gradual loss
of peripheral vision. On inspection of the oral cavity you notice prognathisam and a
large tongue
III) a 41 year old present to the emergency department with vomiting, diarrhoea and
abdominal pains. She is hypotensive and tachycardia. Bam analysis reveals
hypoglycaemia
IV)a 58 year old woman with a past medical history of poly myalgia rheumatica
presents with weight gain , acne and oral candidias

108
109

244-Which one of the following is not cause of a neck lump


A)goitre
B)lymphadenopathy
C)cystic-hygroma
D) subungual naevus
Answer is d

245.Which one of the following circumstances would not raise suspicion of physical
abuse in child?
A) a skull fracture in 6-week old baby who allegedly rolled off the bed
B) a torn fraenum in 2- year old child
C)bruises of various ages over the shins of 6 year old child
D) a report by an 8 year old sibling of an excessive beating
Answer is c

246-When is the best time of the day to treat diabetic patients in order to prevent
hypoglycaemic collpase?
Within 2 h of breakfast and the morning insulin injection

247-Give the hand signs for the following:

1. Rheumatoid arthiritis.
2. Osteoarthritis
3. Lung diease
4. Endocarditis, SLE
5. Liver disease
6. Iron deficiency anaemia
7. Psoriasis, Lichen planus
8. Jaundice and gall stones
9. Diabetes mellitus
10. Hypocalcaemia
11. Nephritic syndrome
Answer is
1. Ulnar deviation
2. Heberden’s nodes
3. Finger clubbing
4. Splinter haemorrhage
5. Palmar erythema
6. Koilonychia
7.leuknykia
8. Nail scratches or nail scratches
9. Paronychia and sclerodacity(thickening of the skin of the hands and is common sign
in BM type1
10. Beau’s lines,carpodepal (spasm of the muscles of the thumb that brings it to the
palm)
11. Leukonychia

109
110

Name of thalassemia test?


•Red blood cells will appear small and abnormally shaped when looked at under a
microscope.
•A complete blood count (CBC) reveals anemia.
•A test called hemoglobin electrophoresis shows the presence of an abnormal form of
hemoglobin.
•A test called mutational analysis can help detect alpha thalassemia

Why the patient with heart failure bleed easily and how to mange such bleeding?

Appointment for asthmatic patient should be given in?


A morning
B Afternoon
C Evening
D Anytime
Answer is b

What are the clinicl finding of FBC ?and how do you manage with patient needing GA?
Clinical problems include six :
1-aneamia
2-crises (painful,aplastic ,dactylitis,infracts of CNS KIDNEY SPLEEN BONE,skin ulcer)
3-haemolysis(jaundice,gallstones,reticulocytosis)
4-imparied growth
5-skeletal deformities
6-suspectibility to infection

Seriological finding include the following :Hb levels in below 9g/dl(5-9)


Reticulocytosis of 5-25%
In sickle cell disease Hb electrophoresis shows mainly HbS and
15% HbF but no HbA
In sickle cell trait Hbs is 40% and HbA is 60%

GA is a hazard for sickle cell patients it carries the risk of alloimmunization in up to


20%
In triat sickle cell if GA is necessary full oxygenation must be maintained .in sickle cell
disease Hb must be corrected up tp at least 10

Clinical features of down syndrome?


Short stature,learning disability,branchycephaly,congenital cardiac disorder
orofacial abnormality :omega shaped palate ,hypodontia,microdontia,,delayed
eruption in both dentition ,bruxism,class III occlusion ,anterior open bite due to
tongue thrust ,retrusive mid face.
Soft tissue involves macroglossia ,tongue fissure ,lip fissure ,chelitis ,drooling .

110
111

What are the clinical features of bronchitis?


Chronic coughing ,excessive production of mucus ,suptum for over 3 months a year

What is cystic fibrosis?


Is a inherted defect affects the exocrine gland so sweat and salivary sodium levels are
raised and mucus is viscid.involved glands may become obstructed
Autosomal resscive trait on the chromosome 7q
It encodes the membrane protein and this leads to cystic fibrosis transmembrane
regulator cAMP which regulates the Cl channels and therefore control the amount of
Na and Cl transport through the epithilia membrance .
Clinical features involve pancreatitis ,infection,sinusitis and growth hormone .

A patient underwent radiotherapy 10 yrs back answer the following questions.


I. if this patient undergoes a third molar extraction he is likely to have -
II. if he is on warfarin he is likely to have -
III. if his third molar lies close to id nerve-
IV. if he gets infection of molar-
V. if he takes steroid and azathioprine-
Answers are I.dry socket,II.bleeding,III.parasthesia,IV.osteomylitis,V.cause fracture
because of the steroids which cause osteoporosis,and bleeding because of
azathioprine which cause macrocytic anaemia

Treatment of rheumatic fever ?


Penicillin is drug of choice and for prophylaxis phenoxymethyl pencillin up to the age
of 20 or sulphadimidine if allergic .

Type of anaemia in rheumatoid arthritis ?


The most common causes of anemia in patients with rheumatoid arthritis are the
anemia of chronic disease (ACD) and iron deficiency anemia (IDA). ACD is more
common than IDA in RA patients, occurring in up to 77% of anemic RA patients.2 In
fact, anemia in RA patients has served as a model for the anemia of chronic disease.2
Differentiating the types of anemia is important in planning diagnostic testing and in
guiding therapy. In ACD, hemoglobin levels are higher than in IDA, ferritin levels tend
to be steady or increasing, and the anemia is most often normocytic and
monochromic.2 If ferritin is decreased, and the anemia is hypochromic, IDA is much
more likely.
It is estimated that iron deficiency anemia occurs in approximately 23% of anemic RA
patients.2 However, Iron deficiency anemia often coexists with ACD in RA patients.9 It
is generally a hypochromic, microcytic anemia most commonly due to gastrointestinal
bleeding secondary to nonsteroidal anti-inflammatory drugs (NSAIDs), or
corticosteroid therapy. “It’s important to recognize,” Dr. Furst said, “that iron
deficiency anemia is not part of RA. It’s the drugs we use to treat our patients, such as
the NSAIDs and the DMARDs (disease-modifying antirheumatic drugs)

Tingling sensation in the extremities during long standing procedure?


Paraesthesia

111
112

Type of immune cells in rheumatoid artheritis?


t-lymphocyte particularly CD4

stains for candida diagnosis?


PAS(periodic acid Schiff stain)

Effect of chemotherapy?or ORAL MANIFESTATIONS IN THE TREATMENT OF GENERAL


CANCER.?
Mucositosis,infection,bleeding ,xerostomia

Type of cell in sarcoidosis ?


Macrophages ,plasma cells,lymphocytes

Questions on Multiple myeloma ?


disseminated plasma cell neoplasm (uncommon grp of B lympho disorders).detected
by electrophoresis by over production of specific IG.(IgG 50%,IgA 25). Predominantly
causes bone lesions.Malignant plasma cells produce defective Igs which release
osteoclast activating factors that cause bone resorption and pain.Disease of middle
aged and elderly.Detected by high ESR,leucopenia,thromocytopenia in routine blood
tests and bence jones proteinuria. Causes bone destruction
,hypercalcemia,suppression of haemopoiesis,normochromic anemia.Osteolytic lesions
in skull radiographs.(round discrete punched out osteolytic lesions). Amyloid may be
deposited in oral soft tissues causing macroglossia.Tx ,chemotherapy with
corticosteroids and Bisphosphonates.

what is the test done for peptic ulcer??


h.pylori detection test:
1-blood test for antibodies to h pylori,
2-urea breathe test..
3-stool antigen test..
4-stomach biopsy,,

Bence jonce proteins & protein urea are seen in ? Multiple myeloma

Periodontolgy
1-gingival inflammation in a patient wearing removable partial denture?

Presence of plaque accumulation

2- Which type of gingival fibers attaches to cementum of adjacent teeth and is


present over the alveolar crest?

a. Alveolar crest fibers

b. Alveologingival fibers

112
113

c-transseptal fibers

d. Circular fibers

answer is c

3- The escapement spaces between teeth and the interdental spaces are called:

a. Marginal ridges.

b. Contact areas.

c. Embrasures.

d. Developmental grooves

answer is c

4-.Perforating fibers consisting of collagen fibers embedded in alveolar bone


proper are known as:

a. Gingival fibers

b- sharpeys fibers

c. Transseptal fibers

d. Alveolar fibers

answer is b

5-. the absence of adequate drainage in a periodontal pocket may result in

A- Cyst formation

B-abscess formation

C- Epithelial hyperplasia

D- Increased calculus formation

Answer...b

6-. For an otherwise healthy patient, with an acute localized periodontal abscess,

initial treatment must include.

A-scaling and polishing

B- Occlusal adjustment

C- Prescription of an antibiotic

D- Prescription of an analgesic

Answer is A

113
114

7- the oral mucosa covering the base of the alveolar bone

A- is normally non-keratinized but can become keratinized in response to physiological


stimulation

B- is closely bound to underlying muscle and bone

C- does not contain elastic fibers

D-merges the kiratinized gingival at the mucogingival junction

E- has a tightly woven dense collagenous corium

Answer is D

8-. The periodontium is the best able to tolerate forces directed to a tooth

A- horizontally

B- laterally

C- obliquely

D—vertically

Answer is D

9- Clinical scenarios for various types of periodontal diseases – Chronic periodontitis,


aggressive/early onset periodontitis, acute periodontitis, chronic gingivitis.Differentiate
between them in terms of signs, symptoms and ages.

Chronic periodontitis...slow to moderate destruction,elder patient ,associated with


subgingival plaque, no marked pattern of distruction.and the amount of plaque deposite is
consistant with the distruction of bone

Aggressive periodontitis....rapid destruction ,adolescents ,presence or absences of


subgingival calculus,familial triat

Acute periodontitis....pain on occlusion,tooth feels slightly elevated,no change in radiograph


only slight widening in the lamina dura, tooth is not sensitive to hot and cold,tooth is tender
to the slightest touch,tooth has a large carious lesion

Chronic gingivitis.....bleeding and swelling gums,and gingival pockect more than


2mm,affects any age

10- Percentage of adults over 16 with (loss of attachment or pocketing-can’t remember


which one) greater than 3.5mm.

45% refrence Dr irfan statistic

11- What is the concentration of chlorhexidine in a gel?

0.2%, 0.5%, 1%, 5%

114
115

I think Answer is 1%

12- What is the concentration of chlorhexidine in mouthwash?

0.2%,0.5%,1%,5%

Answer is 0.2%

13- There was an EMQ on bleeding index, gingival index and oral hygiene index. Some of the
questions were Score for bleeding with probing, spontaneous bleeding, plaque seen only on
disclosing etc

14- Which antibiotic is used in cases of necrotising ulcerative gingivitis?

Metronidazole, clindamycin, myconazole, ketoconazole, tetracycline

Answer is metronedazole 200mg8TDS for 3 days

15- Which type of cells is seen in the early stage of chronic periodontitis?

Basophils, eosinophils, lymphocytes, neutrophils, macrophages

Answer i think is lymphocyte and neutrophilis reference cawson’s page 82

16- Which type of cells is seen in late stage of chronic periodontitis?

I think plasma cell and macrophages

17- Which type of cells is seen after early stage of chronid periodontitis?

Basophils, eosinophils, macrophages, lymphocytes

Cawsons says plasma cells page 82

18- Junction epithelium. Its anatomy and position

Starts from the bottom of the gingival sulcus to the CEJ 2mm attached to the tooth surface it
is a thin layer composed of 3-4 cells which ae non-keratinsed epithilium

19- Gingival index, plaque index, Loe and Sillness index many questions were asked

20- Which cells are present in acute periodontitis

Neutrophils and leucocytes

21- Which cells are present in chronic periodontitis

Inflammatory cells in chronic periodontitis is luecocyte, macrophages and most prominent


cell is plasma cells

115
116

22- Lots of questions on periodontolgy in paper 2.Curretes and design to improve


efficacy.

23-5 questions on periodontal ligaments – how it’s thickness varies;(thinnest in the


middle third) the type of collagen, main fibres of ligament (oblique) (Type 1 and 3),
how it differs from children pdl? Width decrease with age
24- What is required to increase the efficiency and decrease the fatigue of Universal
curette?
Same question repeated 6 times with 4 different choices
I think these are the answers (Ans: sharp, parallel to long axis of tooth surface, lower
shank should be parallel to tooth surface, light weight, small ribbed surface, use a
modified pen grasp.)
25- The ideal rake angle required for the universal curette in sub gingival curettage.
Ans: not sure. (-20,0,15,20) I found out that the angle is 70
26-A patient with gingivitis, how is it treated ?
Oral hygien instruction and chlorohexidine gluconate mouth wash

27- Periodontal calculus removal – 5 questions about instruments (Push


Scaler, Curette, Sickle, Hoe, other different names that I don’t have any
idea what are…)

Push scaler,chisel scaler....removal of supragingival calculus from the proximal


surfaces of anterior maxillary teeth
Gracey Curette...removal of subgingival calculus and furcation
Universal curettes......removal of supra and specially subgingival calculus from the
tooth used following the hoe in root scaling
Sickle scaler...used to remove calculus calculus from the embracer supra and
subgingival 2-3mm
Jacquette scaler...remove supra gingival calculus
Periodontal file,hiresfeld...to crush calculus to facilitate its removal with other
instruments..

28- Best solution to control subgingival bacteria? clorhexidine 0.2%

29-Periodontal condition 5 sub questions – localised, aggressive


periodontitis, ANUG, gingivitis, chronic periodontitis

30- Lots of Qs on indices which were asked typically from master dentistry

116
117

31- Clinical scenario reg peridontitis


32- percentage of periodontities and tooth with plaque in 1998, also caries in less than 12
years
Caries in children less than 12 year old 38% refrence dr irfam statistic
33-type of cell in periodontitis?
Lymphocytes
34- micro organism most responsible for aggressive periodontitis?
Aggregitabacter actinomycetecomitans
35- A man has pockets of more than 5 mm with many teeth. The number of teeth remaining
was given. Which radiograph should the dentist advice?
i think it should be vertical cause the probing measure is more than 5

36- Patient well motivated, good OH, furcation lesion Class II. What’s the best treatment?
GTR with graft and EMD

OHI scaling and root planing, furcation plasty,tunnelling,root sectioning and extraction

37- Best interdental cleaning for perio patients:


interdental brushes

38- periodontitis affecting PMs and molars in a 35 year old lady

39- Periodontitis affecting molars and PMs 4-5 mm pocket, +++plaque


Generalized aggrisive periodontitis...not sure
40- Man 70+ with generalised recession, 5-6mm pockets? Chronic adult periodontitis
41- man with 2-3 mm pocket measurements & plaque? gingivitis

42- Matching questions with terms like gingival , 5-6mm probing depth, sites of mobility
….options-acute perio, chronic perio, acute gingivitis, juvenile, early onset…not quite
straight forward!

43- What’s the percentage of >25 years with periodontitis in UK?

64%...reference is dr irfan statistic

44- Patient well motivated, good OH, furcation lesion Class II. What’s the best treatment?
Guided tissue regeneration, scale and root planing, etc

45- Chronic and acute periodontitis, chronic and acute gingivatis, early onset / aggressive
periodontitis, periapical periodontitis (association with given cases)

46- Best interdental cleaning for perio patients: interdental brushes, dental floss, tooth
picks, etc

Anser is interdental brushes

47- Plaue scores, debris index loe scores

117
118

48- peri ques again clinical cases and what treatment would you give to patient
eg. patient with radiotherapy in head and neck region
49-which cell are found in herpetic stomatitis?
a-lymphocytes
b-monocytes
c-neutrophilis
d-macro cells
e-plasma cells
answer is c .

Q2 A patient on examination was found to have swollen gingiva around a crown that had
been present for several years. The papillae were particularly enlarged.
What is the most important feature of a crown that may be responsible for this?

A. Material of the Crown

B. The occlusion

C. Proximal Contour

D. Labial Contour

E. Surface finish
Answer is A

Q40 Membranes of expanded polytetrafluorethylene have been designed for periodontal


regenerative techniques.
Which of the following defects will respond most predictably to regenerative therapy?

A. Shallow, wide 1-wall defect

B. Shallow, wide 2-wall defect

C. Deep, narrow 3-wall defect

118
119

D. Deep narrow 1-wall defect

E. Shallow, narrow 2-wall defect

Answer is c

Q41 Hamp (1975) classified furcation defects as degree I, II or III.


Which of the following is the ideal treatment for a degree II furcation involvement of a
mandibular molar?

A. Tunnel preparation

B. Root resection

C. Furcation plasty

D. Extraction

E. Guided Tissue Regeneration

Anwer is e pink book page 212

Q42 You examine a patient and find BPE code 4 in all sextants. Radiographs show
generalised horizontal bone loss with a minimum of 50% of bone support remaining on all
teeth.
Which of the following is the most important factor when considering the prognosis for the
teeth?

A. Age of the patient

B. The Oral Hygiene Status

C. Bleeding on Probing Score

D. Mobility

E. Gingival Recession
Answer I think is oral hygiene status others think its mobiltly

Q51 Page and Schroeder (1976) in their description of the pathogenesis of


periodontal disease described four stages in the process, the initial, early,
established and advanced lesion.
Which of the following is the main feature defining the early lesion?
A. Emigration of the phagocytes through the junctional epithelium.

119
120

B. Presence of lymphocytes which are mainly T cells


C. Increase in crevicular fluid.
D. Predominance of plasma cells and B lymphocytes
E. Proliferation of the junctional epithelium.
Answer is c

Q67 “The biological process by which the architecture and function of the lost tissue
is completely restored”.
In Periodontology this is a definition of which of the following
A. New Attachment
B. Regeneration
C. Repair
D. Primary Healing
E. Reattachment

Q80 Chronic periodontitis is a disease of the periodontium initiated and sustained by


microbial plaque.
Which of the following is the main diagnostic feature of the disease?
A. Tooth Mobility
B. Loss of attachment
C. Pain
D. Gingival recession
E. Drifting
Answer is b

Q81 Gingivitis and periodontitis are associated with the loss of normal tissue.
Which of the following is the principal cause of the tissue loss?
A. Bacterial enzymes such as collagenase and hyaluronidase
B. Release of lysosomal enzymes and oxygen free radicals by PMNs and
macrophages

120
121

C. Cytotoxic metabolic by products such as ammonia, hydrogen sulphide and toxic


amides
D. Complement activation
E. Release of bacterial endotoxins and exotoxins.
Answer is B

Q82 Localised gingival recession is recession of the gingiva confined to one surface
of the tooth.
Which of the following is the most important factor leading to the development of
localised gingival recession?
A. Habit activity.
B. Class 2 division 2 malocclusion with complete overbite
C. Excessive toothbrushing
D. Bony dehiscence
E. Fenestration
Answer is B/ I think its c most common tooth associated with gingival reduction due
to tooth brushing is the canine

Q89 Gracey curettes are specially designed for subgingival debridement and are site
specific.
Which curette is recommended for use on the lingual surface of a lower second
molar?
A. Gracey Curette number 1 and 2.
B. Gracey Currette number 3 and 4
C. Gracey Currette number 7 and 8.
D. Gracey Currette number 11 and 12.
E. Gracey Currette number 13 and 14.
Answer is c

Q94

121
122

on probing A new patient attends your practice. At the initial examination, you carry
out a Basic Periodontal Examination (BPE).
What does a BPE score of 1 signify?
A. Periodontal health
B. Probing depths greater than 3.5mm
C. Presence of overhanging restorations
D. Presence of supragingival calculus
E. Presence of bleeding
Answer is e

prevention;

1. What will be the answer?


Stephan's curve represents
a) change in pH of saliva with time
b) change in pH of plaque with time
c) change in pH of saliva with sugar intake.

Answer: B

EXPLANATION: BENEFICIAL EFFECTS OF SALIVA IN REDUCING PLAQUE PH AND


THEREFORE CARIES

 Dr Helen Whelton, Director Oral Health Services Research Centre, University Dental School and Hospital,
Wilton. Cork, Ireland. Email: H.Whelton@ucc.ie

Dental caries is an infectious, communicable disease resulting in demineralisation and destruction of tooth
structure by acid-forming bacteria found in dental plaque, an intraoral biofilm, in the presence of sugar 1. Far
from being static, dental enamel is in a constant state of change in that it undergoes cycles of
demineralisation and remineralisation. Acid produced by oral bacteria in the glycolysis of carbohydrates is
the driving force towards enamel demineralisation and caries progression. Saliva on the other hand is the
driver of repair and remineralisation. In the absence of the protective properties of saliva, acid dissolution of
the teeth would progress unchecked in susceptible individuals with cariogenic bacteria and unfavourable
diets. The fundamental aims of strategies for caries prevention are to reduce demineralisation and/or to
increase remineralisation.

The caries process is illustrated in Figure 1. Interaction between three factors over time is needed for caries to
develop; a susceptible host i.e. a tooth surface, microflora with cariogenic potential i.e. plaque, and
fermentable carbohydrate i.e. sugar.

122
123

Figure 1 Factors necessary for the development of dental caries

The presence of saliva is vital to the maintenance of healthy oral tissues. Its importance in preventing caries
was first illustrated by feeding a cariogenic diet to desalivated rats which developed over four times as much
caries as rats with
intact salivary glands fed the same diet. In humans, although there is a lot of variation in saliva secretion rates,
the average is 0.3ml/minute for unstimulated and 1-2ml per minute for stimulated saliva. The rampant caries
seen in some xerostomia individuals (<0.1ml saliva produced per minute, unstimulated salivary flow rate) is
indicative of the devastation of the dentition that occurs in the absence of saliva. In healthy subjects the teeth
are constantly bathed in 0.5ml of unstimulated saliva which coats them with a film approximately 10µm thick
and which moves as the muscles of the mouth are used.

The impact of saliva in neutralising plaque pH was illustrated by Stephan 2 who measured the changes in plaque
acidity following sugar intake. He used indwelling antimony microelectrodes to monitor changes in plaque pH
in situ following a sucrose rinse. The characteristics of the Stephan Curve are illustrated in Figure 2.

Figure 2 Changes in plaque pH over time following a sucrose rinse

Characteristically, the Stephan Curve shows a rapid drop in plaque pH which is followed by a slow rise until
resting pH is attained. The critical pH below which demineralisation of enamel generally takes place is 5.5.Thus
the shaded part of
the curve indicates the period of demineralisation. The time taken for these changes to occur varies between
individuals and also varies according to the nature of the challenge. The drop in pH usually takes no more than

123
124

five minutes whereas the recovery for the resting pH can take between fifteen and forty minutes depending on
the acid neutralising properties of an individual’s saliva. The initial sharp drop depends upon the speed with
which plaque microbes are able to metabolise sugar.

Large molecules like starch for example, diffuse into the plaque more slowly and take longer to be broken
down resulting in a less steep drop in pH. The rise in pH is dependent upon diffusion of acid by-products out of
the plaque and, secondly salivary bicarbonate diffusing into the plaque buffering the acid by-products. One of
the most important factors governing the overall shape of a Stephan Curve, but particularly the pH recovery, is
saliva flow rate. Saliva exerts two effects. First, it dilutes and carries away metabolites diffusing out of the
plaque. Second it supplies bicarbonate ions which diffuse into plaque and neutralize the by-products of
fermentation (organic acids) in situ. The bicarbonate-mediated acid neutralisation effect is enhanced by the
increase in salivary bicarbonate associated with increased saliva flow which coincides with eating or chewing
e.g. chewing gum.

Source: http://betteroralhealth.info/orbit_complete/professional-area/resources/cpd/saliva-
benefits/beneficial-effects-of-saliva-in-reducing-plaque-ph-and-therefore-caries/index.htm

2-. Cause of early child hood caries? (ECC)

Answer: Fruit based drink

3-. Reducing cariogenicity

1. sucrose, glucose, fructose, maltose (honey)

2. Galactose, lactose

3. Complex carbohydrate (e.g. starch in rice, bread, potatoes)

Foods that stimulate salivary flow and can speed the return of plaque PH to normal

E.g. cheese, sugar free gum, slated peanuts.

4-) which is the drug used for the treatment of tobacco dependance??

A)Bupropion
B)Buprenorphine
C)Benzodiazepine

Answer is a

5-in a studying the impact of dental carries on communities the care index is sometimes
used.this is denoted by the formula F/DMF.what is the most approptiate interpretation of
the care index?

a-shows the incidence of new decays

b-shows the extent to which decay is prevented

124
125

c-shows the prevelancde of decay in the community

d-shows the exyend to which the decay is treated restoratively

e-shows the extent to which the decay is treated by extraction

answer is d

6-number of people who die in the uk from oral cancer?

1000-1500

7-how often do you review to review smoking status?

At least once a year

8-best option of fluoride in non fluoridated areas/

Topical fluoride(tooth paste)

9-fluoride in tooth paste for a 8 year old child?

1350ppm pea size on tooth brush

10- Which of the following is NOT a major mechanism of action for fluoride in caries
inhibition?
a. Increases remineralization of enamel.
b. Inhibits carbohydrate metabolism
c. Reduces enamel solubility.
D. ability to prevent reduction of the pH of plaque
Answer:B

Statistic and evidence based dentistry


1-what is the percentage of dental trauma in both females and males aged 12 years old?

Boys 33%

Girls 19%

2-What is the survival rate of cancer in stage 1?

85%

3-percentage of louridation of water in the UK?

10%

4- WHAT IS THE MOST FLUORIDATED AREA IN THE UK?

125
126

FIRST IS WEST MIDLAND FOLLOWED BY EAST NORTH OF ENGLAND

5-MAXIMUM UNITS OF ALCOHOL ALLOWED TO BOTH SEXES A WEEK?


MALES 21 UNITS/WEEK OR 3-4 UNITS/DAY WITH 2 DAYS WITHOUT ALCOHOL FOR 2 DAYS
FEMALES 14 UNIT/ WEEK OR 2-3UNITS/DAY WITH 2 DAYS ALCOHOL FREE

6- CHILDREN AGED 5 AND UNDER FREE OF CARIES?


55%

7-PERCENTAGE OF ALCOHOLICS IN THE UK?


72% males and 57% females

8-PERCENTAGE OF ALCOHOLICS IN THE UK?


MALES 50% AND FEMALES 30%

9- BITEWING RADIOGRAPHY IS THE MAIN SPECIAL TEXT USED TO HELP IN DIAGNOSIS OF PROXIMAL
CARIES. THE PERFORMANCE (ACCURACY) OF A DIAGNOSTIC TEST LIKE BITEWING RADIOGRAPHY CAN BE
EXPRESSED IN TERMS OF SENSITIVITY AND SPECIFICITY .
WHICH OF THE FOLLOWING IS A REASONABLE SUMMARY OF THE DIAGNOSTIC ACCURACY OF BITEWING
RADIOGRAPHY FOR PROXIMAL CARIES DIAGNOSIS?
A. MODERATE SENSITIVITY AND LOW SPECIFICITY
B. MODERATE SENSITIVITY AND MODERATE SPECIFICITY
C. M ODERATE SENSITIVITY AND HIGH SPECIFICITY
D. HIGH SENSITIVITY AND MODERATE SPECIFICITY
E. HIGH SENSITIVITY AND HIGH SPECIFICITY
ANSWER:E

10-. YOU ARE INTERESTED IN FINDING OUT WHAT THE RISK INDICATORS ARE FOR A RARE FORM OF ORAL
CANCER AND DECIDE TO UNDERTAKE A STUDY TO EXAMINE THIS.
WHAT TYPE OF STUDY WOULD BE THE MOST APPROPRIATE FOR ADDRESSING THIS ISSUE?
A. COHORT
B. PREVALENCE STUDY
C. CLINICAL TRIAL
D. CASE-CONTROL STUDY
E. CASE-SERIES

Answer: A

11- Sex ratio for oral cancer is 2:3 or 1:2?

Answer: In England and Wales Male: female ratio of oral cancer is 2:1 and in Scotland is 3:1

12- What is mean mode n median

Mean: average

126
127

median is described as the numeric value separating the higher half of a sample, a
population, or a probability distribution, from the lower half.

Mode: the mode is the value that occurs most frequently in a data set or a probability
distribution

13-%GE OF CHILDREN SUFFERING FROM DENTAL TRAUMA IN PERMENANT TEETH


ANSWER:
8 YEAR OLDS- 5%

12 YEAR OLDS- 11%

15 YEAR OLDS- 13%

- A new filling material has been developed by the Dental School. After publication of
laboratory results, the researchers conducted a randomised clinical trial in general dental
practice where patients requiring one filling were randomly allocated either to old or new
filling material group. After five year follow up, the mean survival time between two
materials was compared, and P-value of 0.125 was reported.

Which answer is correct?

A. There are no important differences in the properties of the materials

B. There is no difference in restoration mean survival time between the two materials

C. The difference between two material is not clinically impotant

D. The difference between the materials is very small

E. The new material is useless

I think the answer is e

Restorative dentistry
1-what is the definition of freeway space and what is the range?

It is the difference between the rest and the intercuspal posticion(ICP) and it ranges from 3-
4mm(check that number)

2-what bur would you use for preparing the walls of the cavity?

Straight fissure bur

127
128

3-what bur would you use or the floor of the cavity?

Inverted cone bur

4-what is the most common fracture of the amalgam?

Isthmus fracture

5-minimum thickness of amalgam is?

2mm

6-what is group functioning occlusion ?

It is contact of the teeth on the working side during lateral excursion of the mandible while
no contact on the non working side

7-what is the main component of GP?


60-75% zinc oxide eugenol
20% of GP
1-4% wax/resin
Metal sulphate 1.5-1.7

8-what causes radio-opacity of GP?

Metal sulphate 1.5%-1.7%

9-what cement is used for veneers?

Resin composite luting cement.

The fitting surface of the veneer is washed with alcohol ,then saline coupling agent and
dentin adhesive system.the tooth is acid etched and washed carfully and dried application
of dentine adhesive system.on the fitting surface of the veneer a resin composite luting
cement is added and placed over the preparation any excess is removed of cement is
removed by bonding resin before curing

10-what is used as cement for GP? Zinc oxide eugenol

128
129

11-what is used to seal the apex? Calcium hydroxide

12-what cement is used or temporary bridge? Zinc oxide eugenol(temp bond)

13-what is used for the cement of porcelain crowns? Resin composite luting agent

14-what is the irrigation solution and its concentration ? Sodium hypochlorite 2.5%

15-chelating agent used to resolve sclerosis canal? EDTA

17-what is used for bleaching?

Carbamide peroxide concentration for in office 25-40%, home kit 10-15%

18-what is the ideal angle of the cavosurface of amalgam preparation?

110 is the ideal ,70-90 is minimum accepted

19-what is the length of the nayer core?

2-3mm

20-in obstructed canal what irrigation is used ? EDTA

21-most retention post? Parallel serrated

22-the most common cause of overhang of amalgam? Not using the wedges

23-most common problem of composite that you wont get with amalgam?

a-open contact

b-closed contact

d-overhang

129
130

answer is a

24-success of the post depends on ?

The length of the post 5-6 mm

25-what is walking bleach?

It is the internal bleaching of a non vital rct tooth by the application of carbamide peroxidise
35% intra orally and sealing the crown patient should be seen after a week or two.
Alternatively the patient should be given 10% carbamide peroxide that should be change
intracoronally 24 hours plus a splint to bleach externally too.

26-what is the concentration of hydrogen peroxide?

a-Over the counter

b-Used by dentist

a is 0.1% and b is 6%

27-what is the function of a butt joint? Retention

28-why is sodium peroxide not used anymore?

Because it is a concentration of 100%

30- what is the ph of dycal?

11 ph

31-material of choice for post of a canine composite core?

a-prefabricated

b-costum made

c-fiber optic reinforced post

answer is c

32-vertical fracture is most common with which post?

I think its threaded but not sure

130
131

33-function of pin in amalgam restoration?

Retention of restoration

34-the post retains ? The core

35-core retains ? The crown

36-which is prevented using a matrix band?

a-open contact

b-overhang

c-undercontact

answer is a

37-cementation of crown and bridge?

a-temporary by tempo bond ZOE

b-permenant by panavia or GIC

38- butt joint is found in which preparation?

a-amalgum

b-metal crown

c-porcelin jacket crwon

anser is c

39-best propariety of ni/ti?

a-shape memory

b-elasticity

c-corrosion

d-resistance

answer is b

131
132

40- the use of gate Glidden?

It is used in crown down technique in the preparation of orifice opening and in removal of
GP to prepare for post or retreatment of RCT

41-with what permenant restoration do you remove more tissue?

a-pfm

b-veneer

c-gold crown

answer is a

42-first line of treatment for a 5 year old child with a abcess on primary molar?

First line of treatment is drainage if possible through the tooth

If it is associated with flactation then incison and drainage plus antibiotic

43-cement that fail in the presence of saliva contamination?

GIC

44-what happenes if you use an old diamond bur?

Leads to excess heat generation

45- . A 50 year old male patient has a Class III jaw relationship with an anterior open bite. It
is planned to restore his lower right second molar, which has suffered tooth wear and
fracture, with an indirect restoration. This tooth has approximately 2mm of coronal height

What would be the most suitable approach to restore this tooth?


A. Provide an adhesively retained gold onlay
B. Provide a conventional full crown
C. Increase the vertical dimension and provide a full crown
D. Surgically crown lengthen and provide gold crown
E. Provide an adhesively retained ceramic onlay

Answer is a reference page 96 master volume 2/other say answer is d as ther eis not much
tissue left for adhesion

132
133

46- A patient presents with a history of a post-crown having fallen out. The post-crown was
originally placed fifteen years ago and had been successful up until four months ago since
when it has come out and been recemented four times. At recementation there was no
evidence of any caries.
The patient had been a regular attender and not needed any restorative treatment for the
last eight years.
Which of the following is the most likely cause for the failure of this crown?
A.The post was to narrow
B. The post was to short.
C. The root canal treatment was failing.
D. A vertical root fracture was present.
E. There were excessive occlusal loads on the tooth.

Answer is d

47-. Best treatment planning- to replace missing lateral incisor for a 15yr old boy, 26year old
with missing lateral, with good periodontium, bone levels, good oral hygiene, absence of
Dental Caries ?

15 year old ...resin bonded bridge

26 year old implant

48-. A patient on examination was found to have swollen gingiva around a crown that had
been present for several years. The papillae were particularly enlarged.
What is the most important feature of a crown that may be responsible for this?
A.Material of the Crown
B.The-occlusion
C.Proximal-Contour
D.Labial-Contour
E. Surface finish

Answer: A

Explanation: The keyword in the question is enlargement of the papilla. This enlargement is
the result of lack of space (under preparation) for the crown's material (MCC or GSC or Full
porcelain) and resulted in over contoured crown which in turn irritated the papilla and
invaded its space causing it to hypertrophy. Because the papilla are anatomically located
proximal (mesial or distal) to teeth, the best answer would then be proximal contour

133
134

48- A 55 year old female patient is missing her upper right second premolar and upper right
first molar and also is missing the upper left second molar. The upper right second molar is
functional and has an amalgam restoration (MOD and buccal wall) that requires replacing.
The patient has no functional or aesthetic concerns.
What would be the treatment of choice in this situation?
A. Provide an upper removable partial denture
B. Replace the amalgam in the upper right 7 only
C. Provide a full coverage crown in the upper right 7
D. Provide a fixed bridge in the upper right quadrant
E. Provide a full coverage crown in the upper right 7 with guide planes and occlusal rests.

Answer: B

49-. A 23 year old male presents to your surgery. He lost his upper lateral incisors some 10
years ago in a swimming pool accident. Since then he has been wearing a `spoon' denture
which he now feels in aesthetically unacceptable. He has sought an opinion on dental
implants but has been told that he would need bone grafting for this to be successful and he
is not prepared to undergo this

His dentition is excellent with no restorations and a Class I occlusion. He wants some advice
on what the best treatment might be.
Which option would you put first on your list of possibilities?
A. Two fixed resin bonded bridges using the central and canine teeth
B. Two cantilever resin bonded bridges from the central incisors and canines.
C. Two conventional fixed bridges from the canine
D. Conventional cantilever bridges from the canines
E. Cobalt chrome partial denture

Answer: B

49-. A 43year old patient is missing on the upper right first premolar and molar. He has good
oral hygiene and requests a fixed replacement for these teeth. The other teeth on the same
side are all moderately restored with MOD amalgam restorations and are vital, except the
canine, which has a very large restoration and is root-filled. He has group function.
Radiographs show a large sinus cavity and no peri-apical pathology.

What would be the restoration of choice for replacement of the missing teeth?
A. Implant supported crowns
B. A conventional fixed bridge using the 7 and 5 as abutments
C. Two conventional cantilevered bridges, using the 7 and 3 as abutments
D. A resin-bonded bridge, using the 7 and 5 as abutments
E. A conventional fixed-moveable bridge using the 7 and 5 as abutments

Answer is e

134
135

50- A 35 year old male patient who admits to grinding his teeth at night has a number of
wedge-shaped cervical (Class V) lesions on his upper premolar teeth. These are causing
some sensitivity and are approximately 3mm deep.

What is the correct management option?


A. Provide tooth brushing instruction and fluoride
B. Restore the lesions with compomer
C. Restore the lesions with micro-filled composite
D. Restore the lesions with a hybrid composite

 Answer: compomer or mircofill composite

51- what is the bleaching material that is not used for more than 6%?

Hydrogen peroxide

52- why is sodium peroxide is not used anymore ?

Because its concentration is 100%

53- what is the material of choice for restoration for syjorgen syndrome?

Resin modified GI

54- Which sealant is more likely to cause tissue damage if extruded

a-AH pluse

b- grossman's sealer

c-tubliseal.

Answer is a

55-. What is the main function of EDTA?

Answer: Is a chelating agent, Negotiate sclerosed canals, dissolve inorganic component of


smear layer which sodium hypochlorite can’t do.

Sodium hypochlorite: dissolve organic debris and it’s bactericidal, 2.5% conc available
chlorine

56-. A cold stimulus applied to a tooth will produce a hypersensitive response if the tooth
A.is nonvital.
B.has a periodontal pocket.
C.has a hyperemic pulp.
D.has chronic proliferative pulpitis.
Answer: C

57- What is least important for success of post?


A. diameter,

B. material,

135
136

C. luting agent,

D. length

Answer: C

58-. Pulp tester, its readings and what they signify

Answer: Exaggerated response to pulp testing is seen in reversible pulpitis and no response
or a reduced response is seen in irreversible pulpitis.

59-The best approach for diagnosis of odontogenic pain is which of the following?
a.Radiographic-examination
b.Percussion
c.Visual-examination
d.A step-by-step, sequenced examination and testing approach
Answer: D

60-. What is least important for success of post?


A.diameter,
B.material,
C.luting-agent,
D.length

Answer: C

61- What is the BULL rule??

Answer: BULL rule stands for Buccal Upper and Lower Lingual cusp to preserve centric stops
which are normally in occlusion.

62- The pulp horn most likely to be exposed in the preparation of large cavity in a
permanent tooth is:

A.Mesio-lingualin upper firstMolars


B.Mesio-Buccal in upper first molars
C. Disto- Buccal in lower first molars

d-mesio-lingual in the lower first molar

e-mesio-buccal in the lower first molar

answer is e

136
137

63-The colour of the tooth is due to:


A)Hue
B)Chroma
C)value
D)Translucency

A ,,,,,Hue- basic colour depending on the wavelength, chroma-measures the intensity of the
colour, value is darkness or lightness of the colour -

64-How many surfaces does a Pontic have?


A)three
B)five
C)seven
D)four

Answer is b

65- While making a crown for erosion of tooth substance, ideal choice of crown is:
A)porcelain-crown
B)metal-crown
C)PFM-crown
D)Acrylic crown

Answer is b

66- The retention pin in an amalgam restoration should be placed :


A. Parallel to the outer wall
B. Parallel to the long axis of tooth

Answer is a

67-tooth surface loss:

1-V shape loss result from a repeated cyclical loading and unloading resulting in cervical
tissue loss

2-during access for endodontic or following debonding of endodontic brackets

3-charactirised by sensitivity when actively occurring and the area appears shiny and
smooth, amalgam restoration may be noted to sit higher of the remaining tooth

4-results from direct fall

5-demineralization at the lingual pits of the anterior teeth

6-v shaped cut on premolar cervical margin from mechanical force.

Options:

a-abrasion

137
138

b-abfraction

c-attrition

d-caries

e-erossion

f-iatrogenic

g-traumatic

answers:1-b,2-f,3-e,4-g,5-d,6-a

68-Which of the following instruments can be


used for placing gingival bevels on inlay
preparations?
1.Margin-trimmers.
2. Enamel hatchets

3-carbide finishing bur

4-small diamond disc

Answer is marginal trimmer and carbide finishing bur

69-what bur is used for the removal of enamel, dentine and amalgam?

Round diamond bur is used for removal of enamel

Tungsten carbide or diamond for removal of dentin and amalgam

70-when removing dentin with a bur what is the cutting side?

a-tip of bur

b-sides of bur

answer is b

71-Which is the best material for treatment of non-carious Class V cavities in unstable acid
erosion cases?
a-GIC
b-RM-GIC
c-Compomer
d- Composite

138
139

Answer is b
72-Which of these is a compomer?
a-Herculite
b-Gluma
c-FujiIX
d- Dyract ...
73-Which of these lining materials should not be used with composite?
a-Life
b-Poly-F
c-Vitrebond
d-Kalzinol ...

Answer is d

74-A radiolucency near the apex of tooth #28 is seen radiographically. The tooth is
asymptomatic and does not have caries or periodontal problems. Which is most likely the
cause of the radiolucency?
A Submandibular fossa.
B Periapical granuloma.
C Complex compound odontoma

D-Mental foramen

Answer is d

75- Which of these is NOT true about Gates Glidden drills?


a- If they fracture, they are easier to retrieve than Rotary NiTi files
b-They can be 'swept away' from danger areas like furcations, unlike most rotary NiTi's
c-They leave a smooth step-free surface on the canal wall
d-They are cheaper than NiTi's

Answer is c

76- what is the Patency filling?-

It is a canal preparation were the use of small flexible files(less than size 10)0.5-1mm
through the apex. Advantages are to prevent apical blockage by the building up of the
debris w/o enlarging apical foramen the disadvantes if file is push through the apex further
than 1 mm will lead to extrude infected debris and my cause flare up are longer
appointments and inability to control exudates

77- Shape of access cavity in maxillary first molar: thru occlusal surface,triangular shaped
with base buccally and apex palatally

mand molars :triangular shaped.base mesially and apex distsally or trapezoid in shape

max and mand canines and incisors: close to incisal edge on a palatal or lingual
surface.triangular with broadest portion incisally.

139
140

max mand premolars : ovoid buccolingually

78- NiTi instruments are preferred to stainless steel instruments because of hyperflexibility

79- What’s “greater tape” instruments made from?


NiTi

80- How can u locate the apex other than a radiograph?


Electronic apex locator

81-how to remove a fractured instrument?


By pair of fine mosquitos ,ultrasonic tip or Masseran kit(used for removal of post but these
weeken the tooth)

82-what are the greater tapers?


They are files that are made from NiTi when motor is not required or where greater manual
control is required the flutes are in the reverse so used with reverse balanced force method

83- Restoration to be given if marginal ridge is to be protected,contact point is to be


retained?
I think amnswer is amalgam

84- what sort of matrix best for restoring a disto-occlusal restoration of 7?


a-sectional-matrix
b-automatrix
c-tofflemier
Answe is a

85- Restoration of lingual cavity in 65 years old class V?


GIC

86- Periapical with approximal caries in incisor and molar, what is the best access to the
cavity? Direct access. Oclussal access etc.
Occlusal via marginal ridge-most commonly used-aim to form scoop form of cavity using
pear shaped bur
Direct access-when adjacent tooth is missing
Occlusly leaving marginal ridge intact-tunnel prep
Buccally/lingually-when teeth are tilted

87- How to identify caries of enamel?


Probe,
dry tooth,
wet tooth,
transillumination.
Answer is dry tooth
Same question repeated several times with different types of caries so make
a list of the diagnosis aids for different types of caries

140
141

88- Patient with Bulimia - treatment


- Patient with anorexia nervosa treatment option

89- When choose amalgam than composite resin?


posterior extensive cavity

90- Lingual cavity in posterior teeth in Parkinson patient, which material is better to use?
– Glass ionomer?i don’t think it is write because parkisnon diease difficult to control saliva
secretion

91- How do you proceed to eliminate approximal caries in anterior teeth?


Access from palatal aspect.

92- Cavity test ( drilling) , when do you use it ?–


When all other tests have failed used to check crowns

93- - Which root filling material comes in 2 pastes?


Tubli Seal,which is a zinc oide eugenol

94-how to diagnose a Cracked tooth syndrome?


use cotton wool, pain on release of biting

95- A pt has an old Ag filling and there is occlusal wear what are the causes bruxism ,
attrition, abrasion ?
.attrition

96- Best protection for the pulp ?…..


Dentine

97-best method for the diagnosis of the following ?


a-caries half way though enamel on proximal surface
b-caries half way through enamel on occlusal surface
c-caries half way through dentin no cavitation
d-stained fissure
e-hidden occulant caries
options are trasllumination,bite wing,dry surface of the tooth ,wet surface of the tooth
,briaullt probe ,willams probe, straight sharp probe
answers are a-briualt probe
b-dry tooth
c-wet tooth
d-bitwing am not sure
e-bitewing

What are the four zones in enamel caries in order from the advancing edge?

141
142

Translusent zone,dark zone ,establised lesion and suface,,,,,translucent zone is closest to the
dentine

what are the burs needed for the following ?

1-rest seat on occlusal

2-to remove enamel

3-to remove dentine

4-for walls of cavity

5-for floor of cavity

6-slicing the interproximal contact in rampant caries

7-rest on the cingulum

8-bone removal

9-tooth separation in surgery

10-polishing composite

...5-for floor of cavity6-slicing the interproximal contact in rampant caries7-rest on the


cingulum8-bone removal9-tooth separation in surgery10-polishing composite

Prosthetic
1- Which measurement is taken with a single record block in the mouth??

Answer: Resting Vertical Dimension: taken with only the lower denture. Willis gauge used
for measurement.

2- Uses of face bow?

Answer: a device used in dentistry to record the positional relations of the maxillary arch to
the temporomandibular joints and to orient dental casts in this same relationship to the
opening axis of the articulator.

142
143

Or

Used to transfer the relationship of Maxilla to the intercondylar axis of the patient to the
Articulator and the hinge axis of articulator.

3-.what is the Cantilever Bridge indicated for?


answer
Minimal cantilever bridge/ RBB indications:
CLass 1 malocclusion
Class 2 div 1 malocclusion
Anterior teeth
Short span .
Given only upto second premolar
Sinlge tooth missing
Suitable for Anterior teeth only which is resin bonded to its retaniner which is lacated far
away.A Cantilever Bridge is a fixed bridge that is attached to adjacent teeth on one end only.
They are used in cases where a bridge can only be anchored to a tooth on one side of the
gap. Thus, Cantilever Bridges are best suited to areas of your mouth that are less prone to
stress, such as the front teeth. This dental procedure involves anchoring the false tooth to
one side over one or more natural and adjacent teeth. As a result, they are best suited and
ideal for situations where a traditional dental bridge or a dental implant isn’t an option.

4-what is the retention component ?

143
144

Answer it is RPI increases retention of a free-end saddle. this includes the following
component:

Mesial occlusal REST

A proximal guided PLATE

A gingivally approaching I bar

5- a man aged 40 years old with an extracted upper left incisor and will be having an implant
within the next 6 month. At the mean time patient is concerened about aesthetic .what is
the treatment of time being until he receives his treatment?

Immediate acrylic denture

6- how to measure the vertical dimention of occlusion?

Vertical dimension at occlussion=vertical dimension at rest -free way space

7-what is the range of free way space?

It is the differences between the intescuspal occlusion and rest occlusion which is btween 2-
4mm

8-what is the requirement of the lingual bar?

It needs an area more than 7 mm from the floor of the mouth to the gingival margin to give
a 3mm clearance from th gingiva. And the bar needs to be 3mm in thickness

9-what connector is used with a toris palatine?

Horse shoe connector

144
145

10-what is your reference point for placing the maxillary anterior teeth when for the trial
denture?

The anterior teeth should be of 8-10 mm anterior to the posterior of the inscive papella

11-what is the cause of gaseous porosity ,contraction porosity,granular porosity?

Gaseous porosity:is due to the excessive boiling dough in the flask above 100 degree which
is above the boiling point of the monomer leading to shephrical voids in the hottest part of
the dough

Contract porosity :is due to insufficient dough that has been placed to for a flash or due to in
adequate pressure leading to formation of void during curing

Granular porosity:results from the evaporation of the monomer when preparing the dough

12- what is willis gate??

It is a metal ruler used in prosthetic to measure the occlusal vertical dimension from the
base of the nose to the under chin.

13-when manfuracturing an upper and lower denture what is the only thing that you can
not change?

a-Condyler angle

b-canine angle

c-incisive angle

answer is a

14-when taking a secondary impression with a special tray what is the space required for
the following impression materials?

1-ZOE

2-alginate

3-elastomer impression

Options are 0.2,0.5,1, 1.5,2,2.5,3,3.5

Answer is zoe 0.5,alginate 3,elastomer 1.5

15-what is a dental survayor and what are it component?

145
146

It is a dental instrument used to determine the path of insertion ,undercut that are used for
retention and those that need to be blocked in the dental arch.widely used is jelenko and
Ney.

The components :

a-analysing rod:Used in primary analysis it is a metal rod used to determine the undercut
areas and determine the parallelism without marking the cast

b-graphite rod or carbon rod; it is used to draw the surveyor line on the cast to identify the
maximum convexity

146
147

c-undercut gauge ;it is used to identify the position of the desired undercut

d-the trimming knife; this is an instrument to eliminate the unwanted under on the master
cast

16- what is the tripod method?

Using the tripod method, the vertical arm of the surveyor is locked at a
height that allows the tip of the marker to contact the palatal surface of
the ridge in the molar and incisal regions. Three points are marked with
the graphite marker, one on each side posteriorly and one anteriorly. The
points will then be ringed with a pencil so that they are clearly visible.
Alternatively, the analyzing rod is placed against one side of the base of
the cast and a line drawn on the cast parallel to the rod. This is repeated
on the other side and at the back of the cast so that there are three

147
148

widely spaced lines parallel to the path of insertion. And this is used to
determine the horizontal plane of the denture .

17- what are the pontics and its types?

Sanitary pontic(hygienic pontic ); it has no contact with the ridge so it


allows easy cleaning but unaesthetic . Used for posterior teeth .

Modified ridge lap: has minimal contact with the ridge buccally provides
good aesthetic and easy to floss. Most popular type of pontic usually used
for anterior teeth.

Bullet :tip contact with the ridge used on the posterior teeth

Ridge lap pontic:it extends buccally and lingually ,difficult to clean ,less
desirable type

Conical :small convex are of contact at the tip of the centre of the ridge
,easy to clean used for posterior teeth

Ovate pontic:used for the anterior maxillary teeth has a greater mucosal
contact and applies light pressure to the underlying mucosa . Patient
should have excellent oral hygiene.

18- what is the first step after taking an impression?

Wash the impression then disinfect it for 10 minutes need to check with
10000 ppm of sodium hypochlorite

19-what bur is used or preparing a rest seat?

Round bur tungiestin carbide

20-what posterior tooth is chosen for the rest?

Premolars ..not sure check answer

21-an impression material that can not be used for a secondary


impression?

Alginate

22-most common cause of failure in Maryland bridges?

Retention failure(debonding) is the most common problem

148
149

23-An old man with loose lower denture comes to u, you are making every yr a new one
how will you deal with the patient?

a-will u make a new denture,

b-suggest implants,

c-wait for 3 months,

d-refer to specialist,

e-alveoloplasty
Answer: refer to specialist

24- what is confirmative approach?

this approach relies on detailed examination and recording of the static and dynamic
occlusion before we start any work in the tooth

25- what is reorganized approach?

re- organised approach involves changing the existing anterior guidance and intercuspal
position to new , less harmful guidance pattern

26- For optimum esthetics when setting maxillary denture teeth, the incisal edges of the
maxillary incisors should follow the ____.
A Lower lips during smiling
B Upper lips during smiling
C Lower lips when relaxed
D Upper lips when relaxed

Answer is A. Maxillary teeth should contact the wet dry lip line when fricative sounds f, u,
and ph are made. These sounds help to determine the position of the incisal edges of the
maxillary anterior teeth.

27-In a Kennedy Class I arch in which all molars and the first premolar are missing and the
rest of the teeth have good periodontal support, the preferred choice of treatment is ____.
A- A removable partial denture replacing all missing teeth
B- A fixed dental prosthesis replacing the missing premolar and a removable partial denture
replacing the molars
C- Implant supported crowns replacing the first premolars and a removable partial denture
replacing the molars
D- A and B are preferred over c
E -B and C are preferred choice of treatment over A

Answer is a

149
150

28-.which is the minimum distance between the major connector on a maxillary RPD and
gingival margins?
A)1mm
B)6mm
C)12mm
D) 4 mm

Answer is b

29-when to construct a copy denture?

When patient is satisfied with his old one

30-what is the defference between a spoon denture and an every denture?

Spoon denture :it is a small denture used to replace one or two anterior teeth.it has no
clasps for retention and depends on the saliva sunction and patients learn a technique to
use the tongue to hold denture in space.

Every denture it has clasp for better retention and involves the interdental arch for
retention used only for upper jaw.it is a mucosa borne with a special design to ensure
gingival health

31- A patient presents for their first appointment with you. He has complete dentures. He
has worn upper dentures for 30 years and the lower complete for just 3. Prior to this he had
the lower premolar – premolar teeth and did not use the lower denture supplied to him.
Unusually the patient is happy with the lower denture but the upper is causing considerable
problems, especially with regard to retention. After your examination you determine that
the patient has a flabby anterior ridge and this seems to be the source of many of the
problems.

How will you deal with this problem?

A. Surgical excision of the flabby portion of the ridge using an atraumatic technique

B. A selective pressure impression at the secondary impression stage

C. A highly muco-compressive impression to flatten the ridge and improve retention

D. Cut a window in the finished denture to allow the ridge to protrude through unhindered

E. A highly muco-static impression at the primary impression stage to allow the ridge to be
recorded at rest, followed by a normal impression at the secondary stage

I think answer is b

32- You are designing a partial denture for a patient with several missing teeth in the maxilla.

The reason for surveying the model prior to designing the denture is to:
A. Measure and mark out hard and soft tissue undercuts on the casts

150
151

B. Relate the intended position of the inter-papillary plane of the patient to the casts
C. Establish the position of the post dam
D. Relate the maxillary and mandibular casts
E. Aid setting up the prosthetic teeth prior to trial insertion
Answer is A

33- Complete denture articulation should incorporate certain features.Which of the


following is a required feature?
A. Incisal guidance
B. Canine guidance
C. Balancing side contacts
D. Unilateral contacts on excursive movements
E. Posterior disclussion (open bite) on lateral excursion
I think answer is c

34- You are working as a general practitioner and you have referred a 13 year old for an
orthodontic opinion. Your orthodontic colleague has recommended the extraction of the
upper second premolars. You undertake these under local anesthesia without incident. A
week later the child is booked in as an emergency. He has fallen off his bike and the
maxillofacial SHO at the local hospital was forced to extract the upper incisors as they so
badly damaged. Orthodontic treatment has been abandoned and you are now considering a
referral to the restorative department of the dental school for an opinion. You need to write
a referral letter to the restorative Consultant.
Other than those described, the rest of the dentition is intact. What Kennedy classification
will you use to describe the child’s tooth loss?
A. Class I mod 1
B. Class II
C. Class III mod 2
D. Class IV
E. Class III mod 1
Answer is c

35- A 65 year old man presents for a denture review. He complains that his upper denture
causes him some discomfort occasionally but otherwise he has no complaints. Upon removal
of the denture you notice a thick plaque covering the palatal mucosa. When brushed with a
cotton wool roll, it can be removed, and the underlying mucosa is red and areas of it bleed.
Upon questioning the patient you find that he rarely removes his denture. The denture itself is
stained and has heavy calculus deposits on it. The patient’s medical history is clear.
How will you deal with this problem?
A. Swab the mucosa for microbiology, prescribe an anti-fungal cream and advise on denture
hygiene initially
B. Arrange for an incisional biopsy as an emergency appointment at the local DH
C. Arrange for a denture reline to be conducted
D. Prescribe a broad spectrum antibiotic and review in 7 days with a view to replacing the
denture
E. Do a chair-side reline using a tissue conditioner and advise on denture hygiene
Answer is A

151
152

36-Picture of an upper denture with Adam’s cribs on the 6’s what kind of denture is it?
a. Spoon denture
b. Every’s denture
c. Tooth supported denture(correct)

-A Spoon Denture : is a small denture usually to replace just one or two front teeth. The
palate part of the denture on suction to hold it in place as it does not make contact with the
inner surfaces of the back or side teeth. This means that it tends to be unstable and requires
skill on the part of the patient to use their tongue to stabilise it while eating. This lack of
stability is the main disadvantage and the subsequent movement can lead to gum recession
and further loosening. The advantages are that it is cheap and easy to make and as the gum
margins of the other teeth are not contacted by the denture base, there is less likelihood of
decay or gum disease occurring.

Every Denture: is a mucosa borne denture with a specific design to ensure gingival
health.Restricted to the use in upper arch.

37- management of missing incisors in adults and children

38-bridge parts and cr/co denture. kennedy class.

39-type of bridges and indications

40- Partial denture design 5 sub questions – abutment (mesial rest, distal rest),
increase/decrease occlusal plate, increase/decrease support on mucosa area (saddle),
stress-breaking design

41- Questions on Kennedy’s classification of dentures

42- A picture of a man whose central incisor has just been extracted and who is going to
get an implant in the next 6 months what is the best way to preserve the space
a. Chromium cobalt denture
b. Acrylic denture-
c. Orthodontic wire
answer is b

43- What is the length of a clasp?


A retentive clasp should be at least 15 mm in length if it is constructed in cast cobalt-
chromium alloy
A retentive clasp should be at least 7 mm in length if it is constructed in wrought wire.

44- Problems caused by palatine torus?


Anterior posterior palatal strap

152
153

45- Die fail to fit preparation and cast ?


die damage + distorted pg 244 pink book old version

46- Inlay fail to fit to prepare ?


Presence of undercuts

47- Picture of two casts with wax blocks having teeth in occlusion they asked what can u
identify from the picture options were. bite registration,there is class 2 or 3,protrusive
record etc. Have to see the picture to answer this one

48- setting of teeth in relation to ridge and papilla:


-Natural teeth lie 10mm from papilla ; with resorption this comes to lie on ridge so the
anterior teeth should be placed labial and buccal to the ridge to give adequate lip support
and naso-labial angle of 90 degrees.
-Normally 8-10 mm infront of the centre of incisive papilla.Incisal edges of upper central
incisors,canines,both cusps of 1st and 2nd premolars and mesiopalatal cusp of the 1st max
molar should touch occlusal plane.

49– what you can check outside patients mouth in articulator ? read page 114 master
dentistry
On articulator before trial on the patient,we can check complete teeth set up.The
occlusion is then assessed,checking balance in excursive movements.extension of the
denture base

50– Willis bite gauge and surveyor pictures shown and you need to say their names

51- When making Dentures, what you cannot change?


condylar horizontal plane because this the physiological position for the mandible so it
can’t be changed.

52- – Fixed- movable bridge ? read page 268 pink book


A fixed –movable bridge :the pontic is anchored rigidly to the major retainer at one end of
the span and via a movable joint to the minor retainer at the other end the major
abudment provides retention and support whilst the minor abudment provide support
only.this design provide some independent movement of the minor abudment and has
the advantage that the preparation does not need to be parallel

53- - When will you use a butt joint in a crown preparation?

153
154

When an increased thickness of material is required which in turn is required for


increased strength such as PJC

54- Lower premolar is set on the alveolar ridge – yes


Upper anteriors are set labially to the ridge
Upper posteriors are set slightly buccal to the residual ridge, parallel to ala tragus
Lower Anteriors : when little resoption teeth placed marginally in front of the ridge crest ,
cases with excessive resoption teeth placed over buccal sulcus.
Lower Posterior : teeth directly on the ridge

55- Restoration for peg lateral?


Composit build up or porcelain veneer to increase the mesio distal width.

56- What can be adjusted on a cast before the patient comes?


Anatomical tooth position.balance and excursive movements on an articulator before trial

57- Trauma by denture in buccal sulcus?


teeth should be set in neutral zone and over extensive should be trimmed

58- - Group function, canine-guided occlusion :


Group function : multiple tooth contacts on working side during lateral excursions but no
contact on non working side.
Canine guided occ: During lateral excursions there is disculsion of all teeth on the
working side except for canine and no contact on non working side.

59- Bur to prepare “rest” in prosthesis – (reference-nallaswamy textbook chapter 18)


for occlusal rest--no 4  tungsten carbide bur
for cingulum rest -large diamond cylinder bur

60- What is an Akers clasp :


a classic clasp with An Akers' clasp is the classic direct retainer for removable partial
dentures. Named after its inventor, Polk E. Akers, this suprabulge clasp consists of a rest, a
guide plate, a retentive arm and a reciprocal arm. Akers' clasps, as a rule, face away from
an edentulous area. Should they face the edentulous area, they are termed reverse Akers'
clasps
SIMPLE CIRCLET DESIGN (Aker’s clasp)
Widely used
Tooth support RPD
Engage undercut remote from edentulous area
Half round cross sectional
Disadvantages
- Increase circumference clinical crown
- Increase tooth coverage
REVERSE AKER DESIGN
Undercut located adjacent to edentulous area
Infrabulge clasp is contraindicated
Kennedy class I ,II

154
155

Disadvantage
- Reduced strength
- Lack of rest adjacent to edentulous area
- Poor esthetic
MULTIPLE CIRCLET DESIGN
2 simple circlet clasp joined at the terminal aspect of their reciprocal
elements
Principle abutment is periodontal compromised
Disadvantage

61-a 70yr old woman with missing central incisor as a result of periodontal problems. Give
partial denture or resin bonded bridges?
Resin bonded bridge

62- 2 different clinical cases of need to be extracted upper central incisor and the best
way to replace it – options: acrylic dentures, chrome-cobalt partial denture, implant,
adhesive bridge, fixed-fixed bridge
a- for a 45years courier with other lost upper molars and PMs – acrylic denture
b- other quite well-financial situated guy, for immediate replacement – socket
implants.

63- Complete Denture Phonetics


In a full denture case If the patient is having trouble saying F or V, then the incisors have
been placed too far palataly. Or if the patients S sounds more Th, then it could be due to
palatal positioning of incisors or thicker palate. Pg 309 pink book.

Mechanism of speech:
The voice is produced in the larynx and the muscles ,the muscles of the thoraxic and
abdomen control the flow of the air and the nasal cavity works as a resonant chamber
The air from the larynx is divided into two streams by the velum:
1-the upper stream through the nasal cavity to produce the nasal sounds N ,M ,NG
2-the lower stream and altered by the palate ,tongue and teeth which produce different
sounds

Type of sounds:
1-vowel sounds which are produced by the vibration of the vocal cords only and not
affected by any structure in the oral cavity,the tongue is placed in the floor of the mouth in
contact with the lingual aspect of the teeth. A E O I U
2-constant sounds they are produced byconstriction ,obstruction and direction of air stream
when pass through the mouth.

*According to the manner of production


a)nasal sounds (n,m,ng)
b)explosive sound (p,b,t,b,k)produced by complete stop of the air and as pressure build
there is a suddon release of air making the sound
c)silbant sound or fricative (sh,x,ch,z) produced by a friction when the air is forced through a
narrow path way

155
156

*according to the place of production:


a)bilabial (m n b):by the lips meeting only the air from the lungs build up behind the lips and
produced by explosion
b)labiodent (f v):is produced by the lip and the teeth where the upper teeth in contact with
the lower lip
c)lingodental (th):is produced when the tip of the tongue is incontact between the upper
and lower incisor teeth
d)lingoalveolar(s t d) where the tongue is in the anterior postion of the palate

Orthodontics
1-arrange the following into there proper sequence for molar uprighting of tooth requiring
both restorative in periodontal treatment?
a-band
b.complete restorative treatment
c-complete periodontal treatment
d-separate
e-upright
answer is BCDAE

2-The term over jet refer to:


a) Horizontal relationship between the maxillary and mandibular incisors (about 2 mm).
b) Vertical relationship between the maxillary and mandibular incisors (about 3-4 mm).
c) Horizontal relationship between the maxillary and mandibular incisors (about 8.5 mm).
d) Vertical overlap labially of the maxillary incisors and mandibular incisors (normal range 0-
33%)

Answer is a

3-The 1st permanent molars should not be extracted if:


a)The 2nd molar in bucco-version
.b)If the molars are beyond repair.
c)The third molar are absent.
d) All the above

Answer is c

4-which type of malocclusion is associated with mouth breathing?

a-dental open bite

b-skeletal open bite

c-skeletal cross bite

d-dental cross bite

answer is b

156
157

5-1.which of the foll is the normal relationship of primary molars in deciduous dentition?
a.distal-step
b.flush terminal plane
c.mesial-step
d.none of the above

Answer is flush terminal plane

6-in predicting the time of puberty sprout while treating relationship malocclusion of the
jaw in a growing child how the orthodontist to get the most value information?

a-wrist hand radiograph

b-height and weight table

c-dental developmental age

answer is b

7-containdication of serial extraction?

a-class1 skeletal pattern

b-deep bite

c-several crowding

d-mixed dentition

answer is b

8-facial growth of a five year old child is completed by?

a-Cranium 96% and 45% for both maxilla and mandibule

b-cranium 85% and 4% maxilla and 65%mandible

c-cranium85% and 45% for both maxilla and mandible

d-cranium 4%, maxilla 85% and mandible 45%

e-cranium 85% maxilla 65% and mandible 65%

157
158

9-- Multiple supernumerary and unerupted teeth are characteristic feature of:
a.Treacher collins syndrome
b. Ectodermal dysplasia

c. clendocranial dysplasia

answer is c

10- Which of the following statement is correct regarding the submerge primary molar:
a.the tooth is non-vital
b. the tooth is ankylosed
c-there is no permenanat successor
d-there is no evident of root resorption

answer is b

11-which of the following are important in measuring the maxillary mandibular plane?

1-sella,menton,gonoin,ANS

2-the gonoin,menton,ANS and PNS

158
159

3-poginion menton ANS and PNS

Answer is 2

12-when classifying a cleft lip and palate the according to Veaua classification of vaue 2
means?

a-soft palate

b-soft and hard palate

c-soft and hard palate and the alveolus bone on one side

d-soft and hard palate the alveolus bone on both sides

answer is b

13- the most common etiology of non-skeletal anterior cross bite is:
a. delayed eruption of permanent teeth
b. over retention of primary incisors
c. lack of space for the permanent teeth
d. none of above

Answer is b

14- Which works best to prevent incisor tipping from thumb-sucking?


a-A removable appliance with a flat anterior bite plane
b- A removable appliance with a wire “gate” palatally
c-Intermaxillary fixation
d- Advice and persuasion on stopping the habit

Answer is d

15- 1. PREMATURE EXFOLIATION OF PRIMARY CANINE MAY INDICATE:


A . AN ARCH LENGTH DEFICIENCY
B. AN ARCH LENGTH EXCESS
C. SKELETAL MALOCCLUSION
D . ALL OF THE ABOVE
ANSWER IS A

16- EFFECTS OF THUMB SUCKING :


A. BILATERAL POSTERIOR OPEN BITE .
B. REDUCED OVER BITE
ANSWER IS B

17- Which grade of IOTN IS THE cleft lip and palate ?

159
160

Answer is 5P

18- What age has to be considered the most in diagnosing malocclusion?


Chronological age of the patient
Developmental age of the patient

Answer is developmental age

19- The deciduous canine-associated gap is the


a-Eruption follicle
b-Leeway space
c-Primate space
d-Canine groove

Answer is c

20-the "ugly duckling" stage in mixed dentition is characterized by:


a. increase overjet and deep overbite
b. distoangular axial inclination of both maxillary central and lateral incisors, with median
diastema
c. transposition of maxillary central and lateral incisors
d. retroinclination of maxillary central and proclination of lateral incisor

answer is b

21- class II div.2 malocclusion is always associated with:


a.open-bite
b.cross-bite
c.scissor-bite
d. deep bite

Answer is d

22- cephalometric is useful in assessing which of the following relationship:


a.tooth to tooth
b.bone to bone
c.tooth to bone
d. all above

answer is d

23- The term incompetent lips refers to:


a) Lip which do not seal when relaxed with the mandible in its rest position.
b) Lips against which tongue thrusts during swallowing.

160
161

c) Lips that contract during swallowing.


d) Lips which have some dryness.

Answer is a

24-what is the definition of anchorage?

a-the resistant to unwanted tooth movement

b-use of intermaxillary elastics

c-resistance to treating a malocclusion

d-resistance to favoured tooth movement

e-is the use of extraoral traction

answer is a

25-which of the following are the sequence of serial extraction method?

a-deciduous canine, first premolar, first deciduous molar

b-deciduous canine, first deciduous molar and first permanent molar

c-deciduous canine ,first deciduous molar and first premolar

d-deciduous canine firat deciduous molar and second deciduous molar

answer is c

26- Cephalometrics was introduced by


a-Pacini
b-william conrad roentgen
c-Broadbent and Howarth
d-Leonard s Fishman

Answer c

27- A deciduous tooth is displacing its permanent successor. What should you do?
a- Leave it to exfoliate naturally
b- Extract it and fit a space maintainer
c- Extract it.
d- It depends on which tooth

Answer is c

28-This child has an abnormally shaped tooth in the upper left incisor region.
1-What investigation would you carry out?

2-What treatment may be necessary?

161
162

answer is to take radiograph to assess the presence and location of the permanent incisor
and to ascertain the supernumerary tooth

treatment is extraction of the supernumerary tooth

29-LOSS OF WHICH DECIDUOUS TOOTH CAN CAUSE THE MOST PROBLEMS REGARDING FUTURE
CROWDING ?
A -C ANINE
B-LOWER FIRST MOLAR
C-U PPER FIRST MOLAR
D - LOWER SECOND MOLAR
E -THE UPPER SECOND MOLAR

Answer is e

30- Signs of anchorage loss includes all of the following except


a-mesial movements of molars
b-decrease overjet
c-change in molar relation
d-spacing of teeth

Answer is b signs of loss anchorage is increase in the overjet

31-

162
163

32-what is that active component that moves the tooth in a mesio-distally direction?

a-palatal finger spring

b-t spring

c-z spring

d-Robert retractor

e-labial bow

answer is a

33-

163
164

Answer is the osteoblast resorbing the bone

33-

164
165

Answer is b

34-

165
166

Answer is e

35-

166
167

Answer is a majority of dental effect and a minority of skeletal effect.the functional


appliances are used to treat the cl2 malocclusion,usually treatment is by proclination of
the lower incisors ,retroclination of the upper incisors,mesial drifting o the lower molars
and distal tipping of the upper molars.it also has a minor skeletal effect in restraining the
development of maxilla and promoting the growth of mandible.

167
168

36-

Answer is e, the posterior bite plane discludes the occlusion anteriorly allowing the t-spring
to move the in-standing incisors labially. Anterior bite plane will disclude the occlusion
posteriorly and the screw is to correct the cross bite posteriorly.

37-A radiolucency near the apex of tooth #28 is seen radiographically. The tooth is
asymptomatic and does not have caries or periodontal problems. Which is most likely the
cause of the radiolucency?
ASubmandibular fossa.
BPeriapical granuloma.
C Complex compound odontoma.
d-mental formane
answer is d

38- Which of the following is the most likely cause of pulpal necrosis following trauma to a
tooth?
A]Ankylosis
B]Calcific-metamorphosis
C]Pulpal-hyperemia
D]Dilaceration

Answer is c

39- IF A PATIENT LOSES A MAXILLARY FIRST PERMANENT MOLAR BEFORE THE AGE 10, THE:
A. PREMOLAR DRIFTS DISTALLY

168
169

B. MAXILLARY SECOND MOLAR ERUPTS AND MOVES FORWARD


C. OPPOSING TOOTH ERUPTS INTO THE SPACE CREATED
D. OVERBITE INCREASES
I think the answer is c

40-RELAPSE IN ROTATED TEETH CAN BE AVOIDED BY:


A )B Y CSF
B)B Y FRENECTOMY
C)B Y EXTRACTION OF RETAINED TEETH
D )B Y SCALING

41- What it is angle classification of malocclusion ?


Angle classification depends on the relation betwwen the mesiobuccal cusp of the upper
first molar which occlude in the buccal cusp of the lower first molar buccal groove this is a
CLI, a CLII is the mucobuccal cusp occluding mesial to the buccal groove class III which the
mesiobucall cusp occludes distal to thr buccal groove

42- A 7 year old boy has previously had all primary molars restored and a pulpotomy on
upper right E. He has an early mixed dentition with lower lateral incisors erupting. There is a
midline diastema of 2 mm. The upper right E has become symptomatic and requires
extraction.

The most likely long term effect of the extraction on the occlusion is:

A. Early eruption of the second premolar.

B. Loss of upper central line.

C. No significant effect.

D. Overeruption of the lower right teeth.

E. Potential crowding in the upper right quadrant

I think answer is e but not sure

43- An eight year old boy presents with an anterior crossbite involving upper right 1 and
lower right 1. There is labial attrition on the upper right 1 and a mandibular displacement of
2mm.

What component of your upper removable appliance would you use to correct the position
of the upper right 1?

A. Adam’ clasp

B. T spring

C. Palatal finger spring

D. Southend clasp

169
170

E. Labial bow

Answer i think is t spring because the spring pushes the incisor labially to disclude from the
in-standing postion.

44- A 12 year old patient presents with bilateral crossbite and asymmetric increased overjet,
with incomplete overbite.

What is the most likely causative factor?

A. Lower lip resting between upper and lower anterior teeth.

B. Nail-biting.

C. Persistent digit-sucking habit.

D. Tongue thrust on swallowing.

E. Use of a ‘dummy’ or comforter as an infant.

Answer is c

45- An adult patient complains of her prominent upper front teeth and receding chin. She
says she did not get ‘braces’ when she was younger because the family had to move area a
lot. Her oral care and health is good, and she has a 9 mm overjet.

Where is the most suitable place to refer her?

A. A GDP friend who has a special interest in orthodontics and has been trained to use fixed
appliances.

B. A specialist practitioner who uses a lot of functional appliances.

C. A private specialist practitioner, because she’s too old to get NHS treatment now.

D. A hospital consultant, as it is likely she will need surgery now to correct her problem.

E. The nearest dental hospital, although it is 70 miles away.

Answer is d

46- You are meeting a three year old patient for the first time. His mother explains that he
had a unilateral cleft lip and palate that was repaired in infancy. She has heard that
orthodontics will usually be required when he is older. She asks you what is the commonest
orthodontic problem that occurs with a repaired cleft palate.

What is the most appropriate answer?

A. A contracted maxillary arch

B. An anterior open bite.

C. An elongated maxillary arch.

170
171

D. An expanded maxillary arch.

E. An increased overjet

Answer is a

47- A nine year old boy presents with a class I occlusion with no crowding or overjet with a
grossly carious upper left 6 which is not suitable for restoration. The upper left 7 is very near
to eruption. Upper right 6, lower right 6 and lower left 6 are sound and fissure sealed. The
oral hygiene is good.

What is the most appropriate extraction pattern for this patient?

A. Extract all four 1st permanent molars

B. Extract upper left 6 and lower left 6

C. Extract upper left 6 and lower right 6

D. Extract upper right 6 and upper left 6

E. Extract upper left 6 only

Answer is e pink book page 138

48- A child of 9 with a crowded mixed dentition attends with her parent. The child reports
that the upper first permanent molars are very sensitive. On examination, they appear
hypoplastic and very broken down.

Which of the following management strategies are you most likely to recommend?

A. Early extraction of the hypoplastic upper molars with compensating extraction of the
lower first permanent molars.

B. Restore the first permanent molars.

C. Serial extractions.

D. Put on regular review.

E. Extract the first molars when she is 12.

Answer is d

49- A 7-year-old has a 4-mm maxillary midline diastema. Which of the following should be
done?
A]Brackets should be placed to close it.
B]A radiograph should be taken to rule out the presence of a supernumerary tooth.
C]Nothing should be done. It will close on its own.
D]Nothing should be done. Treatment should be deferred until the rest of the permanent
dentition erupts.

171
172

Answer is b . When a large diastema greater than 2 mm is present, it will probably not close
on its own. Diagnostic tests, such as a radiograph, should be accomplished to rule out the
presence of a super-numerary tooth, usually a mesiodens

50- a convex facial profile is associated with the:


a. class I jaw relationship
b. class II jaw relationship
c. class III jaw relationship
d. none of above

Answe is b

51- Which appliance is tissue borne?


Bionator
Anderson appliance
Frankles appliance

Answer is frankles all appliances page 322 master dentistry

52-what is the skeletal classification of the following?

ANB=6,ANB=1
1-is skeletal cl 2
2-is skeletal cl 3

53-treatment of mild crowding for adults?


Patient should accept no treatment is required

54-treatment of severe crowding of adult?


Remove the tooth that is most displaced

56-most cause of anterior cross bite?


Thumb sucking

57-what are the type of head gears?


a-reverse head gear (class 3 orthodontic problem):it corrects the cross bite by forward
pulling of the maxilla promoting its growth and restraining the growth of mandible
b-high pull gear
c-cervical pull gear; it reduce the overbite by allowing the extrusion of the molars .force
applied is 500mg and needs to be worn for 14-16 hours per day

58-what does thumb sucking do?


Increases the overjet,tipping of the mandibular teeth,anterior open bite,narrowing of the
maxilla and posterior cross bite

59-impacted canine what is the iont?


5i

172
173

60-extensive hypodontia what IONT?


5h

61-malocclusion what age do you look for?


a-chronological age
b-developmental age

answer is b

62-adam clasp is used for what and what is its thickness?


Adam clasp is used for retention
Thickness is 0.7mm for permanent molars
0.6mm for premolars and deciduous molars

63-any othrognathic required wait until the growth spurt which is?
Female
Males 13-14

64-leeway space?
It is the deference between CDE and the 345 in the lower arch greater than the upper
arch,3.5mm and 1.5mm respectively.

65-what is a quad helix appliance?


A very slow expansion appliance made of a w shaped 1mm wrought wire and attached
bands on 6activated by expanding ½ tooth width per side before placement for correction of
crossbite

66- how to treat an anterior cross bite?


Z spring

67-active component of an orthodontic appliance?


Spring ,,screw and elastics

68-what are the passive component of orthodontic appliance?


Adam clasp,base plate

69-type of functional appliance?


Twin block
Median opening activator
Herbest

70-what is a fixed functional appliance ?


Herbst

71-2 adam clasps and a plastic denture what is that appliance?


Quad helix appliance

173
174

72-What is Holdaway line and ricketts line?


Holdaway line:the line which joins the upper lip and chin; for optimal facial aesthetics the
lower lip should lie ±1 mm to this
line and when extended, the line should bisect the nose
rickett line :is the line extending from the tip of the nose to the chin where the lower lip is
placed 2mm infront the E plane the upper lip is lyning slightly behind

73-what are the passive orthodontic appliances?


Retainer appliance , habit deterrent and space appliance

Sedation and analagesia


1-The primary mechanism of action of local anesthetics involves the blockade of which of
the following ion channels?
A.Inactivated, voltage-gated, Na+ channels
B.Inactivated, ligand-gated, K+ channels
C.Resting, voltage-gated, K+ channels
D.Resting, ligand-gated, Na+ channels.

Answer is a

2-Which of the following molecular events makes the nerve membrane more sensitive to
the action of local anesthetics?
A.Increased extracellular concentration of Ca2+
B.Increased extracellular concentration Na+
C.Decreased extracellular concentration of Cl-
D.Increased extracellular concentration of K+
E-increase in the extracellular concentration of the mg++

Anser is d

3-When a local anesthetic is applied to a nerve fiber, which of the following nerve properties
will most likely increase?
A.Rate of rise of the action potential
B.Conduction velocity of the nerve
C.Amplitude of the action potential
D.Resting potential of nerve membrane
E-refractory potentional of the nerve
Answer is e

4- Which of the following functions is blocked last when a local anesthetic is applied to a
nerve?
A.Motor-function
B.Pain-sensation
C.Muscle-tone
D.Temperature sensation
E-sympathatic activity

174
175

Answer is a
"Differential Blockade." Type B fibers (sympathetic tone) are the most sensitive followed by
Type C (Pain), Type A delta (temperature), Type A gamma (proprioception), Type A beta
(sensory touch and pressure) and Type A alpha (motor). Although Type B fibers are thicker
than Type C fibers, they are myelinated, and thus are blocked before the unmyelinated, thin
C Fiber.
5-Which of the following nerve fibers are most sensitive to the action of local anesthetics?
A.Resting fibers
B.Rapidly firing fibers
C.Fibers with high conduction velocity
D.Unmyelinated fibers
E.Fiber of large diameter

Answer is b

6- What is the optimal volume of local anaesthetic solution for a true anterior superior
alveolar nerve block
A)1.0ml
B)1.5ml
C)0.5 ml
D) 2 ml

Answer is a

7-What is the duration of a slow injection of 1 ml of local anaesthetic?


A)30-sec
B)60-sec
C)15-sec
D)2 min

Answeris-a

8-which depth of penetration places the needle tip in the vicinity of the foramina through
which the posterior superior alveolar nerves enter the posterior maxilla in an adult
A)16mm
B)20mm
C)10mm
D)30 mm

Answer is c i think

9- Which local anaesthetic agent is most appropriate for use in children


A) 2% bupivacaine with 1:20000 levonordefrin
B) 2% lidocaine with 1:100000 epinephrine

175
176

C) 2% mepivacaine
D) 0.5% bupivacaine with 1:200000 epinephrine

Answer is b

10- local anaesthesia used to anasthetise the uppwr and lower molars?

Mandibular molar…inferior dental block

Maxillary molar…infilteration and posterior superior alveolar block(rarely used)

11- Picture of LA cartridge asking which anaesthetic,xylocaine was written on it

Lidocaine

12-. Anaesthetic that produces toxicity,longest acting,short acting,used for


restoration,penetrates bone, cardiotoxic and which to be given to heart patients

BUPIVACAINE …long acting anesthesia and toxic

Prilocaine and felypressin / mupivacaine without adrenaline….patient with heart


problem such as ischemic heart diseases.

Mepivacaine……..short acting anesthesia, for restoration

Lindocaine….heart patients

Articaine …penetrates bone

Bupivacaine …most cardiotoxic,scully page 53

Articaine 4%...are mildly nuerotoxic ,occasionly neurotoxic scully page 53,

13- what is the action mechanism of lidocaine?

Answer it inhibits conduction by blocking the sodium voltage gated channel

14.What LA has the longest duration of action?

Bupivacaine

176
177

15- Which LA is neurotoxic at 4%?

articaine

16- Which LA do you give a patient with cardiac arrhythmias?

Mepivacaine

17-whicl local ansthetic agent is used in pulpotomy of a child?

Lidocanine

18- LA for a normal & healthy adult?

Lidocaine with adrenaline

19-true or false: is .Intra-ligamentary injection of LA is an adjunct to the inferior


alveolar nerve block.

It is true pink book page 609

20- Picture with a dot marked on maxillary canine. Which type of anesthesia?

Labial and palatal infilteration

21-.Which nerve block for upper first premolar?

The used of buccal and palatal inflteration or infraorbital block to anasthetis the
middle superior alveolar nerve

22- Along with inferior alveolar nerve block & lingual nerve block, which other nerve
has to be anesthetised?

The long buccal nerve

177
178

23- when taking mono amino oxidase inhibitors ( MAOI) which are is contra indicated:
A.Barbiturate
B.Local anaesthetic
C.Pethifine
D. Acetyl salicylic acid

Answer is c

24-. What’s the concentration of lignocaine in topical anaesthetic?

a-0,5%,

b- 1%

c- 2%

d-5%
answer is d

25- What can you reach during ID block?

If the needle is inserted too far laterally you can reach the temporalis

If the needle is inserted too fae medially it will inter the pterygoid

If the needl is inserted centrally it will enter the parotid gland

Reference is mcminn atlas page 267

26- Which one of the following types of pain is most likely to be associated with cranio
Mandibular disorders
A.Exacerbated pain by hot or cold food
B.Keeps patient awake at night
C.Associated with muscle tenderness
D. Associated with trigger spots related to the trigeminal nerve

Answer is d

27- . Everything about intraligamentary anaesthesia: can it affect permanent tooth


germ?

Does the needle need to be smaller than periodontal ligament? Is it painful? Soft
tissue anaesthesis in intraligamental anaesthesis

Intraligementary injection can be used to produce anaesthesia of a single tooth. It


can be used in conjunction with other LA to produce a more profound
anaesthesia.special small ultra fine needles are used injection into the periodontal

178
179

membrane can be painful after because of tooth extrusion.can predisposed infictive


endocarditis and indicated in for mandibular teeth and patient with bleeding
disorders and yes it does affect the permanent tooth bud ,sot tissue anaesthesia is
limited.

28- What’s the most predictable anaesthetic technique for lower lateral incisor? And
for lower molars

Lower lateral incisor...infilteration

Lower molars...inferior dental block

29- Iff ID block is given for a restoration on 6 then what would u ask the pt not to do?

Not to bite on the cheeks or chew on the lips and not to have hot drinks

30- ID block which muscle is pierced? The buccinators

31-. Do all the areas which will be anaesthetisized when giving blocks inferior
dental.infraorbital

Inferior dental block:lower teeth ,lips,chin, buccal mucousa from midline to


premolar

Lingual nerve: 2/3 of the anterior of tongue,floor of the mouth and lingual gingivae

Long buccal :buccal mucosa of lower molars

Nasopalatine block:upper incisors and canine

Ifraorbital block:cental incisors up to first premolar(anterior and middle superior


alveolar block) upper lip ,cheek, angle of nose, lower eyelid and buccal gingiva

Greater palatine block; maxillary molars ,premolars and canine, palatal


mucoperosteal

32-toxicity as a result of anaesthetic solution can be seen more when:


A.Injection in supine position
B.Injection into vascular area
C.Injection without vasoconstrictors
D. Intravenous injection

179
180

Answer is d

33- . Local anaesthesia short acting long acting, which is good for bone which is
present in the topical preparation.

Long acting….bupivacaine

Short acting…mepuvicaine

Good bone…..articaine

Topical anesthesia…lidocaine and benzocaine

34- Concerning local anaesthetics which one

i) has d longest duration of action ,,,,,bupivacaine

(ii) has shortest duration of action,,,,,,mepivacaine

iii) is commonly used for topical anaesthesia,,,,,,,lidocaine and benzocaine

(iv)penetrates bone the most……articaine

v)causes methaemoglobinaemia…..prilocaine and benzocaine

35-.what is the best LA for a pregnant woman in last semester?

a-lidocaine

b-mepivacaine

c-bupivacaine,

d-amethocaine,

e-perilocaine,

answer is a

36-. how many ml of lignocaine 2% 1/80000 can be injected to a 20 kilogram


person?

a- 2.2ml

b-4.4ml ,

c-6.6ml,

180
181

d-12ml

answer is 4.4ml

37- how many ml of lignocaine 2% 1/80000 can be injected to a 125 kilogram


person?

a-16ml

b-24ml

c-26ml,

d-30ml

38-maximum number of LA cartilage 2% lidocaine with adrenalin of 1;8000 for a


seven year old child weighs 23 kg

Every cartilage contains 1.8ml and 2% licocaine means there is a 20 mg in every 1 ml.

So by doing ratio and proportion ;

20mg 1 ml

X 1.8ml this will lead to the following equation X=1.8*20=36mg of lidocain in a


1.8ml

Again to find the maximum does of lido in a 23 kg child we do ratio and proportion
again maximum does per kilo is 4.4mg

4.4mg 1kg

X 23kg this will lead to X=4.4*23=101.2 mg maximum does of lidocaine

To find the number of catriges =101.2/36mg=2.8 catrilages

If we wanted it in ml we divide by twenty =101.2/20=5ml

39- Local anaesthesia – many different questions

a- maximum dose for child 5 years with 20 kg – 2 cartilages ,2*4.4 ml

b- maximum dose for adult 125 kg – 20ml

181
182

c- longest duration –

d- you’re not using adrenaline but requires vasoconstrictor for long duration –

answers a-4ml,b-7cartliages master dentisrty-10 cartligaes pink book,c-


bupivacaine,d-felypressin

40-substance in LA that causes allergy is?

Sodium metadisulphite,latex due to the rubber and methylparaben

41-what is the spray local anaesthesia?

Lidocain which is 10 %

42- Anaesthetic solution for child with herpetic gingivostomatitis before eating?

a-Benzocaine

b-lidocaine

answer is b

cause benzo is found in form of gel and paste whereas lignocaine is found as gel,spray
solution and ointment

42More toxic anaesthetic –

mepivacaine……. check answer

43--Maximum dose of anesthesia??

Lidocaine….4.4mg/kg up to 300mg or 7 cartridges ……children…2 cartridges

Articaine…7.0mg/kg up to 400mg or 5 cartridges……children…1.5 cartridge

Prilocaine..6.0mg/kg up to 400mg or 4.5 cartridges…..children 1.4cartrigdes

Bupivacaine…2.5mg/kg

44- lidocaine max dose for a 5 yearold patient 20 kg?

182
183

4.4 by 20=88mg

1ml contains 20mg so syringe of 1.8ml contains 36mg

88/36=2.4 cratrigdes but because the maximum does of a child 20 kg body weigh is
only two cartilages we should not excessed that limit and therefore the maximum nu
of cartilages given is two

45- anasthesia given to heart disease??

Prilocaine with or without felypressin

46- needle gauge in intraligamantary injection?

30 (the higher the number the lower the lumen)

47- what is the best LA for pregnant women in the last semester?

a-lignocaine

b-mepivacaine

c-bupivacaine

d-amethocaine

e-prilocaine

anser is a

48- block given to anesthetize buccal side of 3rd molar extraction?

a-nasopalatine

b-inferior alveolar

c-long buccal

answer is c

49- which nerve is anesthetized in the anterior border of ramus and 1cm above the
occlusal plane of the lower posterior teeth?

a-lingual nerve

b-long buccal

183
184

answer is b

50- anesthesia for epileptic patient? Lidocaine

Scully page 346

51-what is the dose for bupivacaine for calculation? 2.5mg/kg

52-prilocaine syringe how many ml?

2.2ml….reference BNF page 798

53-articaine syringe how many ml?

2.2ml

54-which anesthesia is cardiotoxic?

Bupivacaine

55-which la causes neuropathy?

Articaine

56-which LA technique best describeds the following;

1-blocks sensation in trigeminal nerve mandibular branch by deposting anesthesia at


head of the condyle? Gow gates technique

2-injection 0.5-1ml of LA and buccal to the last molar? Long buccal nerve

3-injection apical to and between lower 1st and 2nd premolar? Mental nerve block

4-in edentate this technique should be done higher than the ridge of the alveolar
bone? Inferior alveolar block

5-needle should be positioned submucosally lingual to premolar and inject 0.5ml


LA? Sublingual nerve block

6-rare technique and done by inserting needle distal to the upper second molar? PSA

184
185

Options

a-inferior alveolar block

b-long buccal nerve

c-gow gates technique

d-posterior superior alveolar nerve block

e-mental nerve block

f-sublingual nerve block

57-what is the duration of action of bupivacaine? 6-8 hours

58--lingual nerve damage causes

a-taste sensation on the tip of the tongue affected

b-taste sensation on the tip of the tongue spared

c-taste sensation on the same side is lost

answer is c..reference www.dentalindia.com/lingualnerveingury

59- what anesthesia should be avoided in pregnant women/

a-lignocaine 2%
b-mepivacaine 2%
c-mepivacaine 3%
d-prilocaine 3%
e-prilocaine 4%
answer is d…cause it contains 0.3IU of felypressin which act the same as oxytocin
hormone and induces labour

60--what is the best ansethsia for a pregnant women in her last semester? and best
useful for molar filling for an adult female?

a-lindocain

185
186

b-mepivacaine

c-prilocaine

d-amithocaine

answer is a

61-Which sedative can be given orally 1 hour before surgery?


Temazepam
Midazolam
Nitrazepam

Answer is temazepam

62-Anaesthetic drug of choice for COPD patients


Ketamine
Etomidate
Propofol

Answer is ketamine reference in scully page 62.ketamine is a vasodilator and is useful in


sever bronchospasm.

63-Which is the most commonly used intravenous GA? Propofol

64-Reversal agent of midazolam?


Fluroethane
Flumazenil
Flamazine

Answer is flumazenil

65-Action of benzodiazepines on GABA receptors?


Enhances
Blocks

Answer is enhances

66-Which of d following is a sign or symptom of lidocaine overdose?


Tachycardia
Light headed ness
Rash
Hypertension

Answer is lightheadness

186
187

67-best sedation for extraction of wisdom tooth?

Intravenous sedation with midazolam 10mg/5ml

68-. What type of injection is contraindicated in primary teeth?


A.Intraosseous
B.Infraorbital
C.Periodontal ligament
D.Posterior superior alveolar
E.Inferior alveolar

I think its periodontal ligemant because it will affect the t permanent tooth germ

69- an 18 year old anxious patient with asthma in a boarding school went to you
dental office for multiple extraction of 3rd molar what is the best sedation technique//

a-inhalation sedation
b-IV
c-Deep sedation
d-hypnotic

70--nitrous oxide is not used alone as a general anesthetic agent because of?

a-difficulties in maintaining an adequate o2 concentration

b-adverse effect on the liver

c-poor analgesic effect

answers is c (answer has been checked on www.rsceng.ac.uk)

71- lidocaine/adrenaline –duration of action

Pulpal;1.5 hours and soft tissue 3hours

72-for teenager doing exo for all her premolar,will we choose IV sedation or
inhalation?

187
188

Inhalation…I am not convenced check this one

73- which is the water solubale benzodiazepan? Midazolam

74- antagonist of benzodiazepan? Flumazenil

75- Anaesthesia and Sedation 5 sub questions for different clinical situations –

a-anxious pregnant woman

b – un- co-operating child with 4 molars to be treated

c-nervous patient for extraction

d- normal pt for simple restorative procedure

option are:

1-LA

2-inhalation sedation

3-IV

Answers i think are a: IV if in the first 3 months of pregnancy and inhalation sedation
if in the last two trimester

b-inhalation or GA

c-IV

d-LA

76- asthmatic patient analgesic medication after extraction ? paracetomol

77-child with 12 years medication after extraction ? Paracetomol

76- Anaesthesia indicated for patients with COPD ?

GA is given only if an absolute necessary use ketamine

LA is preferred to be used agent is lidocaine avoid injection bilateral madibular and


palatalinjection should be avoided

188
189

Diazepam and midazolam is contra indicated in because of respiratory depression

Inhalation sedation is contra indicated because of an increase in oxygen and


therefore affecting the hypoxia which drive the respitaory process ( usually it is the
co2 change but in COPD it is the hypoxia)

78- Anaesthesia indicated for pregnant woman ? Ligocaine

79-Anaesthesia indicated for epileptic patient ? lidocaine

80-which is mandatory during inhalation sedation

a-ECG

b-pulse oxymeter

answer is b i think this question is wrong (note that in conscious sedation in dentistry IN
SCOTTISH GUIDLINE recommend that pluse oximetry IS NOT A ROUTINELY REQUIRED FOR
INHALATION) sscavening is maditory . IV sedation pluse oxymeter is manditory

81- Questions on conscious sedation and GA; their applications in various clinical
settings; inhalational and iv sedation

Type of sedation in upper respiratory disease…..IV

Type of sedation used in the first trimester…….iv

Type of sedation for epileptic patient…..Inhalation sedation

Type of sedation for teenagers doing multiple extraction….iv sedation

Type of sedation for children doing orthodontic extraction...inhalation

82- What is nitrous oxide…soluble or insoluble in blood?

Only poorly soluble so it’s considered insoluble

83-inhalation sedation is of the most benefit to which of the following?

a-medically comprised

b-teenagers

189
190

c-pregnant women

d-children

answer is d

84-Where is iv sedation indicated where contraindicated? Scully page 56

Indicated- patient with learning impairment, faint with local anesthesia, stress may
induce angina or asthma or hypertension or epilepsy, upper airway obstruction,
claustrophobic patient

Contraindication; psychotic,sever heart disease,airway obstruction,COPD,in children


.pregnancy.liver diseases,needle phobic,mysthemia gravis and glaucoma

Indications and contraindications of sedation.

Page 582 pink book fifth edition and 50 page master dentistry contra indication as
follow:

Cardiorespiratory, renal, liver or psychiatric pathology

Unescorted patient

Demonstrate adverse reaction to sedative agents

Pregnancy during which benzodiazepine should be avoided (midazolam can be used


safely if required during the first six months of pregenancy)

85-what is pKa?

Is used to describe strength of acids. The larger the pKa value, the more dissociation of the
molecules in solution and thus the stronger the acid.

86) A patient receives ketamine for sedation and analgesia. Her eyes are open but her
breathing is labored and she is completely unresponsive to both verbal and painful
stimulation. This corresponds most closely to a state of
A.Anxiolysis
B.Moderate sedation

190
191

C.Deep sedation
D. General anesthesia

Answer is c

87. which are the nerve fibres responsible for sharp stabbing pain??

A) A delta fibres
B)C delta fibers
C)none

Answer is a

88-which nerve fibres are responsible for visceral pain ??

A) A delta fibres
B) C delta fibres
C) Both

89-which are the main excitatory neurotransmitters for pain??

A) Substance-P
B)Glutamate
C)Both

90-Respiratory depression caused by opioids is reversed by??

A)Naloxone
B)Flumazenil
C)morphine

Answer is c

91-which of the following Local anaesthetics can cross the placenta??

A)Lidocaine & Articaine


B)Mepivacaine & Bupivacaine
C)Lidocaine & Mepivacaine

Answer is c

92-which of the following statements are true??

A) Propofol is a ultra-short acting anaesthtic


B) conscious sedation is suitable for patients who come under 1&2 category of ASA
(American Society of Anaesthesia)

191
192

C) Nitrous oxide is a weak analgesic but excellent anaesthetic


D)Diazepam is supersede by Diazemuls
E)all are tue
F)A&C are true

Answer is e

Q46 You decide to refer an eight year old child to the oral surgery department in your local
hospital for extractions under a general anaesthetic.

What key reason for asking for a general anaesthetic would you put in the referral letter?

A. Parents request GA.

B. Failed to complete treatment under inhalation sedation.

C. Child would not accept local analgesia

D. Parents think hospital more convenient.

E. Not one of my regular patients.

Q75 Any patient receiving treatment under IV sedation must have their blood pressure
recorded as part of their assessment.

What is the maximum blood pressure that is generally regarded as being compatible with
safe sedation in general dental practice?

A. 160/95

B. 140/95

C. 160/90

D. 170/100

E. 120/80

Answer is C

Q90 Lignocaine (2%) is widely used in dental procedures. It is most often used in
combination with epinephrine (1 in 80,000).

In which one of the following patients is the use of epinephrine containing local analgesia
potentially hazardous?

192
193

A. Patient with severe hypertension

B. Patient on tricyclic antidepressants

C. Patient who is an alcoholic

D. Patient on monoamine oxidase inhibitors (MAOIs)

E. Patient with Grand Mal epilepsy

Answer is a reference page 273, it does not interfere with tricyclic or MAOI ,no proof it
prevokes a fir, in alcoholic patient doeses should be reduced due to the possibility of liver
damage , well know epinephirine to increase enhace hypertension due to voasoconstriction

A patient is having a lower right 8 extracted under intravenous sedation with


midazolam.

Which one of the following would be a sign of over-sedation?

A. Tachycardia

B. Bradycardia

C. Hyperventilation

D. Flushing of the skin

E. Sweating

Answer is bradycardia

PHARMACOLOGY

1-A patient in need of antibiotic but allergic to penicillin and can not swallow?

Azathomycin and or erythromycin (it interferes with bacteria protein synthesis)

2- DRUG THAT CAUSES CONSTIPATION ? CODEINE


3- ANTAGONIST OF HEPARIN? PROTAMINE
4-Definition of INR ?International normalized ratio (INR) is a calculation made to
standardize prothrombin time. INR is based on the ratio of the patient's prothrombin
time and the normal mean prothrombin time
5.Have to know the main medication used for the most important
diseases,Interaction, Collateral effects (dry mouth, gingival growth for example )
Carabmazepine…..epilepsy and TN

193
194

Vancomycin…….MRSA (methicillin resistant staphylococcus aureus)


6-. Antifungal (Thrush) Topical :
nystatin suspension 100 000 rinsed and then swallowed QDS for 7 days or
nystatin pastille 1 QDS
7.Drug is most likely to cause rashes in infectious mononucleosis ? Ampicillin
9.Which drug should be avoided in patient on warfarin-metronidazole? fluconazole
10.Which medicine to avoid in asthmatics? NSAIDS
10.Which antibiotic is contraindicated in lactating mothers?
Metronidazole (significant amount in breast milk manufactures advice to avoid),
tetracycline(cause discoloration to infant teeth)
11.Age of MMR vaccine
one month after the child’s first birthday ( 13months) and a booster dose can be
given just before starting school at around 3-4 years of age
12.Antibiotic alone to granuloma? Options: mixed infections, antibiotic doesn’t
penetrate in the bone…check this question
13.A lot of questions on warfarin, its interactions with various drugs and INR
Safe INR is (2- 3) Warfarin interacts with fluconazole, metronidazole and
erythromycin
14. Lots of questions on Penicillins, their modes of administration and doses
ORAL, IV AND IM.
250 -500mgs QDS
Penicillin G is a benzylpenicillin doesn’t work orally so should be given parenterally
Penicillin V is a phenomethylpenicillin given orally
15.Drugs causing xerostomia
Atropine, certizine, chlorpromazine, diazepam, morphine.
16. Drugs used to treat Candidiasis in HIV infected condition- fluconazole
17. Lots of questions on NSAIDS
18.Drug is most likely to cause a rashes in infectious mononucleosis (glandular
fever)?
a.Aspirin
b.Penicilllin G
c.Peniicillin V

194
195

answer is c

19.Which drug should be avoided in pt. On warfarin?


a.Fluconazole-yes
b.Miconazole
c.Nystatin
d.Amphoterecin
20. antibiotic for ear use
Amoxicillin is the most common one (scully)/ I FOUND PENCILLIN PAGE 355
SCULLY.
21.antibiotic which can be given thrice daily ? amoxcillin
22. what antibiotic used for beta –lactamase (penicillenase) producing organisms?
co amoxiclav reference is Scottish guideline second line antibiotic
23.analgesic that can be given in warfarin ? paracetamol
24.What drug potentiates warfarin?
http://www.parkhurstexchange.com/files/PDF/2008/08-06%20Major%20Bleeds
%20(Checklist).pdf
25.What drugs should not be given in pregnancy?
a. Nalixdicic acid
b. Metronidazole-yes
26. What antibiotic is used to treat ANUG?
metronidazole 200mg tds for 3 days
27. What antibiotic is used to manage a super infected herpetic lip lesion?
a. Ciprofloxacin
b. Metronidazole
c. Cefuroxime (TRUE)
28. What drugs are inhibited by beta lactamase? Pencillins

Which medicine give red colour spitting saliva? Rifampicine

Asthma patient who takes steroid inhaler QDS feel disorientated during biopsy. What to
administer

Hydrocortisone

195
196

GTN

Aspirin

Chlorphenaramine

Glucose

Glucagon

Answer glucose

29.Side effects of dapsone?


Dapsone is a drug combination of rifampicin and clofazimine used for treatment of
leposy and toxoplasomsis
Hemolysis ( most common ) and metheglobinemia, hepatitis, cholistatic jaundice,
nausea, headache, rash,nausea ,loss of apetite
30. Side effects of Azathioprine, same options as side effects of dapsone ?
Azathioprine is an immunosuppresent
Most common side effects are gastrointestinal: diarrhea (5%), nausea (3%),
abdominal pain (3%), and vomiting.
Occasionally, patients have developed cholestatic hepatitis or delirium.
In scully found the following side effects …myelosuppression
hepatotoxicity,leukopenia thrombocytopenia,low TPMT (thiopurine
methyltransferase), infections,lymphoproliferation disorder, malignancies,
tetragenicity.
31. What analgesics cause
a. Ringing in the ears after overdose   Aspirin

b- Liver damage after overdose? acetaminophen (paracetamol)


http://www.medicinenet.com/tylenol_liver_damage/article.htm
c. Constipation Constipation is a classic side effect of narcotic analgesics like
morphine, codeine or dihydrocodiene
32. A 60 year old man on anti-hypertensive has gingival enlargement what drug can
cause it
a. Ciclosporin
b. Phenytoin
c. amlodipine

196
197

all drugs cause gingival hyperplasia ( ciclosporin and phenyton cause gingival
enlargement) but the patient is a hypertensive so is most likely on amlodipine
33.Drug given 3 times a day? amoxycillin
34.Which antibiotic is contraindicated in lactating mothers? Tetracycline
35. Drugs causing gingival hyperplasia
Ciclosporin………………………………………………………………………immunosuppressive
, Ca channel blockers like amlodipine and nifedipine………….antihypertensive drug
, phenytoin …………………………………………………………………………..anticonvulsant drug
and sometimes oral contraceptives
36. MRSA which medicine is effective ?Vancomycin
37. An EMQ on use of antibiotics in various clinical scenarios eg in asthma, upper
resp tract infection, TB ,which drug in overdose causes hepatotoxicity? etc whereas
the options for the drugs were Aspirin, Ibuprofen, Paracetamol, Diazepam,
tetracycline.
38. Patient with peptic ulcer which analgesic would you prescribe?
aspirin,
ibuprofen,
PARACETAMOL is the right one.
39. Which of the following analgesics produce constipation after 5 days of taking it?
Codeine was the correct answer Norcatic analgesic cause constipation
40. Which drug causes severe abdominal cramps? Clindamycin ( antibiotic induced
colitis)
41. Which drug causes constipation? Dihydrocodeine
42. Picture of a vial marked Xylocaine. Contains lidocaine/lignocaine
43. Picture of medial surface of mandible with a syringe.
Inferior alveolar nerve block
44. Drug given to a patient with dry socket? Metronidazol
45.Drug for a patient after 3rd molar extraction? Ibuprofen, amoxicillin.
46. Drug for angular cheilitis? 2% Miconazole
47. Which antibiotic is prescribed in chronic sinusitis?
Responds better to drainage plus metronidazole with amoxicillin, erythromycin,
clindamycin or a cephalosporin

197
198

Acute sinusitis :10 day course of amoxicillin or tetracycline or clindamycin (scully)


According to scotttish guidelines it it amoxicillin 500mg *4 for 7 days
48.Vitamin K antagonist? Warfarin
49. Test to be done in a patient taking warfarin? INR
50. Which drug potentiates the effect of warfarin?
Metronidazole althought there are other drugs such as erythromycin,
51. Patient with allergy to penicillin.
Clindamycin/erythromycin or metroniadazole
52. Drug for herpes zoster? Aciclovir
53.Which drug do you prescribe to a patient 2 days after the extraction of a 3rd molar
if the socket has not yet healed & there are 2extra-oral draining sinuses?
Penicillin………….i think it is as severe infection as there are two exral oral sinuses
patient should have a combination of metronidazole and pencillin
54. While writing a report after LA what should be noted? Blood aspiration
55. Drug for trigeminal neuralgia?
Carbamazepine start with 100-200mg twice a day and may be increased to QDS upto
1600mg
Side effect of carbamezapine , ataxia ,drowsiness , leukopenia , dry mouth,erythemia
multiforme and dyskenisea.
Moa it stabilizes the action of sodium channles
56. What should be added to beta-lactamase to make it more efficient against
anerobes?
Co-amoxyclav Clavulanic acid
57.Healthy young patient. What’s the INR?
0, 1, 2, 3, 4, 5 1
0.8-1.2
58.What’s the depth of topical anaesthetic?
2-3mm…..master dentistry page 63
59.What’s chlorhexidine family
quaternary ammonium compound, bisbiguonade, phenol
The answer according to wikipidia states bisbiguonade
60. Phenytoin X

198
199

gingival hyperplasia
61.Warfarin antagonist? vitamin K
62. Infective endocarditis ab cover several questions
63. Cholhexidine mouth wash and gel percentage? 0.2 % mouthwash and gel is 1%
64. Mast cell stabilizer?
(options were salbutamol,adrenaline,ipratonium bromide, cromolyn or sodium
cromoglycate
65.Which drug causes a fixed ulcer?
According to scully page 630..antibiotics,antiseptic,barbiturates,dentifrices, mouth
washes,phenacetin,sulphonamides,tetracycline
66. Antibacterial in toothpaste? Triclosan
67. Chlorhexidine its side effects
disturbed taste and brownish discolouration and swelling of the parotid gland
http://www.drugs.com/sfx/chlorhexidine-side-effects.html

68. Prophylaxsis of infective endo carditis different scenarios, for eg-dose in kids,
dose with allergy ?
According to NICE guidelines no antibiotic prophylaxis is required
69. Concentration of chlorhexidine solution commonly used in uk? -0.2%
70. Concentration of chlorhexidine gel commonly used in uk? - 1%
115. Which of these drugs stabilizes mast cells, stopping release of histamine
a)adrenaline (b)salbutamol (c)ipratropium (d)triamcenolone acetonide
answer is salbutamol (beta 2 adrenergic agonist).

71-what is the best sedation opt for extraction of wisdom teeth? Intravenous sedation
72-Which one you use if you want to have anaesthetic longer after operation?
(bupivacaine)
73. which options is mandatory during doing inhalation sedation?
Opt: Pulse oximetry
74.Special syringe for soft tissue anaesthesia….
Intraligamentry
75. INR in a 70kg man not under anticoagulation?
.1 Normal range between 0.8-1,2

199
200

76.Antibiotics action:
- Trimethoprim (inhibit folate)
- Ciprofloxacin (inhibit DNA metabolism)
- Aciclovir action – causes inactivation of DNA polymerase
- Medicaments action - competitive, not competitive, functional, etc. (I don’t
remember the medications… ACE inhibitor)
- Bacterial resistance – plasmids
77. Vaccine more common with inactivated virus –
Influenza ( Flu vaccine is of inactivated virus )
78.MMR vaccine age –
after 13 months ( 1 month after the child’s first birthday, booster dose pre school )
79. Medication for sinus (not direct asking, gave us clinical situation to identify
sinus: headache, pressure on moving head) –
Ampicillin,erythromycin or tetracycline plus ephedrine
80-which analagesia causes ringing in the ear after overdose?
Aspirin causes tinnitus other drugs include erythromycin ,frusemide ,quniune
81-do we give prophylaxis antibiotic for patient with mitral valve defect? No
82-do we give metronidazole for persistent dry socket ?
No unless persistent with infection
83-best treatment for patient with cured denture stomatitis but resistant angular
chelitis?
a-micronazole
b-fluconazole
c-nystatin
answer is a
use combination of hydrocortisone 1% and 2%micronazole twice a day for 7 days
84-antibiotic in child allergic to penicillin and cant swallow pills??
Erythromycin or metronidazole suspension
85-which of the following does not cause dental ulceration?
a-NSAID
b-atenolol

200
201

c-captopril
d-methotrexate
e-essential oil
answer is e
ATENOLOL;IS A BETA BLOCKER
CAPTOPRIL IS ANGIOTENSINE CONVERTING ENZYME
Nefidipine;ca+ channel blocker
Amlopidine is a calcium channel blocker
Methotrexate is used in treatment of cancer and ectopic pregnancy it inhibites folic
acid adhesion
86-inhalation anesthesia is the most benefit to which of the following?
a-medically compromised
b-teenager
c-pregnant women
d-children
answer is d
87-which one of the following would be a sign of oversedation?
a-tachycardia
b-bradycardia
c-hyperventalation
d-flushing of the skin
e-sweating
answer is c
88-which local anesthesia is preferred for a confirmed hypertensive patient?
a-3% prilocaine with felypressin
b-mepivacaine 3% without adrenaline
answer is a
87-antimicrobial for periodontitis;
1.aggressive periodontitis A-amoxicillin 250mg 1*3 +metronidazole 200mg
TDS 7-10days

201
202

2.NUG B-metronidazole 200mg TDS 3days


3.chronic periodontitis c-not indicated
4.periodontal abcess and fever d-debriment +amoxicillin 250mg TDS
+metronidazole 200mg TDS 7-10 days
e-erythomycin 250mg TDS 5-7 days
answer:
1-a
2-b
3-c
4-d
89-patient had a stroke what is the first drug to administer as anticoagulant?
a-warfarin
b-aspirin
c-statin
answer is b….i ythink answer is streptokinase check this question again
90-anasthesia for canine extraction? infilteration
91-emq about drug reaction;
a.blind patient taking antihypertensive drugs
b-patient with 3 months of abdominal sepsis and lingual parasthesis
rifampicin…check answer
c-new born wil cleft lip and palate mother is epileptic ? phyneton
92-photo was given with an enlarged gingiva need to match medical condition with
drugs
a-cyclosporine
b-phenyton
c-nefidipine
93-best analegisic for a 12 year old patient;
a-aspirin
b-ibuprofin
c-paracetamol

202
203

answer;c
94-emergency drugs and doses asked indirectly
95-someone on antibiotic for treatment of abdominal pain for three weeks complaine
of disordered taste what antibiotic? Metronidazole
96-which drug used for systemic candidosis? Fluconazole
97-which drug causes diarrhea and cramps in high doses?? Clindamycin (causes
colitis)
98-which antibiotic prescribed for gram + streptococcal infection? amoxillin
99-which antibiotic causes puesdomembranous closits? clindamycin
100-analagesia following tooth extraction? ibuprofin
101-drug of choice for herpes zoster? aciclovir
103-which antibiotic absorption is inhibited by chelation of milk?? Tetracycline
104-drug that is given to patient quitting smoking?
Varenicilline or dupropione ……it is a drug that mimics nicotine which smokers are
addict to.
105-hypertensive patient taking medication.facial swelling occurs .which drug?
metronidazole

106-A SORE BUT NORMAL APPEARING TONGUE ( ORAL DYAESTHESIA ) CAN BE CAUSED BY WHICH
DRUG ?
Angiotensine converting enzyme inhibitor (captopril)

107-)which drug is the common offender of allergic reactions??


A)NSAIDs
B)penicillis
C)antiviral
Answer is B

108-)which drug causes agranulocytosis leading to severe oral ulceration and can induce
aplastic anemia which effects haemostasis??
A)phenytoin
B)chloramphenicol
C)amlodipine
Answer is b

109-which drug causes Folate deficiency, Macrocytic anaemia that produces severe
aphthous stomatistis , on long term use??
A)phenytoin
B)Chloramphenicol
C)Nifedepine
Answer is A scully page 345

203
204

110-)Which of the following drugs cause lichenoid eruptions??


A)Beta blockers
B)NSAIDs
C)treatment with gold for rheumatoid arthritis
D)oral hypoglycemics
E)All of the above
F)B&D
Answer is

Medical emergency
1-Dose of aspirin for MI?
Answer is 300mg
2-. Dose for adrenaline for anaphylaxis ?
0.5mL of 1:1000 IM repeated every 5 minutes for adults
Children 0.15 ml……for 6 months to 6years
0.3ml…….6years-12years
0.5 ml……over 12 years old
3.Way of 1mg of glucagon is given?
Answer is IM
4. Emergency drugs and their modes of administration
Aspirin.. orally
Adrenaline…IM
Glucagon…IM
TGN…spray or sublingual
Glucose…orally
Midazolam…intranasal or buccal
5. Amount of adrenaline in emergency dosage;
Options are
a 50µgms,
b 500µgms,
c 50mg,
d 500mg
answer is b

204
205

as per me is- Adrenaline in emergency is 0.5mg/ml in concentration of 1:1000, i.e.,


1/1000x1000000µgms= 1000µgms/ml; if 1ml of solution
contains 1000µgms then 0.5 ml should contain 500µgm
6.Which medicine to avoid in asthmatics?
a. aspirin
b. Paracetamol/panadol
c. Tramodol
d. Penicillin
answer is aspirin
7. What is the dose of aspirin for myocardial infarction?
a. 100 mg
b. 200 mg
c. 300mg
d. 400mg
e. 500mg
answer is c
8. What drug should not be given to asthmatics? aspirin
9.Which drug causes bronchospasm in asthmatic? Ibuprofen and aspirin NSAID
10. Crushing pain in the chest radiating to arm & relieved by? Sublingual GTN
11.What is the role of aspirin in MI? fibrinolytic
12. Best drug for asthma? Salbutamol
13. How do you know that a patient has penicillin allergy? rashes, hives and itchy
skin
14. what is Salbutamol?
Short acting β2-adrenergic agonist
15. Analgesic for asthmatic?
Aspirin,
NSAID,
Paracetamol
Answer is paracetamol

205
206

16.Aspirin function? anti-platelet, anticoagulant, etc


17.Chronotropic,ionotropic definitions and what does adrenaline do?
Ionotropic: is an agent that alters the force or energy of muscular contractions.
Negatively inotropic agents weaken the force of muscularcontractions. Positively
inotropic agents increase the strength of muscular contraction.
Chronotropic: Chronotropic drugs may change the heart rate by affecting
the nerves controlling the heart, or by changing the rhythm produced by
the sinoatrial node. Positive chronotropes increase heart rate; negative chronotropes
decrease heart rate.
Adrenaline is a positive chronotopic
18-mode of action of glyceryl trinitrate GTN?
Relaxes arterial and venous smooth muscle to decrease the preload with mild after
load reduction;venous effect is more than the arterial effect
Decreases pulmonary vascular resistance
Coronary vasodilation
Improves ventricular compliance
19-what is the side effect of nitroglycerine?
a-tachycardia
b-headache
c-hypovolumia
d-hypotension
pulmonary vasodilation may worsen pulmonary shunt
20-which one is more effective GTN spray or sublingual tab?and what other forms of
GTN are there?
Tablets are more effective than spray, in terms of other forms it can be found as
patches , injections and ointment
21-if an adult patient is having a prolong seizure (status eplipticus) midazolam may
be given while waiting for the ambulance , the doses are:
a-1mg
b-5mg
c-10mg
d-20mg
answer is c

206
207

22-dosage of drug for anaphylactic 8 year old patient?


Adrenaline 0.3Ml Im
23-which of the following drugs are used in a mild allergic reaction?
a-isoproterol
b-meperidine hydrochloride
c-diphenhydramine hydrochlorid
d-propoxyphene
answer is c ,,,,,,propoxyphene is an analagesic
meperidine hydrochloride is an analagesic and sedative

isoproterol is a beta 2 adrenogenic antagonist

diphenhydramine hydrochloride is used to treat allergic reaction

OF THE FOLLOWING WHICH IS THE MOST IMPORTANT PRIOR TO PERFORMING DENTAL TREATMENT
INCLUDING GIVING LOCAL ANESTHESIA ?
A.VITAL SIGNS TO DETERMINE THE BASELINE IN THE EVENT OF AN EMERGENCY
B.VISUAL INSPECTION TO DETERMINE THE OVERALL PHYSICAL STATE OF THE PATIENT
C.DIALOGUE HISTORY , TO ENSURE THE PATIENT HAS ANSWERED TRUTHFULLY
D.M EDICAL HISTORY QUESTIONNAIRE , THOROUGHLY REVIEWED AND UPDATED
E.TREATMENT PLAN AND PATIENT CONSENT FORMS COMPLETED

2. WHY DO WE IMPLEMENT "IDEAL PATIENT POSITIONING"?


A.TO REDUCE THE RISK OF SYNCOPE
B.TO SPEED THE ONSET OF ANESTHESIA
C.TO KEEP THE SYRINGE OUT OF A PATIENT'S SIGHT
D.TO ENSURE CLINICIAN CAN OBTAIN A FIRM HAND REST
E.TO SLOW THE ABSORPTION OF ANESTHETIC INTO BLOODSTREAM

3.WHICH OF THE FOLLOWING IS THE BEST PATIENT POSITIONING WHILE DELIVERING LOCAL ANESTHETIC?
A.KNEES ABOVE SEA LEVEL
B.HEART AND HANDS PARALLEL TO THE FLOOR
C.HEAD AND HEART PARALLEL TO THE FLOOR
D.TOES ABOVE THE NOSE
E.HEAR AND HEART PARALLEL, TOES SLIGHTLY ELEVATED

4.THE MAIN REASON TO INJECT SLOWLY IS TO ACCOMPLISH WHICH OF THE FOLLOWING ?


A.M AKE THE INJECTION AS COMFORTABLE AS POSSIBLE
B.NOT DISTURB THE HOMEOSTASIS OF THE TISSUES
C.LESSEN THE CHANCE OF ANESTHETIC OVERDOSE

207
208

D.E NSURE YOU ARE NOT IN A BLOOD VESSEL


E.LESSEN THE CHANCE OF A HEMATOMA DEVELOPING

5.THE NERVE THAT CAN BE INADVERTANTLY ANESTHETIZED DURING AN IA INJECTION IS WHAT?


A.FACIAL NERVE
B.TRIGEMINAL NERVE
C.ZYGOMATIC NERVE
D.M IDDLE MENINGEAL NERVE
E.BUCCAL NERVE

6.WHICH OF THE FOLLOWING INJECTION TYPES WILL PROVIDE THE LONGEST DURATION?
A.NERVE BLOCK
B.FIELD BLOCK
C.SUPRAPERIOSTEAL
D.INFILTRATION
E.INTRASSEPTAL

7.THE MAXILLARY DIVISION OF THE TRIGEMINAL NERVE TRAVELS ANTERIORLY AND DOWNWARD TO EXIT
THE CRANIUM THROUGH WHICH FORAMEN ?
AOVALE
B.SPINOSUM
C.M AGNUM
D.ROTUNDUM
E.SUPERIOR ORBITAL

8.THE VAZIRANI-AKINOSI MANDIBULAR TECHNIQUE IS RECOMMENDED FOR WHAT TYPE OF PATIENTS?


A.CHILDREN
B.LARGE BONED
C.REDUCED OPENING
D.E NLARGED TONGUE
E.AUTISTIC

9.PRESSURE SYRINGES ARE PRIMARILY DESIGNED TO DELIVER WHAT TYPE OF INJECTION ?


A.PALATAL
B.INTRAOSSEOUS
C.INTRASEPTAL
D.PERIODONTAL LIGAMENT
E.INCISIVE/MENTAL

10.I F A RIGHT-HANDED CLINICIAN CONTACTS BONE TOO SOON (ONE HALF PENETRATION DEPTH OR LESS)
ON THE LEFT IA NERVE BLOCK, WHAT SHOULD BE DONE ?
A.WITHDRAW AND SELECT A PENETRATION SITE MORE POSTERIOR
B.IMMEDIATELY REDIRECT NEEDLE/SYRINGE MORE LATERALLY

208
209

C.WITHDRAW SLIGHTLY , REDIRECT NEEDLE/SYRINGE MORE MEDIALLY


D.WITHDRAW AND SELECT A PENETRATION SITE MORE MEDIALLY
E.WITHDRAW SLIGHTLY , REDIRECT NEEDLE/SYRINGE MORE LATERALLY

11.W HAT IS THE RECCOMENDED AMOUNT OF ANESTHETIC DEPOSITED FOR THE GOW-GATES INJECTION
TECHNIQUE ?
A.1.5 ML
B.1.8 ML
C.1.2 ML
D.1.3 ML
E.1.0 ML

13. APPLYING TOPICAL FOR LONGER THAN INDICATED CAN RESULT IN SLOUGHING OF THE TISSUES - THIS
IS OFFICIALLY CALLED WHAT ?
A.E PITHELIAL DESQUAMATION
B.STERILE ABSCESS
C.CHEEK ROT
D.WHITE LACY STRIATIONS
E.EPITHELIAL DYSPLASIA
F.E PITHELIAL DYSPLASIA

14. PERSISTENT PARESTHESIA FOLLOWING A LOCAL ANESTHETIC INJECTION


A.IS ALWAYS PREVENTABLE
B.IS OFTEN CAUSED BY A NEEDLE PENETRATING THE CHEEK
C.IS ALWAYS REVERSIBLE
D.IS DESIRABLE
E.IS THE MOST COMMON ON THE TIP /SIDE OF THE TONGUE

Radiology

1-Intensifying screens should be used with films for:


a)Panorals
b)Occlusals
c)Bitewings
d)All the above

Answer is a eric whaite page 176

2-Angulation to be kept for IOPA of Maxillary second premolar?

Answe at 40 degrees eric and whaite page 104

209
210

3-X-Ray to determine Fractured Condyle of mandible? Reserved towen’s radiograpg

4-X-ray of choice to determine bone strength in maxilla or mandible?

CT and panoramic radiograaph

5-What type of film allows the entire dentition to be viewed on a single film?
a.Occlusal
b.Cephelometric
c.Panoramic
d. All of the above
Answer is c

6-WHAT TYPE OF DEVICE DOES DIGITAL RADIOGRAPHY USE TO RECORD IMAGES TAKEN OF THE PATIENT 'S
TEETH ?
A . S TANDARD X- RAY FILM, WITH SCAN
B. E LECTRONIC SENSOR
C. MRI SENSOR
D . ALL OF THE ABOVE
Answer electronic sensors

7-WHICH OF THE FOLLOWING INFLICTS THE MOST RADIATION ON A PATIENT?


A . FMX
B. P ANORAMIC
C. B ITEWING SERIES
D . PA
Answer is a

8-WHY DOES DIGITAL RADIOGRAPHY REQUIRE LESS RADIATION THAN TRADITIONAL X-RAYS?
A . THE X -RAY BEAMS ARE MORE POWERFUL .
B. I T USES LOWER WAVE -LENGTH RADIATION .
C. S ENSORS ARE MORE SENSITIVE .
D . ALL OF THE ABOVE
Answer is b

9-QUALITY OF X RAY BEAM IS GOVERNED BY


A] KVP

210
211

B]MAS
C] FILAMENT CURRENT
D] LENGTH OF X-RAY TUBE
Answer is a

10-The following are the possible X-ray causes or faults why film is too dark except:

a.Using old film stock


b. Developer solution too concentrated
c. Excessive thickness of patients tissues
d.faulty of dark room due to leakage or stray of light
answer isc

11.IN DENTAL RADIOLOGY , WHAT IS JUSTIFICATION ?


A) ENSURING THE X RAY SET GIVES THE LOWEST DOSE THAT IS REASONABLY PRACTICABLE
B) ENSURING THE BENEFITS OF THE X RAY WILL OUTWEIGH THE RISK
C) EXAMINING THE PATIENT BEFORE PRESCRIBING X RAY

D )SCREEING OF DENTAL DIEASES THAT ARE LIKELY TO BE PRESENT

ANSWER IS B

12-FOR A PATIENT WITH A HIGH CARIES RISK BITEWINGS MAY BE INDICATED


A) 24 MONTHS INTERVALS
B) 6 MONTHS INTERVALS
C) 12 MONTHS INTERVALS
D) 18 MONTHS INTERVALS

ANSWER IS B

13-WHAT IS RADIATION EXPOSURE WHEN TAKING AN OPG


A) 0.001 MSV
B)0.01MSV
C) 0.1 MSV
D)1.0 MSV
ANSWER IS B

211
212

14-WHAT X RAY MAY BE APPROPRIATE WHEN ASSESSING CYSTS IN THE MANDIBLE


A) PA MANDIBLE
B) AP MANDIBLE
C) LOWER OCCLUSAL
D) REVERSE TOWNES
ANSWER IS C

15-IN ORDER TO LIMIT THE DOSE FOR A PERIAPICAL RADIOGRAPH


A) USE A LOW SPEED FILM
B) USE A LEAD APRON
C) USE A RECTANGULAR COLLIMATOR
D) USE BISECTING ANGLE TECHNIQUEIN ORDER TO LIMIT THE DOSE FOR A PERIAPICAL RADIOGRAPH
Answer is c

16-What are the Radiographs used for the following:

1.ATYPICAL FACIAL PAIN


2. TRIGEMINAL NEURALGIA
3. DENTAL ABSCESS AND/OR FACIAL SWELLING
4. FRACTURES OF THE ZYGOMATIC ARCHES AND PATHOLOGY IN THE PALATE AND SKULL BASE.
5. FOR ORTHODONTIC PURPOSES
6. TO ASSESS FRACTURES AND PATHOLOGY IN THE POSTERIOR MANDIBLE AND CONDYLES.
7. TO ASSESS CONDYLES
8. USED TO DETECT PATHOLOGY AND BURIED OR SUPERNUMERARY TEETH IN THE PALATE.
9. USEFUL FOR IMAGING PATHOLOGY , DENTOALVEOLAR FRACTURES AND ROOTS DISPLACED INTO THE
ANTRUM.
10. USED TO IMAGE THE CROWNS OF THE TEETH IN BOTH ARCHES AND USUALLY ALLOWS THE ALVEOLAR
BONE LEVELS TO BE ASSESSED
ANSWERS
1. SUGGEST A PANORAMIC RADIOGRAPH OF THE AFFECTED SIDE. AN MRI SCAN MAY BE INDICATED IF
THERE IS NO RESPONSE TO MEDICAL TREATMENT
2. AN MRI SCAN IS INDICATED TO CHECK FOR A NEOPLASM ALONG THE COURSE OF THE NERVE OR FOR
EVIDENCE OF MULTIPLE SCLEROSIS (DEMYELINATION ) IN YOUNGER PATIENTS (UNDER 40)
3.PERI APICALVIEWOR SECTIONAL PANORAMIC RADIOGRAPH
4. SUBMENTOVERTEX
5. LATERAL CEPHALOMETRIC RADIOGRAPH
6. PA – P OSTERIOR ANTERIOR
7. REVERSES TOWNE’S
8. UPPER STANDARD OCCLUSAL
9. UPPER OBLIQUE OCCLUSAL
10. BITEWINGS

212
213

17-method of detection of proximal caries?


Bitewings

18-Which Xray is needed for implants?


Cone beam CT and OPG

19-Position of unerupted canine , which is the best xray technique to identify its position? –
Vertical parallax techinique(opg+upper anterior occlusal radiograpg) and horizontal
parallex technique(opg+ periapical radiograpg)

20-X-ray with defect x-ray placed on the wrong side (there were foil impressions)?what is
the cause

21-Patient comes to you after 2 years of not visiting a dentist, which radiograph will you
take?
Bite-wings

22-13 years old child, decidious teeth to be extracted , You want to check permanent germs
which radiograph??
A)DPT
B)Bimolar
C)Both
D)none
Answer is a

23-What is the X-ray to be done for a patient with Flattened Zygoma and bilateral blackened
eyes ??

Submentalvertix radiograph

24-what is the X ray defect in which there are fixer splashes??

Light or radio-opique dots on the film due to fixer being flashed causing emulsion before the
film was place in the developer

25-What is Penumbra Effect??


The focal spot should ideally be a point source to reduce the blurring of an image.

26-Radiogaphic fault- fim blackening least likely cause??

Thin patient tissue,,,check answer

27-What is JUSTIFICATION?

213
214

Benefits of taking the radiograph should outweigh the risks and taking an x-ray should be
based on previous radiographs,the reasons of taking aradiograph,diagnostic benefit
,radiation risk and alternative technique that might achieve the same porpuse

28-What is optimisation??

Optimisation means ensuring that the dose a patient receives is the minimum required to
obtain a useful radiograph

29-X-RAY EQUIPMENT MUST BE SERVICED REGULARLY AND RADIATION SAFETY TESTS CARRIED OUT AT
LEAST??
A)EVERY YEAR
B)EVERY 6 MONTHS
C)EVERY 3 YEARS
D)EVERY 2 YEARS
3 YEARS ANSWER IS C

30-Radigraphs in the assessment of following facial bones??


A)dentoalveolar fracture
B)mandibular fracture
C)Zygomatic fracture
D)Le Fort Fracture
Options are:
i)Occipetomental and lateral skull views , followed by CT scan
ii)A periapical vies and Upper oblique occlusal view
iii)Occipetomental view and submentovertex view
iv)Panoramic and posterior anterior view
a-2,b-4,c-3,d-1

31.which one of the following annual dose limits is the correct lonising Radiation
Regulations (IRR) 1999 limit
A) general public-2 mSv
B) non classified workers-2mSv
C) classified workers-20 mSV
D) non classified workers 20mSv
Answer is c

32.the correct order of the stages of processing radiographic film is


A) developing, washing, fixing, washing,drying
B) fixation, washing, developing, washing, drying
C) washing, developing, washing, fixation,drying
D) washing, fixing,washing, developing, drying

214
215

Answer is a

33.what type of filter is used in x Ray machines


A) tungsten
B) tin
C) copper
D) aluminium
Answer is d
34. Which part of the x Ray machine performs the following function:it removes the
peripheral x- Ray, therefore minimising the dose to the patient
A) collimator
B) filter
C) imagine intensifier
D) generator
Answer is a

35. What is the optimal shape for a collimator


A) round
B) rectangular
C) square
D) hexagonal.
Answer is b

36-Which of the following best describes the appearance of bone on a radiograph?


A- cortical bone appears radiopaque, cancellous bone appears radiolucent
B- cortical bone appears radiolucent, cancelleous bone appears radiopaque
C- all bones appear radiolucent
D- all bones appear radiopaque
Answer is d

37- The inverted Y landmark is composed of which two structures?


A- junction of the right and left nasal cavities
B- Inferior border of the nasal cavity and anterior border of maxillary sinus
C- floor of orbit and floor of maxillary sinus
D- floor of orbit and anterior border of maxillary sinus

38. Which of these structures appear radiopaque?


A-Maxillary sinus
B-nasal fossa
C-maxillary tuberosity
D-mental foramen
Answer is c

39. Which is the proper method for mouting radiographs?


A-radiographs should be mounted as if you were facing the patient
B- radiographs should be mounted as if you were looking out from patients toungue

215
216

C-radiographs should be mounted with dot toward the distal


D-radiographs should be mounted with dot toward the mesial
Answer is a

40-Who is allowed to make an initial interpretation of a dental radiograph?


A- the patient
B- the assistant
C- Trained assistant who passes DANB Radiation exam
D- None of the above
Answer is c

41-What it is radiograph of mixed dentition identify age and abnormalities ?


OPG

42-A STEP-WEDGE TEST


A - USES AN ALUMINIUM DISC
B- SHOULD IDEALLY BE DONE AT THE START OF EVERY DAY
C- WILL SHOW THE DIFFERENCE BETWEEN A DEVELOPER FAULT AND AN EXPOSURE FAULT
D - IS USEFUL FOR IDENTIFYING PANORAMIC RADIOGRAPH FAULTS
ANSWER IS A

43-WHICH OF THESE ARE NOT PART OF THE CRITICAL EXAMINATION ?


A - LOCATION OF THE CONTROLLED ZONE
B- EXAMINING A TEST RADIOGRAPH TAKEN ON A PATIENT
C- POSITION OF WARNING SIGNS
D - ACCESS TO THE EXPOSURE CONTROL
ANSWER IS B

44-AFTER INSTALLATION OF THE EQUIPMENT, IT MUST UNDERGO


A - A CRITICAL FUNCTION TEST
B- A COMPLETE INSTALLATION TEST
C-A FULL APPROVAL TEST
D - A DETAILED ACCEPTANCE TEST
ANSWE D

45-CCD (AS IN CCD SENSOR) STANDS FOR


A - CHARGE -COUPLED D IGITAL
B- COUNTER- CHARGED DEVICE
C- CHARGE -COUPLED DEVICE
D - CHARGE -CARRYING DISC
ANSWER IS C

216
217

46-What x-Ray for a patient with flattened zygoma & bilateral black eyes?
Occiptomental and submentalvertix

47-IN THE RADIOGRAPHIC PRINCIPLE , "SLOB RULE, "SAME SIDE MOVEMENT OF THE OBJECT IS ______
AND THE OPPSITE SIDE MOVEMENT OF THE OBJECT IS ______. W HAT ARE THE MISSING WORDS IN THE
SAME SEQUENCE.
A.LATERAL AND BUCCAL
B.LINGUAL AND BUCCAL
C.LINGUAL AND BOTH SIDES
D.LATERAL AND BOTH SIDES
ANSWER B

48- An example of an electro-magnetic radiation is


A.Alpha rays
B.Beta rays
C.Cathode rays
D.Gamma rays
E.None of the above.
Answer is d

49-The "rotating anode" design in an x-ray tube limits the amount of


A. lighting at anode
B. heat production at anode
C. electron production at cathode
D. speed of electrons from cathode to anode
E. x-rays that are produced at the anode
Answer is b

50-The agent responsible for prevention of oxidation of developer by air :


1. Sodium bisulphide.
2. Sodium trisulphite.
3. Sodium sulphite
4. Potassium sulphite
Answer is 3

51-Tooth contact-point overlap is


a- Caused by incorrect vertical angulation of the tube.
b- Unavoidable with crowded arches
c- Caused by the patient moving during the exposure
d-Useful for identifying occlusal caries on bitewings
answer is b

52-The Diagnosic Reference Level


a- Is the average dose of x-rays required for a particular view
b-Is set by the Health and Safety Executive (in the UK)
c- Helps identifies practices that use a too-high dose of x-rays for an exposure

217
218

d- Is used to ensure developing techniques are

53-3 years old child , baby teeth to be extracted . You want to check permanent germs .
Which radiograph ?
Answer is bimolar

54-The sharpness of the image is determined to a large extent by the


A.shape of the object
B.The focal spot size
C.the make of the x-ray machine
D.the distance between the anode and cathode
E.None of the above
Answer is b

55-radiation causes cell death by


a]charring of nucleoproteins
b]destroying thier mitochondria
c]ionization
d]disruption of cytosol
answer is c

56-What is the best radiography for TMJ?


Panoramic tmj radiography, transpharyngeal radiography, multidirectional tomography, ct,
Mr, reverse Townes

57-What it is indication of periapical radiography ?


Detection of apical infection, assessment of the periodontal status, assessment and position
of unerupted teeth, assessment of root morphology, after trauma to teeth and alveolar
bone,

58-What is clearing time ?


It is how long it takes to remove the unsensitized silver halide crystal from films and it
averages 2-4 mins. In a 20° C

59- A radioluscency in the angle of the mandible located beneath the inferior dental canal ?
is most likely- staphne’s idiopathic bone cavity, which is a monuclar on the lingual aspect of
the mandibile near the lower border frequently said to contain aberrant salivary gland EW
PAGE 358

60-- On radiograph , a mutilocular radioluscency in the region of the angle/ramus of the


mandible is most likely?

a)ameloblastoma
(b)pleomorphic adenoma
(c) staphne’s idiopathic bone cavity
Answer is a EW page 342
develops from remnant of epithelium of dental lamina or enamel organ

218
219

61-order to determine the working length because of limited mouth opening but you have
a pre-op assessment radiograph what will you do
a)take a dental panoramic radiograph
b)use an electronic apex locator
c)use tactile means to determine the apical stop

62- Radiology secondary radiation for how long – 2, 4, 8, 16, 32 hours???

63-Swiss Cheese pattern is seen in what condition???


Adenoid cystic cancinoma

64-image of radicular cyst?


It usually occurs in 20-50 years old

65-Match the following with they correct option given below??

A) Elongated image
B) Foreshortened Image
C) Double exposure
D) Conning off

1) the same film packet was used for two different projections
2) the vertical angulation of x ray tube head was too shallow
3) the vertical angulation of the x ray tubehead was too steep
4) the x ray tubehead was placed too far posteriorly so that the anterior part of the film was
not exposed
D-4,c-1,a-2,b-3

63-) identify the radiolucency? it develops from what part of the tooth?? and usually at
what age???

219
220

Dentyrogerous cyst,develop from the remenanat of the reduced enamel epithilium after the
tooth is formed and is occurs in young adult

64-.child with decay in the upper and lower ipsilateral molars, what radiograph would you
prescribe?

a-vertical bitewing,

b- horizontal bitewing,

c-bimolar,

d-iopas

answer is [ horizontal bitewings]

65- Best radiological view or investigations Showing :

• successor teeth in a young child---[panoramic ]

• caries in 3 year old---[ bitewing ]

66-best x-ray to locate the upper canine that is not erupted?periapical tube shift technique
or opg with occlusal?

If palbale tube shift technique if not occlual

220
221

Peadiatric
1-Which one of the following is not a good method of child behaviour management
A) behaviour shaping
B) desensitisation
C) tell, show, do
D) sensitisation
Answer is d

2. What teeth should a 9 year old child have in a given quadrant


A) 12CE6
B) 1BCD6
C)12CDE
D) 1234e6
Answer is a

3.at what age should you try to locate the upper canines
A) 6 years
B)8 years
C) 10 years
D) 13 years
Answer is c

4.an injury to the supporting tissue of the tooth without displacement of the tooth defines
which type of injury
A) concussion
B) location
C) subluxation
D) intrusion
Answer is a

5.Which condition is associated with mulberry molars in children


A) autism
B) Down's syndrome
C) osgood syndrome
D) syphilis
Answer is d

6-Which of the following are typical consequence of dental crowding , assuming no primary
teeth has been lost prematurely ?
A. Overlapping of lower incisors
B. Palatal displacement of upper canines
C. Impaction of 15 and 25 between first premolars and first molars.
D. Mesial tipping of 16 and 26
E. Rotation of 16 and 26

221
222

Over lapping of lower insciors

7-A health 6 year old child presents with carious maxillary second primary molar with a
necrotic pulp. Which treatment would be preferred ?
- extraction
- indirect pulp treatment
- pulpotomy
- pulpectomy
- antibiotic coverage
Pulpectomy

8-A child had sustained a traumatic exposure of primary central incisors , he presents to you
for treatment two days after the injury . Which of the following should be considered ?
A. Pulpotomy and Ca( OH)2
B. Pulpotomy and formocresol
C. Direct pulp capping
D. Pulpectomy (RCT)
Answer is d

9-A 10 year old boy presents with small greyish white lesion surrounded by a red halos on
the soft palate and tonsillar pillars, small vesicles are found . He has fever and pain in ear.
What it is probable diagnosis?
Herpengenia

10-Loss of tooth in mixed dentition affects the:


A. Same quadrant
B. The relevant jaw
C. The whole mouth
D. The relevant quadrant
Answer is d

11- Upper right c erupt at what age


A) 6-9months
B) 9-12 months
C) 15-18 months
D) 18-21 months
Answer is d this is different to bb it says 16-18month

12. The loosening of a tooth within its socket without any displacement is
A) avulsion
B) extrusion
C) luxation
D) subluxation
Answer is d

222
223

13.describing the treatment which is going to be performed, then visually demonstrating it,
followed by performing the treatment on the child. What is this type of behaviour
management
A) desensitisation
B) reinforcements
C) show, tell, do
D) tell, show, do
Answer is d

14.an enamel- lined invagination sometimes present on the palatal surface of upper incisor
is
A) palatal invagination
B) dens in dente
C) odontome
D) Cingular pit
Answer is b

15. Pointing the brush apically, at 45 degree to long axis of the teeth vibrate the brush, not
changing the position of the bristles. What brushing technic is this
A) Bass
B) Fones
C) Stillman
D) Charters
Answer is a

16- Primary teeth are more constricted at the cervical 3rd than their permanent
counterparts. Pulp chambers are comparatively smaller in primary teeth.
A]Both statements are true.
B]Both statements are false.
C]The first statement is true; the second is false.
Answer is c

17- Calcified teeth at birth


a) all deciduous and permanent incisors
b) all primary teeth only
c) all primary teeth and first permanent molars
d) all deciduous teeth and all permanent molars
answer is c

18- Child lived in a fluoridated area with 4 ppm till 8 years then moved in non fluoridated
area what teeth can show fluorosis?
a- all teeth
b- Central incisor
c-All teeth except third molars

answer is c

223
224

19- is a process in which a single tooth germ splits or shows an


attempt at splitting to form two completely or partially separated crowns.
• concrescence
• gemination
• fusion
• dens in dente
Answer is germination

20-what is the ferric acid concentration?


15.5%

21-what is the most common cause of missing permenat upper central tooth?
Supernumery tooth (mesodense)

22-hypodontia occurs in what condition?


Hereditary gingival fibromatosis,down syndrom,gardner
syndrome,hypothyrodism,cleidocranial dystosis,ricktes

23-what is the cause of the presence of a white line on the enamel?


a-Amelogensis imperfecta
b-Dentinogensis imperfecta
c-Flurosis
d-Enamel hypoplasia

Answer is enamel hypoplasia

24-what is the main cause of staining in the uk?


a-flurosis
b-amelogensis imperfecta
c-dentinogenesis imperfecta
d-tetracyclin

ansewr is a

25-material used for non-vital pulpotomy?


Beechwood cresol

26-material used for primary teeth pulpectomy?


ZOE, non-setthing Ca(OH)2 or idoform

27-indication for apexogensis?


Permenant tooth ,vital and incomplete root formation,
Apexogensis:is the removal of only 3-4 mm of the coronal pulp and application non-setting
calcium hydroxide and semi –perminant restoration to mantanain the formation of the
calcified barrier to allow the development of the radicular pulp and closure of the apex
success rate is over 80% if completed 24 hours of injury
Reference master dentistery 2 page 182

224
225

28- indication of apexification?


Used for none-vital ,incompelet root formation
Pulpectomoy of the tooth then application of none-setting calicium hydroxide which should
be replace after 1-2 weeks then 3 monthly,average time for stop formation is one year.

29-what is cvek pulpotomy ?


It is a partial pulputomy where only 2-4 of pulp is removed until health pulp is reached this
is used for vital teeth with exposure of more than 1mm and injury time more than 24 hours,
non setting calcium hydroxide is used and cavity is sealed with GIC.

30-what is material is used for vital pulpotomy?


Ferric sulfate 15.5%

31-A dental fracture that is exquisitely tender and is bleeding from a central
pinkish area is most likely an:
A]Ellis I dental fracture
B]Ellis II dental fracture
C]Ellis III dental fracture
D]Ellis IV dental fracture
Answer is c
Ellis Classification of Dental Fractures

Ellis I
Includes crown fractures that extend only through the enamel.
Teeth are usually nontender, and without visible color change, but have rough edges.
Ellis II
Fractures that involve the enamel and dentin layers.
Teeth are typically sensitive to cold, hot, touch and/or air exposure. A yellow layer of dentin
may be visible on examination
Ellis III
Involve the enamel, dentin, and pulp layers.
Teeth are extremely sensitive, and have a visible area of pink, red, or even blood at the
center of the tooth.

32-an 8 year old underwent trauma while playing football. on complete radiographic and
clinical examination it was found that there is lateral displacement and fractured alveolar
process. recommended time for splinting such tooth.
a 1-2 weeks
b 2-4 weeks
c 4-6 weeks'
d 6-8 weeks
answer is c

33-Movement done during extraction of E’s


a-buccal and lingual

225
226

b-rotational
c-both
answer is a

34-First sign after the formocresol pulpotomy failure is


A.internal resorption
B. pain
C.external resorption
D. bleeding
Answer is internal bleeding

35-In permanent incisors with open apices:


a. Pulpotomy is preferred to pulp cap for pulpal
exposures
b. Pulpotomy medicament of choice is calcium
hydroxide
c. Pulpotomy enables radicular development with
vital pulp tissue
d. Non-vital pulpectomy with non-setting calcium
hydroxide stimulates root-end closure
e. Non-vital pulpectomy root-end closure occurs
in 6 months
answer is a,b,c,d
apulpotomy is preferred to pulp capping in permenant teeth with open apices because
pulpotomy provides apexogensis which help in the closure of the apex of the tooth root
dycal is medicament of choice premenant pulpotomy and ferric sulphite is medicament of
choice for pulpotomy of primary teeth
none-vital pulpectomy root end clouser occurs with 9 months to one year

36-Fluoride supplementation should be given:


a. Systemically to all patients with special needs
b. As a mouthwash to a 5-year-old
c. From birth
d. In only one systemic form of delivery
e. In topical form to all patients with orthodontic
appliances
answer is e and d

37-A 9 yr old child had trauma to TMJ and


ankylosis. He had undergone arthroplasty. Which
of the following graft is best suited to fill the
gap?
a) Allograft
b) Xenograft
c) Costochondral graft
d) Alloplastic
answer is c

226
227

Autogenous costochondral interpositional graft is an


effective procedure for treatment of TMJ ankylosis by
articular reconstruction technique; especially if it is associated with mandibular hypoplasia.

38-which one of the following is the cause of dilacerations?


a) trauma to the tooth germ during root development
b) abnormal displacement of tooth germ during root development
c) abnormal proliferation of enamel epithelium during tooth development
d) abnormal displacement of amelobalsts during tooth formation
answer is a

39-most common type of trauma injury in children under the age of 5 is?
a-crown fracture
b-extrusion
c-luxation
d-intrusion
answer is c,,,,pink book page 98 says intrision is the most common injury in primary teeth

40.At which age will a permanent maxillary central incisor have a completed and closed
apex
A)10 years
B) 9 years
C)11 years
D) 7 years
a
41- A town has water with a fluoride ion level of 0.4 ppm. What fluoride supplementation is
necessary for 2- year old child
A) 0.25 mg/ day
B) 0.5 mg/ day
C) 1 mg/ day
D) none
Answer is d

42.Which but should be used to prepare a class II various lesion on a deciduous molar?
A) 330 bur
B) 34 bur
C) #6 round bur
D)558 bur
a
43. Which type of patient will least benefit from the tel- show- do technique
A)adolescent
B)mentally challenged child
C) adult with trisomy 21
D) apprehensive child
Answer is adeloscent because they have a higher intellgence level and have a wider
inderstanding

227
228

44. A patient has enamel hypoplasia near the Invisalign edges of all permanent incisors and
cuspids, except for the maxillary lateral incisor. At what age did the systemic problem
begin?
A) birth to 1 year of age
B) 1-2 year of age
C) before birth
D) 2-3 years of age
Answer is a

45- how old is the patient most likely to have a trauma that damages tooth germ of
permanent incisor?
Answer is less than 4 years old

46- sealants for 1st molars best age?


Striaght after tooth erupt 6-7 years old

47-what is maximun time of avulsion tooth extra- orally which can be re-implanted?
within an 1 hour is the best prognosis for closed apecies and 30 minute for open apex.

48-what is the best Meduim in which an avulsed tooth is stored?


Saline> cold milk> Water> air.
49-what is the length of time Delayed reimplantation can take place?
It can be within 24 hours.

50-how do you treat a tooth with delayed reimplantation?


RCT with GP emerse in sodume fluoride 2.4% for 20 minutes then reimplant and splint for 6
weeks

51- Child with several caries, anxious. Which anaesthetic to use?


Nitrous oxide

52-what is the indication for stainless steel crown?


* Most durable restoration for primary molars.
* Badly broken down primary molars
* After pulp therapy in primary molars
* In secondary molars as an interim restoration where crowns are required but the patient
is too young
* In developmental anomalies
* Severe tooth loss due to bruxism/erosion
* As a temporary coverage
*if cavity includes more than two surfaces or two surfaces and a cusp

53-picture of a tooth with rampant caries, how would you manage it?
Dietry analysis,OHI ,teeth restoration
Treatment for early stage include proximal discing of the tooth and topical fluoride
application and in advance stage use of strip down technique which uses a celluloid crown
with light cure composite to restore the tooth

228
229

54- rampant caries what would be first step in the treatment of ?


A-dietary analysis,
b-no treatment,
c-extraction,
d-pain relief,
e-restoration of lesions.
Answer is d

55-management of trauma, pulp exposure in a 7 yr old , central permanent incisor, over 72


hours?
As child is 7 years old apex of the root is still open x-ray is required to conform and see if
accompanied with any fractures in the root . check vitality if tooth is vital then treatment is
by pulutomy because pulp has been exposed for more than 24 hour. By the use of calicium
hydroxide to produce apexogensis to form a calcified barrier arounfd the apex.if non vital
then pulpectomy and or apexification.

56-. management of palatally intruded tooth some hours ago?[ immediate aims to prevent
external bone resorption. Periapical x ray to determine if the tooth has got an open or
closed apex. For open apex make sure the pulp is vital and let it erupt naturally and if it
doesn’t then orthodontic intervention may be required.
Peri Apical Xray if open apex leave it to re erupt if closed reposition it.]

57-Tooth at the level of gingival ,in two month old


child without any radiographic evidence of
roots,treatment is
a. surgically remove it under La
b. Let it remain as it is for future tooth and root
formation
c. preserve it and reassure the parent
d. extract if mobile under GA
answer is c

58-Avulsed teeth replanted without pulpectomy usually exhibits


a- surface resorption
b- revascularization
c- inflammatory resorption
d-ankylosis
answer is c

59-external resorption may be found in


a-diabetes
b-leukaemia
c-hypoparathyroidism
d-none of the above
answer is b

229
230

60-Comparing dental trauma in boys and girls


a-the rate of trauma is increasing more rapidly in girls
b-injuries are common in boys rare in girls
c-boys have more injuries in maxilla and girls in mandible
d-girls have more injuries
answer is a

61-Third degree tetracycline staining is


a-dark gray or blue with banding
b- dark yellow or blue
c-dark gray
d-too dark to bleach
answer is a

62-Inflammatory resorption is the mechanism of


a-sequestration of bone
b-scar tissue formation
c-eliminating infected calcified tissues
d-none of the above
answer is d

63-Best way to examine midline discrepancy with respect to upper incisors in 10 year old
child?
1. Alignment incisor midline to labial frenum
2. Seeing alignment of incisor midline to nose
Answer is 2

64-The most common bone tumour that occurs in children is


A. Osteosarcoma
B. Ewing's sarcome
C. Metastatic carcinoma
D. Multiple myeloma
Answer is b

65-In a 9 yr old child, an hour old Ellis class 3 fracture with a small pulp exposure. treatment
is:
a) direct pulp capping
b) pulpotomy
c) pulpectomy
d) apexification
answer is a

66-An advantage of the rubber dam is that it


a. Muffles any screams the child may emit
b. Aids in the management of child behavior
c. Enables the operator to salvage silver scraps
d. Reduces the necessity for a carefully carved cervical margin

230
231

answer is b

67-when is the first appointment of children recommended??


A)6 months
B)3 months
C)in first 15 days of birth
Answer is a

68-match the following with the options given below???


1)Fleeting pain on hot or cold or sweet stimuli
2)longer lasting pain on hot or cold or sweet stimuli
3)Spontaneous pain with no initiating factor
4)pain on biting and pressure and or swelling and tenderness of adjacent tissues mobility
Options are:
A)irreversible pulpitis
B)reversible pulpitis
C)acute periradicular periodontitis
1-b,2-a,3-a,4-c

69-what are the three main types of amelogenesis imperfecta?


Hypoplasia
Hypo mineralization
hypocalcification

70-How do you manage extrinsic stains caused by chlorhexidine mouthwash?

I think by micro-abrasion

71- which of the following are indications for stainless steel crown?
A)Badly broken down primary molar
B)After pulp therapy primary molars
C)developmental anomalies
D)all of the above
E)A & B
Answer is d
72-what is the dental recall interval for an adult with low caries index?
24 months

73-an 8 year old boy presented with pain for three days that keept him awake on
examination you see a grossly carious lower left 6 and associated buccal swelling .whic of
the following is the most appropriate to give immediate relief of his pain?
a-extraction of the LL6
b-gentaly excavate the caries and obtain drainage
c-give antibiotic
d-incise the swelling
e-refer for GA
answer is d

231
232

74- cement used in apexification?


Non setting calicium hydroxide and MTA

75-. first line treatment patient 5years old with an abscess on buccal side of lower first
temporary molar ?
drainage first and then antibiotics but only in cases where the infection is not localized and
patient has systemic symptoms like fever malaise

76- antibiotic for child allergic to penicillin that can’t swallow pills ?
azithromycin suspension

77-. 8 year old, high caries rate. Prescribe fluoride content of toothpaste?
• 500 ppm
• 800 ppm
• 1000 ppm
• 1350 ppm
Answer is 1350 ppm

78- How to locate the working length in a tooth with open apex?
conventional periapical radiograph which is 1-2 mm short of the radiographic apex

79-What’s the most effective method?


a-Tell the parents not to give sweets for the child,
b-dentist tells the child which are good and bad foods,
c-nurse talk to the child,
d-send the child to a dietician,
answer is c, nurse talks to the child

80-.%of children having plaque on their teeth?


72%

81-.a boy has proximal cavities on some of his teeth. Which one of following would you do?
• -give him a new toothbrush
• -ask him to add sugar to his drink
• Show him how to floss
Answer is show him how to floss

82- Ca(oh)2 setting and non-setting-indications?


Setting is used for lining and sealing the canal, pulp capping.
Non setting is for apexification and intra canal medicament

83- Best concentration of fluoride tooth paste for a 4 year old?


1000ppm fluoride tooth paste pea size

84- Pulpotomy which is the best material to use opt: formocresol, ferric sulfate etc.ca oh2
wasn’t an option----[ferric sulphate]

232
233

85- Extrinsic and Intrinsic Staining of teeth:


• Fluorosis---- white opacities which becomes brown later
• Tetracycline----blue or brown
• ,bilirrubin staining----greenish
• Congenital porphyria---- red
• Pulp necrosis--- grey,
• Iron ---- black stains,
• minocycline ---- black or brown
• Poor Oral Hygiene with chromogenic bacteria ---- green orange
(Pink book is pg 72 and masters dentistry is Pg 172 table 15)

86- In which of following is more likely to maintain vitality?


a-concussion,
b-subluxation,
c-intrusive luxation,
d-avulsion
answer is a,concussion

87- Commonest cause of enamel staining?


Extrinsic cause Food / smoking, Intrinsic is Caries

88-.l.a.-maximum dose for child 5 years with 20 kg?


weight of the person x 4.4/ 20 = 4.4 and then divide that the 2.2 mg lignocaine in a
cartridge. So 4.4/2.2 = 2. So the max dose is 2 cartridges

89- Eruption dates of lower canine and lower second molar?


lower canine 9- 10 years and lower second molar is 11-13 years

90- Management of palataly intruded tooth some hours ago?i think this question is
incomplet
I think extraction if it is a decideous tooth because it affects the tooth germ.if permenant
wait and watch it should erupt within 1-6 month if open apex or surgical reposition and
slpint for 1-2 weeks

91-. What would cause red or brown discoloration in a primary teeth? [Congenital
porphyria]

92- 1. first stage in management of rampant caries


a.excavation and temporization
b.[prevention and stabilization]---- http://jsn.sagepub.com/content/21/5/272.abstract

93-date of eruption of permanent maxillary central incisor?


a—6-7
b-7-8
c-8-9
answer is b

233
234

94- eruption date of permanent mandibular second molar?


a-11-13
b-10-11
c-10-12
answer is a

95- eruption date of permanent maxillary second premolar?


a-9-10
b-10-12
c-12-13
answer is b

96-date of calcification of permanent maxillary central incisor?


a-12-16 weeks in utero
b-3-4 years
c-3-4months
answer is c

97- date of calcification of permanent mandibular first molar?


a-20 weeks utero
b-at birth
c-3-4 months
answer is b

98- safest material for vital pulpotomy for primary teeth?


ferrous sulphate

99-material of choice for vital pulpotomy in permenant dentition?


Calcium hydroxide

100- Eruption dates for permanent teeth?


Eruption dates of milk teeth?

101- trauma-pulp exposure in a 7 year old, central permanent incisor-over 72 hrs?


Pulpectomy

102-the deciduous canine associated gap is the


A) eruption follicle
B) leeway space
C) primate space
D) canine groove
Answer is c

103.supernumary teeth in the deciduous dentition are followed by permanent


supernumaries in the same place in how many cases?
A) 5-15%
B) 15-35%

234
235

C)35-55%
D)50-70%
Answer is c

104. On average, at what ages are the two main periods of activity regarding eruption of
the permanent dentition?
A) 6-8 and11-12
B) 5-6 and 11-13
C) 6-8 and 12-13
D) 5-6 and 11-12
Answer is a

105-which teeth erupted in the first period?


A) lower incisor
B) canines
C) premolars
D) first molar
a/d

106-true or false? Spacing in the deciduous dentition is


A) normal
B)often increases with jaw growth
C) often mesial to the upper canine
D)often mesial to the lower canine
a/c

107-Trauma to tooth with closed apex?


Check the tooth if tooth is vital restore if not RTC or extraction

108-Best analgesic for children?


a-2% lignocaine
b-Prilocaine 3%
c-articaine
answer is a

109-a 3yr old child present with necrotic pulp in decidous second molar the line of
treatment should be?
1). partial pulpectomy
2). antibiotics
3). relieve acute sympotoms
4). extract tooth
Answer is partial pulpectomy/ I think answer is 3

Law and ethics

235
236

1-A type of consent where a patient will consent to part of a treatment plan but withhold
consent for treatment that may become necessary so that the procedure can be given further
consideration?
Restricted consent

2-Dental hygienists and dental therapists are permitted to treat patients under conscious
sedation or general anaesthesia ?
True
False
Answer is true

3-for how long soup need to be on hands during hand washing?


15 seconds

4-The preliminary proceeding committee does not have the power to suspend dentists?
True correct
False

5-The indemnity cover is requirement for general practitioners who might be involved in
claims with patients ?
True
False

6-Obtaining consent to treatment can be delegated to the staff. True or False? true

7-Can dental hygienist give dental nerve block under supervision of dentist? Also can she/he
take an impression!?
Answer is yes they cam

8-The GDC will introduce a new revalidation scheme includes a portfolio of evidence for
practitioners over the 5 years to stay on the register ?
A. True
B. False

9- number of CPD hours for Dentist??


A)350
B)100
C)250
D)500
Answer is c

10-What happens if dentist does not pay his annual retention fee on time??
A)license is cancelled
B)the dentist has to pay fine
C)the Registrar must remove the name of the dentist
D)Dentist degree will be cancelled
Answer is c

11-which of the following is the major function of GDC??


A)Regulating a dental practice

236
237

B)maintaining a register of dentists


Answer is b

12-A 15 year old girl in boarding school comes to your surgery for an extraction which of the
following cannot give consent on her behalf??

A)her grandfather with legal guardianship


B)her biological father who has separated from her mother
C)the girl herself
D)the older sister

13-The CQC regulates which of the following??


A)Hospitals
B)dental practices
C)care homes
D)home care services
E)all of the above
Answer is e

14-true or false?the Woolf reforms for civil justice


A) give the courts, rather than the parties legal advisers, control of the timetable
B) are designed to save expense
C) affect all civil courts in the UK
D)allocated an appropriate time for each case
Answer is a,b,d

15-it is illegal to discriminate against people because of their


A) race
B) dietary habits
C) sex
D) age
A,c,d

16-the dentist can use the following in his defence


A) the Bolam principle/ test
B) that the incident occurred 10 years ago
C)the fact that patient was under 18 years
D)that patient was carrying out a ' Good Samaritan ' act
A and b

17-dental care professionals include


A) dental nurse
B) dental therapists
C) dental practice manager
D) dental health educators
A and b

18-How many members have GDC


A) 29 members
B)12 members

237
238

C)30 members
D) 24 members
Answer is b

19-How often should consent be obtained?


Before every treatment and examination

20-Which of the following authority of UK provides "guideline for referral"?


A. NHS
B. GDC
C. NICE
D. BDA
NICE

21- How long is maternity leave given by gdc?


Maternity leave minimum 26 weeks regardless of how long tge employee has worked for the
employer the employee can take an additional 26 weeks which makes 52 of which first 8
weeks are fully paid nwxt 13 half pay and statuary pay next 13 no pay only statuary pay and
last 18 weeks unpaid
Minimum that an employee should take is 2 weeks and for someone working in factory
setting is 4 weeks

22- How long one is suspended due to health issues?and How long one is suspended due to
conduct issues?
In both health n conduct issues aftr confirmation 12 months suspension

23-If a patient remains undecided about particular items of treatment after a full discussion,
what can you do to help them?
Offer a second opinion from an independent dentist

24-Normally competent people can become “incompetent” at the dentist, and unable to make
sensible decisions. Why?
Because of fear and pain

25-When is it OK to get Implied Consent?


If a patient lies in the chair and opens their mouth

26-which of the following conditions must be met in order to prove dental negligence? select
all that apply.
a-the pt was not happy with the treatment as it was of poor standard
b- dentist had the duty of care to the pt.
c-pt was overcharged for the treatment
d-duty of care was breached
e-breach of care resulted in damage
bda

27- Which container is used to dispose off waste amalgam?


a.Sharps container
b.White container with lid
c.Open container, placed under water

238
239

answer is b

28- Time of hand wash?


Social hand was is 10-15s, hygeinic hand wash 15-30s, and surgical scrub 2-3 minutes

29-from which year CPD has come into force for dentist and dental nurses?
Dentist from the 1st of Jan 2002
Dental nurses from the 1st of August 2008

30-who holds the responsibility sanctioned to sanction a dentist?


PCC…Professional Conduct Committee

31-where is sharpes disposed?


In yellow sharp box

32-a pt with no positive history came along for scaling as soon as you pick up the scaler you
punch ur finger, what should u do?
a. complete procedure as if nothing has happened
b. chk pts blood for hep b antibody HBsAB
c. check pts blood for hep b antigen
d. chk dentists blood for hep b antibody and hiv antigen
e. chk dentists blood for hep b antigen and hiv antibody
f. dentist should go and take HBsAb vaccine
answer is a

33-Do dental practice manager need a gdc registration?


No

34-A 20 year old female present to your practice having URI knocked out during a course of
domestic dispute by her boyfriend.your next step should be?
A.provide treatment but with good record
B.inform the police
C.refer to GP to discuss your fears
D.call local authority of GDC
E.consultation with colleague then provide treatment
Answer is a

35-According to the gdc, well trained dental hygienist can do all the following except?
A.interpret radiograph
B.scaling and root planning
C.simple restoration
D.tooth whitening
E.administer IAN block
Answer is C
Dental hygienists are registered dental professionals who help patients maintain their oral
health by preventing and treating periodontal disease and promoting good oral health
practice. They carry out treatment direct to patients or under prescription from a dentist.As a
dental hygienist, you can undertake the following if you are trained, competentand
indemnified:
• prescribe radiographs

239
240

• take, process and interpret various film views used in general dental practice
Dental hygienists do not:
• restore teeth
• carry out pulp treatments
• adjust unrestored surfaces
• extract teeth

36-What is redundancy?
Form of dissmisal

37-What injuries are more prone for blood borne viral infections in a dental clinic?
Needle stick injury

38-A dental professional who employs a dental nurse


who is subsequently found to be infected with a blood
borne virus should seek advice from whom?
a)local occupational health services
b)UKAP
c)CQC
d)None of the above
answer is a

39-can the nurse re- sheath and/ or dispose of used LA needles?


No it can only be done by the clinician ,therapist or hygeinist

40-.which is correct use of masks?


A) a new mask should be worn for each patient
B) masks should be changed at least once per hour
C) mask should be changed more frequently with heavy aerosols
D) all of the above
Answer is a

41-which is the process by which an antimicrobial destroys or inhibits the growth of


microorganisms on surfaces?
A) antiseptic
B)sterilization
C) micro bacterial control
D) disinfection
Answer is d

42.which is the most frequently used sterilization method?


A) ethylene oxide
B) dry heat
C) autoclave
D) chemi-clave
Answer is c

43.in what order should PPE be removed( assuming face mask, gloves, apron)?
A) gloves, plastic disposable apron, face mask
B) plastic disposable apron, face mask, gloves

240
241

C) plastic disposable apron, gloves, face mask


D) face mask, plastic disposable apron, gloves
Answer is a

44-The preliminary proceeding committee does not have the power to suspend dentists?
False. It can suspend him immediately if member of public at risk, pending the case to
professional conduct committee

45-Who places the rubber dam on patients?


Dentist

46-the only exception where treatment can be provided to a patient by a DCP without seeing
the dentist or a prescription from the dentist is:
a- dental therapist
b- dental hygienist
c-clinical dental technician
d-orthodontics therapist
e-dental technician
answer is c
Only clinical dental technician can provide treatment for the public for COMPLETE
DENTURES ONLY without a prescription from the dentist this answer is according to the
old rules now it has changed

47-where to dispose endodontic irrigation needles which contained bleach?


In sharps

48-Should used endodontic files be decontaminated and sterilised?


They are single use,but can be used again for the same patient

49-How should extracted teeth be disposed of?


If they do NOT contain amalgam, put into a sharps container (Not into a clinical waste bag).
Teeth with amalgam should be disposed of into an Amalgam Waste Toothbox provided for
this purpose.

50-Can the nurse re-sheath and / or dispose of used LA needles?


No. This can only be done by the operator

51-What personal protective equipment should be used for operative procedures?


Eye protection (glasses/visor), mask, gloves, tunic, shoes (not open-toed).
If there is likely to be considerable aerosol or blood spatter, additionally a disposable apron
and (if necessary) disposable sleeves should be used.

52-Disposable gloves should be used for patient contacts. When should they be disposed?
After every treatment and a new one should be used for the next patient

53-The use of alcohol as a disinfectant or detergent has a very high efficacy in reducing
contamination on surfaces. True or False.
It is false. If there is obvious blood contamination, the presence of protein will compromise
the efficacy of alcohol-based wipes. Alcohol has been shown to bind blood and protein to
stainless steel. The use of alcohol with dental instruments should therefore be avoided.

241
242

54-What bacteria can be found in Dental Unit Water Lines?


Legellonila and psuedomonas aergenosia

55-DUWLs or Dental Unit Water Lines should be flushed out for how long between patients?
And for how long at the beginning and end of day?
20-30 seconds between patients and 2 minutes in the morning and at the end of the clinical
session

56-
Arrange the following in order in the removal of PPE:
A. Face mask.
B. Gloves
C. Wash hands
D. Plastic disposable apron
E. Face and eye protection
Answer is BDAEC

57-Protection against hepatitis B is indicated by what level of antibody??


A)HbsAb>100mIu/L
B)HbsAb>200mIU/L
C)HbsAb<10mIU/L
D)HbsAb level 10 -100mIU/L
Answer is a

58-at the end of the clinical session ,unused trays with sterilized instruments are:
a-stored after being wrapped
b-processed again
answer is B
Its should be considered contaminated regardless if it has been used or not.

59-If a dentist can show that his actions were in line with those of a large no of his
colleagues, he is unlikely to be held negligent ..what is this test called?
BOLAM TEST

60-Any unexpected death (eg death in the dental chair) should be reported either directly or
through police to which Court??
Coroner court
Any death in unexpected death should be reported to the coroners of england wales and NI
and to the PF in sscotland

61-what is the form used for exemption claim?


FS15

62-how do you deal with blood spillage?


blood spillage should be completely covered by disposable towels, than treated with sodium
hypochlorite solution 10000ppm, allow at least 5 min to elapse before cleaning and disposing
of towels as clinical waste.

63-How many days after sterilisation(autoclave cycle) can pouched instruments be stored for:

242
243

1. N type Autoclave or Non vacuum


2. B/S type Autoclave or Vacuum
Answer is 1 year for both(it has changed from 21 or 30 days to one year in new HMT0105 )

64-Audit documents should be stored for how many years?


They should be stored for atleast two years and should not be moved from the premises

65-Dental practices should audit their decontamination practices every how many months?
Audit for Decontamination processes should be EVERY 6 months

66-After how many years can a dentist apply for restoration to the Register after being erased
from the Dentists' register?
After 5 years

67-A dentist's name can be erased from the register for a maximum of 12 months only. True
or False
False erasure is permenant unless dentisit apply

68-Patient's records can be shown to the following. Select two answers.


a. The patient
b. The patient's spouse
c. A police officer
d. A specialist to whom the patient is referred
e. A court of law
a and d

69-what items in this list are single use?


A) 3-in-1 plastic syringe tips
B) demonstration toothbrush used by patients
C)rubber prophy cups
D) hand pieces
Abc

70.perform is used to disinfect impressions, and dentures( including bites and try- ins). How
long should an item be soaked in perform to disinfect it?
A) 5 min
B) 10 min
C) 20 min
D) 15 min
B

71.how is the autoclave tested during routine use?


for type N brownie tst strip
for type B helix test strip

73-.if the patient complains, should the complaint details form part of the record?
A) yes
B) no
b

243
244

74-.how full is a clinical' soft waste'bag before it must be sealed?


A) two thirds
B) one thirds
C) full
D) half
A

75-how often is electric equipment tested?


In dental practice it is often 2-3 years , in low risk assessment it is 5 years

76-who does audit and quality assurance?

77- in case of a complaint stiation after local resolution take place and patient is not satisfied
what is the next step?
Ombudsman for nhs
Dental complaint service for for private patient

78-what are the five things that must be displayed in the surgery?
Infection control, hand cleaning, needle stick injery, local rules and safety policy (not sure
about the answer)

79- A statutory body responsible for enforcing the HSW (Health and Safety Work) Act and
providing an advisory service?
Health and Safety Executive

80- Core subject for CPD – all staff


Dentist 250/cycle of 5 years, 75 hours verified, core subject is 10 hours ME, 5 hours
Radiology and 5 hours infection control
Dental nurses150/cycle of 5 years, 50 hours verified is same core subject

81-what is law?
Law is a set of rules usually inforced by a set of instituations

82- What is the statutory sick pay?


You can get 87.55 pound per week paid by the employer to you for upto 28 week ,you need
to be sick for a contineous 4 days including none working days(weekends and bank holidays)

83-which is better when washing hands ? cold or warm water and why ?
Warm, When combined with a detergent, the warmth helps to dissolve the natural oils on the
hands that harbour bacteria.

84-liquid or solid soap ?


Ordinary soap is not antibacterial – in fact, bacteria can grow well on soap. Using ordinary
soap will remove obvious dirt, but may actually increase the amount of pathogens on the
hands.
Even antibacterial solid soap will be remain contaminated on the surface after it is used.
Liquid or foam detergent hand wash, and dispenser
These are recommended for use in dental practices. The dispenser should ideally be “hands-
free” – an infra-red beam detects when a hand is under the dispenser. Most experienced
dental staff will have seen how dirty a “push-dispenser” gets after a while.

244
245

85-orange sticks or nail brushes ???


orange sticks should be used when hands are initially washed to clean dirt from under the
nails. This should be done gently. It is recommended (by HTM01-05) that nail brushes should
not be used.
Scrubbing brushes
The dental team should not use scrubbing brushes to clean the skin. Because repeated use can
cause damage to the skin, making it more likely to harbour pathogenic bacteria. There are
also reports that scrubbing actually increases the surface bacterial count, perhaps by bringing
deep bacteria up to the surface.

86-How does the GDC protect patients from registered Dental Professionals? True/false
A) by setting standards of patient care
B) by setting standards of behaviour in the practice
C) by routinely inspecting dental practices
D) by approving CPD courses
Answer is a

87. If collimator intra- oral x Ray tube is placed further away from the patient
A) they will receive less radiation dose
B) they will receive the same radiation dose
C) they will receive more. Radiation dose

88. The optimal film speed for intra oral radiography is


A) c speed
B) d speed
C) e speed
D) f speed
Answer is d

89.which of the following protocols must be included in the" written practice protocols"?
A) disposal of hazardous waste
B) disposal of sharps
C) annual leave entitlements
D) radiation protection
E) autoclaving
Answer is c

should be able to deal with emergency- all

4-- How will you treat a spillage of less than 30 ml blood?

5-- How will you dispose news paper?

6--Which container is used to dispose waste amalgam, wooden wedges?

7-- How will you dispose clinical waste.yellow containers

245
246

8-- What is presterilization, how is it done

9-- CROSS INFECTION AND THE DIFFERENT TYPE OF BACTERIA IE; MOST INFECTIOUS, MOST
RESISTANT TO STERILIZATION ETC1--Consent different types like who can give consent in
which circumstances:

2--Questions about inform consent : Can be verbal or nonverbal

3--An informed consent should be taken from subjects of a study for it to be - valid

4--Which type of consent is not present? Delegated

5-- From what age can a person give consent for treatment? 16 years

6-- Is there an age limit for confidentiality of information? No age limit

7--What is not important in consent taking? Whether the patient can read or write

8-- What is not intended in consent taking? Informing the patient

9--What type of consent is required for conscious sedation? Written

10-- To whom is the dentist obliged to give a patient’s confidential information? Court of law

11-- The most valid consent is given by the mother of a 4-year old child.

12-- The best way of giving information. Written information is more effective than verbal
information

13-- Ques on consent:if a patient under 16yrs, parents not present whom will u

take consent from?was not direct so pls read in detail about it.

14-- year old girl in boarding school comes to your surgery for an extraction which of the
following cannot give consent on her behalf?

a. Her grandfather with legal guardianship ( PLEASE CONFIRM )/her grandfather has
lega rights he can

b. Her biological father who has separated from her mother

c. The girl herself

d. Her older sister/can not

15-- Consent is needed from a patient to share information, which of the following needs
explicit consent?

a. To share information with an insurance company

b. To share information with other doctors in the practice

c. To share information with the therapist treating the patient

246
247

d. To share information with the patients GMP

- 1st importance of consent – care of the patient

16--What would you do when a patient comes with an asymptomatic root canal treated
tooth with periapical radioluscency treated by some other dentist?

You speak to the previous dentist for old radiographs/I think you wait and observe

17--Questions on consent given by parents of 14-16 year olds who participated in an oral
health survey

Explicit Consent

20. Which is not a method of taking consent. Delegation

21. IV sedation –which consent is taken Written

22--Which among the following is least important in taking consent?

The trainee understands the procedure of taking consent, he has done it previously, he
understands the risks associated with the procedure, he explains the risk to the patient, he
follows the procedure from a textbook

CPD
1--The number of hours of CPD recommended for dentist to avoid removal from the dental
register

2-CPD hours requirements for dentists and DCPs 250 and 75 varifiable

3-How many cpd hrs do the nurses need from july 2008 150 and 50 varifiable

4-- How many cpd hrs do the dentist need-250-75 verifiable

5--- Core subject for CPD – all staff ( Medical Emergency 10hours , Radiology and Radiation
protection 5 hours, Infection Control 5 hours )

6How many hours of C.P.D. do dentists have to complete in order to avoid being struck off
the register? 250 hours

7- How many hours do DCPs have to do for CPD? 150 hours

8- What is the core subject For CPD ALL STAFF -? – Medical Emergencies

9- From which month & year did CPD come into force – august 2008

10- What job hygenist, technician and dental therapist can do

247
248

Hygienist ( Scale, polish, clean, apply prophylactic material to the tooth, Can administer LA if
they are qualified prior to 1992 and have attended courses or have taken Diploma in Dental
Therapy )

Technicians ( can take shade for prosthesis, Impression of study model )

Therapist ( a part from what dental hygienist can do, Dental Therapist can extract primary
teeth, undertake simple fillings, administer LA, they are also able to carry out work under
regional block)

11In which year is GDC going to conduct revalidation?

a. 2010

b. 2011

c. 2012

d. 2013 ( PLEASE CONFIRM )

GDC

1- Aim of GDC

Protect general public

2-functions of gdc

Maintain a register of Qualified Dental Professionals

Set Standards of dental practice/ conduct

Assure quality of dental education

Protect patients and assist with complaints.

3-function of NICE guidelines.

4-According to Data Protection Act, the patient has access to all computerised records.

which group of people are not required to be registered with the GDC by July 2008

a)dentist b)dental nurses c)dental technicians d)practice managers

5-questions about Fitness to practice and Professional Performance Committee and their
duty.

Please read Master Dentistry page 311

6-Can’t consider patient complaint? after 12 months.

7- Who’s not exempt of NHS charges? Adults on benefits, older people on retirement…..

248
249

You do not have to pay for NHS dental treatment if, when the treatment starts, you
are:
 aged under 18
 under 19 and receiving full-time education
 pregnant or have had a baby in the previous 12 months
 staying in an NHS hospital and your treatment is carried out by the hospital
dentist
 an NHS hospital dental service outpatient (however, you may have to pay for
your dentures or bridges).
You also do not have to pay if, when the treatment starts, you are receiving:
 Income Support
 Income-related Employment and Support Allowance
 Income-based Jobseeker’s Allowance
 Pension Credit guarantee credit
or
 you are named on a valid NHS tax credit exemption certificate or you are
entitled to an NHS tax credit exemption certificate
 you are named on a valid HC2 certificate

8--What happens if the dentist does not pay his annual retention fees on time?

Struck off from the register

9Which of the following is the major function of the GDC

A. Regulating dental practice

B. Maintaining a register of dentists

10GDC main function – protect the patient

11- GDC registration not needed by 31st July – Dental Practice Manager

12-GDC – conduct – Fitness to Practice Panel

13- GDC – health illness (dentist) – Practice Committee

14--Patient arrives saying “I don’t like dentists”. What do you do? Say “I don’t like dentists
either”, ask what the reasons are, suggest IV sedation, etc

249
250

15--What’s the maximum of time the GDC leaves the dentist physically impaired away from
his profession? 12 months

16-Whats the best way of giving oral health education e.g options- nurse advices, leaflet,
diagrams, make sure the patient understands what causes gum diease& dental caries

17---Enquring about pts oral hygiene care- open or closed questions, some questions given
so you select one.

18---Needle stick injury ---what do you do.?.(not the usual stuff of bleed the area)..the
choices were; go to A&E, visit GP, cont. Working, Contact your occupational health advisor

19---new employee with low antibodies to Hepatitis B… booster dose has to be given

20-A patient wants their records for some insurance stuff- which act do you match it with
( Data Protection Act 1998 )

21- A 62 yr old is denied a job because of his age -which act do you match it with
Discrimination Act

22-- A new dental nurse wants to do radiograph-which act do you match it with Radiation
Protection

23--A A female nurse is denied leave -which act do you match it with Equality and Diversity

24--Some of the options were-data control act, equality and diversity, freedom of
information, radiation protection….

25--Urgent referrals should be seen with what time. 24 hours/ I think its 2 weeks

26--What is the work of NICE?

Providing national guidance on promoting good health and preventing and treating ill health

27.How long can the GDC suspend someone for not being fit to practice. 1 year or 12
months

28 --How many months leave for a pregnant employee and how many months of pay she is
entitled to from April 1st,2007. 6 weeks full pay

29--Laws and regulations governing different scenarios e.g Dental Nurse wanting to take
radiograph. Options included IRMER, Health and Safety, COSHH

30--• Dental nurse that had needle stick injury, can she go back to work? YES

31--GILIK law Competent child can give consent

INFECTION CONTROL

1--What sort of working surface is best in a clinical working environment? Impervious

250
251

2-- Why is making a dental chart important? Helps in future diagnosis

3-- Who all in a dental surgery

10-infection control new guidelines pls read the sign for single use

11-- instruments,chronological order for sterilisation cycle,washer disinfector system,hand


washing

12-- Before sending impression to lab, what is the most important thing to do? Disinfection?
Put in running water?

13-- What is the hand-washing time that is most effective and practical? 2 minutes

14 Colour of first aid box…..green + white cross

15-- Waste management-

16-- Where do you dispose clinical wedges?

17-- How do you dispose amalgam?

a.Sharps container

b.White container with lid

c.Open container, placed under water

18- Which container is used to dispose off radiology chemicals?

19Instruments management 2 questions (5 sub questions) – autoclave, disinfection, sharps


bin, clinical waste (yellow

20-PROTOCOLS FOR CLINICAL AND NON CLINICAL WASTE

21--What kind of organisms cannot be destroyed by sterilization?

a. Spores

b. Prions

c. Thermophiles

22Most effective method of reducing infection is sterilisation.

23- Most commonly used hand-was is chlorhexidene gluconate.

24- Most transmission of prions is by the aspirator tip.

25- Experimentally the most proven method of disinfection is the washing of hands before
see a patient.

26-How would you treat a person with TB, HIV, Hepatitis, MRSA-options send them to a
hospital, use single use equipment, treat with sterile instruments, treat using standard
conditions of infection control

251
252

27-----how often a dentist has to wash his hands

a,after every procedure

b after shaking hands

c,after examining every patient

28--which is not a PPE(personal protective equipemnt)

a,surgery gown

b,tunic

c,gloves

d, mask

29--n- type of autoclave some question on that..

30--what PPE you remove first

a,gloves

b,mask

31--should have separate room for sterilization?

Dental material
1-what does an enamel bonding agent consist of
A) unfilled resin
B) filled resin
C) wetting agents or resins
D) prime and bond mixture
Answer is b

2.what is the bond strength achievable using enamel bonding agents on etched enamel
A) 0.1 MPa
B) 5 MPa
C)30 MPa
D)80 Mpa
Answer is c

3.what is percentage of copper in a high copper dental amalgam alloy


A) 2-12%

252
253

B) 12-32%
C) 32-52%
D) 52-70%
Answer is b

4.which one of these chemical composition is the gamma 1 phase?


A) Ag3Sn
B) Ag2Hg3
C)AgCu
D)Cu6Sn5
Answer is b

5.how longit takes for amalgam to form bond with tooth


A) 3 minutes
B) 3 hours
C) 3 days
D) 3 months
Answer is d

6-which acid is added to lengthen the working time of glass-ionomer?


Tartarict acid

7-Dental amalgam wears at a rate similar tooth structure T/F


Answer is true

8- What is the maximum thickness of a composite increment that allows for proper cure
A) 2-4 mm
B) 4-5 mm
C) 1-2 mm
D) there is no maximum thick restriction
Answer is c

9.what is the half life of mercury in the human body


A)55 days
B) 25 days
C) 10 days
D) 90 days
Answer is a

10. Which is not property of a glass ionomer base material


A) chemical bond to tooth structure
B) fluorid release
C) anticariogenic activity
D) relies on mechanical retention from the cavity preparation
Answer is d

11. Which material is associated with the most marginal leakage due to temperature
A) composite resin
B) cast gold alloy
C) amalgam alloy

253
254

D)unfilled resin
Answer is d

12. Which material has the highest coefficient of linear expansion


A) tooth structure
B) amalgam
C) composite resin
D)direct gold
Answer is composite resin

13-which imp material is contraindicated for undercuts?


Zinc oxide eugeole

14-what type of gloves can retard setting of putty materials?


Powdered latex gloves

15-stiffness of material is measured by


proportional limit
modulus of elasticity
stree/strain
ultimate tensile strength
answer is modulus of elasticity

16-what is the greatest risk associated with dental amalgam?


Mercury inhalation

17-Alginate impression material is:


A.Expensive compared to other impression materials
B.Easy to use
C.Not affected by gain or loss of water
D.Well known for its long-term stability
It is easy to use

18-Impression materials that have mechanical properties permitting considerable elastic


deformation but that return to their original form are classified as:
A.thermoplastic
B.elastometric
C.inelastic
D.resins
Elastomer

19-Which of the following dental materials is an example of an aqueous elastometric?


A.Impression compound
B.ZOE impression paste
C.Polysulfide
D.Irreversible hydrocolloid
E.Addition silicon
Answer is d

20-Dental impression compound is known as a/an

254
255

A.chemoplastic material
B.irreversible material
C.thermoplastic material
Answer is c

21-Suck-Back porosity is commonly seen in_______?


Inside of the crown opposite the sprue.

22-Strength of porcelain decreases due to


1-Over firing
2-Entrapped air bubbles
3-Sudden cooling
4-All of the above
Answer is 4

23-Zinc is added to amalgam alloys because it makes the amalgam more....


1-Brittle
2-Plastic
3-Hard and Strengthened
4-Quick setting
Answer is plastic

24-Ideal dental stone for models is


1-Type I
2-Type II
3-Type III
4-Type IV
5-Type V
Answer is 2

25-Young's modulus defines?


1-Elastic limit
2-Stiffness
3-Resilience
4-Flexibility
Answer is 2

26-what component of amalgam causes tooth blackening?


Silver

27-which is more likely to happen to an interproximal composite filling rather than amalgam
filling?
1- over contouring
2- under contouring
3- overhang
4- fracture
Answer is 2

28-Impression in patient with resorbed lower ridge ?


Answer is compound impression material

255
256

29-Which impression material cannot be used for crown and bridge impression ?
Alginate

30-What is the depth of curing composite ?


Answer is 2mm

31-Control of the mercury content during mixing and condensing of amalgam must be
carefully considered because:
A: The higher the mercury content, the higher the strength and the lower the marginal
breakdown
B. The lower the mercury content, the higher the strength and lower the marginal breakdown
C. The higher the mercury content, the higher the strength and the greater the marginal
breakdown
D. The lower the mercury content, the higher the strength and the greater the marginal
breakdown
Amswer is b

32-What it is advantage of high copper amalgam ?


Prevent the gamma phase 2.less corrosion

33-Which is an example of an auxillary dental material?


a)ceramic
b)Amalgam
c)Composite
d)Cast alloy
e)gypsum product
answer is e

34-Which one of the following materials does NOT release fluoride?


a. Traditional glass ionomer
b. Resin-modified glass ionomer
c. ART restorations
d. Provisional restorations
e. Compomer
answer is d

35-Calcium hydroxide its action and its uses??


is a bacteriostatic causes meniraliztion of the underlying pulp used for pulpcapping,root end
surgical restoration ,perforation restoration and apexigenesis,composed of calicium
hydroxide and polysilicate acid contanin a PH of 11

36-what is the percentage of silver in amalgam??


A)50%
B)55%
C)65%
Answer is c

37-what is the composition of glass ionomer cement??


Alminum silicate and poly alkalonic acid which gives alaminuim alkenoate salt and calicium

256
257

38-what component of amalgam increases the strength??


Copper and silver

39-what causes amalgam expansion??


zinc

40-imbibition is seen in which impression material??


Alginate

41-Which impression material cannot be used for crown and bridge impressions??
A)elastomer
B)alginate
C)reversible hydrocolloid
Answer is b

42-What type of material is ChemFil


A) composite
B) composer
C) glass ionomer
D) resin modified glass ionomer
Answer is c

43. ChemFil' s elasticity ( elastic modulus) matches that of


A) cementum
B) dentine
C) enamel
D) non of these
Answer is b

44.chemFil is suitable to treat


A) erosive cavities
B) toothbrush abrasion cavities
C) cervical caries
D) occlusal cavities as it has good compressive strength
Answer is b

45.what is the typical particle size in microfilmed composite, in micrometers


A) 0.04
B) 0.4
C) 4
D)40
Answer is a

46. Which of these lining materials should not be used with composite
A) life
B) poly-F
C) zinc phosphate
D) kalzinol
Answer is d

257
258

47-which of the following is hydrophilic in nature?


a-polyether
b-polysulfide
c-addition silicone
d-condensation silicone
answer is a

48-)What is the point of the water spraying?


a)To ensure all the acid is washed off
b)To ensure the calcium phosphate precipitates caused by the etching are washed away
c)Both of these
answer is c

49-What is 'conditioner' ?
Acid
Alcohol
Dentine priming agent
Dentine bonding agent
Acetone
Answer is acid (dentine conditioner is 10% poly alkenoic acid)

50-Commonly used etchant gel?


Phosphoric acid 30-50%

51-How much general surface enamel is removed by etching (in micrometres) ?


0.1
1
10
100
Answer is 10 micromillimeter

52-How should you protect adjacent teeth from being etched?


a)Rubber dam
b)Vaseline
c)Mylar matrix strip
d)Cavity varnish
answer is C

53-Cement for cementing temporary bridge ?


Zinc oxide euogenol or zinc oxide eugenol free

54-which resin is also called Bowen's resin?


a-TEG-DMA
b-Bis-GMA
c-UDMA
d-unfilled acrylic resin
e-composite
answer is b

258
259

55-shelf life of polyether imp material is


a-less than 1 year
2-more than 2 years
3-between 1 and 2 years
4-none of the above
Answer is 2

56-dustless alginate contains


a-glycerol
b-glycol
c-glacial acetic acid
d-silica coated powdered particles
answer is b

57-size of monomer/resin and amount of filler content effects which property of composite?
a-thermal contraction
b-thermal expansion
c-biocompatibility
d-both 1 and 2
e-polymerization shrinkage
answer is e

58-What is the type of composite used for veneers?


Hybride composite

59-impression material for resorbed ridge ?


Compound material

60-impressiom material for flabby ridge?


Selective impressiom or mucostatic impression bt the use of tissue conditioner
ZOE or silicone for mild condition more sever conditiona selective pressure impression in a
special tray opening a window at the site of the flabby tissue the zoe is removed from the
window and another impression is taken with a plaster impressiom

61-high gag refex impression material?


Fast setting alginate

62- impression for pocelin fused metal crown?


Addition silicon

63-impersion for special tray?


Alginate

64-impressiom material for CR/CO denture?


Polyether and additional silicon

65-primary expansion of amalgum is due to ?


zinc

66-why did they add zinc to the amalgum fillinfg in the past?

259
260

Scavengering

67-which material undergoes syneresis and imbibiton?


Alginate

68-setting reaction of alginate?


Na alginate+CaSo4==sodium sulphate+calcium alginate

69-what is the duration of time for fast setting alginate and long setting alginate?
Fast setting is 2 minutes
Long setting is 4 minutes

70-GP soften and melts at what degrees?


Soften at 60C
Melts at 100C

71-what provides the best polished surface?


Enamel hybride

72-what is ZOE trade name?


Tubuseal for root canal sealer and kalzinol for temporary dressing

73-adams clasp is made of ?


Stianless steal

74-a cement that is used as a root canal sealer and is a resin based?
AH +

75-what is the impression material; used for ?


a-primary and secondary impression with falbby ridge?
b-patient with gag reflex
c-resorbtion of ridge
d-dry mouth
answers a-primary impression is alginate
secondary impression is mucocompressive orselective using a special tray with a
window over the flabby area make impression with zoe then remove zoe from window and
then with impression plaster
b-patient with gag reflex additional cured silicon or zinc
c-impression compound
d-elastomers ZOE is C/I

76-which following aids in wetting in dentine bonding?


a-acetone
b-water
c-orthophosphoric acid
d-ethylene
e-carbon tertachloride
answer is a

77-impression for a complete denture for a flappy ridge?

260
261

Mild- alginate or elastomer in a perforated tray


Sever-2 stage technique special with window over the fappy tissue first impresiom with zoe
and second impresion in a lower viscosit elastomere or impresssion plaster

78- Glass ionomer cements are used in restoring Class V cavities.


Which of the following constituents are most likely to be present in glass ionomer cements?
A.. Alumino-silicate glass and phosphoric acid
B. Phosphoric acid and zinc oxide
C. Polyacrylic acid and zinc oxide
D. Aluminosilicate glass and polyacrylic acid
E. Itaconic acid and zinc oxide
I think answer is d

79-Dental amalgams contain different levels of copper (> 6 %) in the alloy.


What is the function of the copper in the amalgam?
A. To act as a scavenger
B. To reduce the setting time
C. To prevent an exothermic reaction on setting
D. To reduce/eliminate the formation of the gamma 2 phase
E. To prevent hydrolysis
Answer is d

80- What causes Amalgam expansion? What makes Amalgam black after sometime?
Amalgam Expansion:
1- Moisture contamination during mixing and condensation operation.
2- Moisture in the saliva is the potential contaminant for the amalgam.
3- Zinc containing amalgam, the presence of saliva on the amalgam during condensation
probably a possible source of expansion.

Amalgum Black:
1- corrosion.
2- Reaction of the phases.
3- Marginal leakage.

81-what is the compostion of GP?


20% gutta percha,60% zinc oxide ,11% heavy metal sulphate and 3 %wax andn resin

82- when the impression is stored and sent to the lab covered with a wet cotton what happens
to
the alginate material?
Synerisis decrease
Synerisis is extraction of water from material
Imbibition is absorption of water

83- Cement for cementing porcelain veneers?


Resin composite cement/ Panavia

84- Cement which requires polyacrylic acid?


Conventional glass inonomer cement

261
262

85- Where does GIC bond better to?


a-Dentine,
b-enamel
c-colagen
answer is enamel

86- Irrigating solutions other than sodium hypochlorite ?


chlorhexidene,EDTA and urea peroxide

87- what is the choice of crown for a tooth with MOD amalgam filling and fractured palatal
cusp?
a-full gold crown,
b- ¾ gold crown,
c-porcelain bonded crown
answer is a but more preservative is ¾ crown

88- what cement you use to glue a alumina core ceramic crown ( full cermic)?
a-Panavia,
b- zinc phosphate,
c-RMGI,
d-Poly carboxylate,
c-GIC
answer I think is a

89- which one is polished the best?


a-Microfilled,
b-glass ionomer,
c-resin modified glass ionomer
I think answer is a

90- Greatest wear resistance?


Setting contraction of composite (6% by volume can be reduced by using small increments)
versus GIC 3% ……..Conventional GICs have NO polymerization shrinkage

91-what is the two paste zinc oxide eugenol system?


Tubliseal

92- – Amalgm Marginal Angle angle (more than 70°) and CVS angle (ideal 90°), Nayyar
core is 2-3mm preparation in the canals.

93- cavosurface angle(smallest and largest possible angles to avoid fracture)


options:50,70,90,110,130
answer is ( 70-110)

94- the angle between the prepared and unprepared surfaces in amalgam restoration occlusal
– 90o
*the angle between the prepared and unprepared surfaces in amalgam restoration 70-110o

95- what kind of alloy can you make implants from?


Tetanium

262
263

96- metal for pfm crown?


a-Medium gold
b- high gold
c-silver palladium ,
d-nickel chromium
answer is a page 651 pink book

97-luting cement for PFM?


Panavia and gic

98- which bur would you use to cut enamel dentinal junction?
straight fissure diamond bur

99- .different types of irrigants and their main function in rct?


Na hypochrite ( anti bacterial , dissolves debris)
EDTA ( used in obstructed canals
Chorlexidine ( anti bacterial
Urea Peroxide ( releases nascent oxygen which kills the anaerobes and removed debris from
canal)

100- rct sealer resin-based?


AH plus, endorez, acro seal
Answer is AH

101- toxic root canal sealantS?


AH26

102- which cement is placed after treating with polyacrilic acid?


GIC

103- .ni-ti quality that makes them better than stainless steel?
superelasticity, shape memory

104- the first thing to do after taking an alginate impressions?


wash first and then disinfect

105- a dentist using a soflex disc without finger support.what would happen?
• Mucosal burn
• Mucosal tear
• Gingival burn
• Gingival tear
• Damage to the adjacent tooth
Gingival tear
106- which cement is more susceptible to fail if is contaminated with saliva?
GIC

107- - what is% of duraphat fluoride varnish?


2.26 % ( 22600 ppm of fluoride

263
264

108-size of monomer/resin and amount of filler content effects which property of composite?
a-thermal contraction
b-thermal expansion
c-biocompatibility
d-both 1 and 2
e-polymerization shrinkage
answer is e

109-Which material is not recommended for root end filling?


A-MTA
B-EBA
C-glass ionomer
D-composite resin(with a dentine bonding agent)
E-reinforced zno eugenol
F-amalgam
Answer is F

110-For application of porcelain to a ceramo-metal alloy, the correct viscosity is achieved by


mixing the porcelain powder with
A. a porcelain modifier.
B. pure methyl alcohol.
C. a mild detergent.
D. distilled water
Answer is A

111-In metal-ceramic crowns, the bond between the materials is


A. mainly mechanical.
B. mainly chemical.
C. decreased by oxides on the metal surface.
D. decreased when the metal has a high yield point
Mainly chemical

112-Which of the following materials are LEAST suitable for impressions for cast gold
restorations?
A. Polysulfides.
B. Polyvinyl siloxanes.
C. Polyethers.
D. Irreversible hydrocolloids
Hydrocolloids

113-Nickel-chromium alloys designed for porcelain bonded to metal crowns should be used
with caution because
A. nickel is an allergen.
B. the modulus of elasticity is low.
C. these alloys cannot be soldered.
D. None of the above
Answer is A

114--Impression trays should be


A. rigid.

264
265

B. flexible enough to permit easy insertion.


C. carefully polished on the interior.
D. held in place by the patien
Answer is A

115-A model prepared from a vacuum mixed stone has higher strength because
A. less water is required for vacuum mixing.
B. there is less porosity.
C. some of the water is removed by the vacuum.
D. the nuclei for crystallization are more numerous
Answer is A

116-)what is the choice of a crown for a tooth with amalgam filling and fratured palatal
cusp??
A)full gold crown
B)3/4 crown
C)Porcelain bonded crown
Answer is A

117-T/F
A)AH plus is a resin based paste.
B)setting contraction of composite 6% by volume can be reduced by using small increments
C)setting contraction of GIC is 3%
A-T,B-T,C-T

118-what is the depth of curing composite?


A)1mm
B)2mm
C)3mm
Answer is B

119-At what temperature Gutta-percha softens and melts??


It softens at 65 and melts at 100

120-what cements are used in stepwise excavation?


Calcium hydroxide and GIC

121-which of the following materials exhibit syneresis?


a-polysulphide
b-polyether
c-condinsation silicone
d-addition silicone
e-none of the above
answer is condinsation silicone

Oral surgery

265
266

1- Balancing extraction is a tooth extraction from the opposite side of the same arch, designed
to minimise centreline shift. Compensation extraction means extraction of tooth from the
opposite side of the same arch , designed to minimise centreline shift.
A) first sentence is correct, second false
B) both are true
C) both are false
D) first sentence is false, second is true
Answer is a

2.ideal timing for extraction of the first permanent molar is


A) 7-8 years
B) 8-9 years
C) 8-10 years
D) 12-14years
Answer is c

3.which one of the following is not a complication of removal of mandibular wisdom teeth?
A) dry socket
B) anaesthesia of the inferior dental nerve
C) paraesthesia of the facial nerve
D) trismus
Answer is c

4.which one of the following is an indication for extraction of a lower wisdom tooth?
A) anterior crowding
B) the tooth is distoangular in position
C)the patient has had two episodes of pericoronitis
D) to appease a patient who has atypical pain
Answer is c

5. Which of the following are contraindications to surgical endodontic


A) unusual bony or root configuration
B) possible involvement of neurovascular structure
C) when tooth is subsequently in restorable
D)where a biopsy of peri radicular tissue is required
Answer is a,b,c

6-What is the main risk of canine ectopia?


Resorption of the root of the lateral incisors

7-Which method gives an accurate assessment of future crowding by using the sizes of
erupted lower permanent incisors?
Mixed dentition analysis(MDA) which predict the sizes of unerupted permanent canines and
premolars

8-Exfoliation of which primary tooth during the eruption of permnt lateral incisor is a sign of
severe crowding?
C

9. Which is not a classification feature for mandibular fracture

266
267

A) anatomical location
B) angulation of the fracture and muscle pull
C) leFort level
D) description of the bone fragmentsat fracture si
Answer is c

10-What is this instrument?


It is a left cowhorn maxillary molar forceps

11-Patients on oral Bisphosphonates:


a)Must not be treated in General Dental practice.
b)will develop osteonecrosis of the jaw after undergoing tooth extraction.
c)Require antibiotic prophylaxis with 3g amoxicillin provided they are not allergic to
penicillin.
d)most probably they have osteonecrosis or metastatic bone disease.
e)Must have their bisphosphonate medication stopped 24 hours prior to dental treatment.
Please do justify the answer as well.
Answer is D, BRON is a condition where alveolar bone is exposed the mouth in patients
taking bisphosphonates drugs

12-what is the most common type or oral cancer ? how common ?


Squamous cell carcinoma 90%

13-Where are oral cancers most commonly situated?


Oral cancer appears most commonly in the lateral border of the tongue, and floor of the
mouth.
In Southeast Asia buccal (cheek) mucosa is the commonest site (betel etc chewing habit).
However, it may also occur on the buccal mucosa, gingiva, lips, or palate.

14-Who is more likely to get oral cancer in the UK – men or women? Older or younger?
The Incidence is higher in men, and in older compared with younger people.

15-What percentage of oral cancers in younger people do NOT appear to be related to


alcohol/tobacco use?
About 25%. It is thus important to examine every patient thoroughly, even if they appear to
have no obvious risk factors. The Human Papilloma Virus (HPV, or Wart Virus) is believed
to have a role, being found in up to 70% of tongue and pharyngeal cancers. This is the same
virus that causes cervical cancer

16-What are the four main and most likely reasons for a dentist to refer a patient regarding
possible oral cancer?
unexplained ulceration of the oral mucosa or mass persisting for more than 3 weeks
unexplained red and white patches (including suspected lichen planus) of the oral mucosa
that are painful or swollen or bleeding
unexplained tooth mobility not associated with periodontal disease
For patients with persistent symptoms or signs related to the oral cavity in whom a definitive
diagnosis of a benign lesion cannot be made, refer or follow up until the symptoms and signs
disappear. If the symptoms and signs have not disappeared after 6 weeks, make an urgent
referral

267
268

17- what does LeFort 1 fracture cause??


A)Loosening of maxilla
B)crack pot sound
C)involves the maxillary sinus
D)all the above
Answer is B

18- What is the condition expected after extracting a tooth in a smoker??


A)Dry socket
B)BRON
C)osteoporosis
Answer is A

19- All of the following are non resorbable sutures except


A-silk and nylon
B-prolene
C-plain gut
D-mersilene
Answer is C

20- A man with multiple myeloma comes for an extraction and comes back 6 weeks later and
his socket has not healed what can cause this ??
A)Multiple myeloma of the mandible
B)Drug induced osteonecrosis
C)Dry socket
D)Drug induced osteosclerosis
Answer is B

21- Sutures are generally left for the longest time on what part of the body?
A-face
B-scalp
C-joint
D-trunk
Answer is c

22- Which of the following suture is stronger and is preferred when there is tension on the
wound
A-simple interrupted
B-horizontal mattress
C-continuous without interlocking
D-a&c
E-a&b
Answer is B

23- Which of the following is incorrect regarding principles of suturing technique?


A-the needles should be perpendicular when it enters the tissues
B-the suture should be placed at an equal distance from the wound margin(2-3mm) and at
equal depths
C-tissues should be closed under tension

268
269

D-sutures should be placed from mobile tissue to fixed tissue and suture knot should be on
the side of the wound.
Answer is C

24- Suturing of the tongue should be preferred with


A-chromic catgut
B-catgut
C-vicryl(semisynthetic absorbable suture material)
D-black silk
Answer is C

25- : extraction of 3rd molar with dentigerous cyst in the angle of the mandible , sup and Inf
border of mandible very thin. What can happen during the extraction?
Fracture of the mandible

26- ORO-ANTRAL FISTULA TREATMENT?

27- SUTURE MATERIALS ARE USED FOR BIOPSY IN LIP TRAUMA


Transfixing with a 3/o BSS FOR biopsy and prolene for lip

28- FIRST THING TO DO WHEN A PT COMES BACK TWO HOURS AFTER


EXTRACTION WITH BLEEDING?
29-What are the two most common features of a failed implant?
30-Which type of impacted teeth present the greatest surgical challenge.
A. Horizontal Impaction
B. Vertical Impaction
C. Mesio-angular impaction
D. Disto-angular impaction
Answer is distoangular as it is plased distally there will be more bone needed to be removed

31- Which is the resorbable suture of choice when suturing intra orally after a surgical
extraction
A) black silk suture 3/0
B) polypropylene 3/0
C) vicryl 3/0
D) catgut 3/0
Answer is c

32. An incisional biopsy is indicated in which one of the following lesions


A) squamous cell carcinoma
B) fibroepithelial polyp of the lip
C) buccal haemangioma
D)amalgam tattoo
Answer is A

33.which one of the following is not common sign of a fractured zygoma


A) Diplopia
B) paraesthesia of the infra orbital nerve
C) epistaxis
D) anosmia

269
270

Answer is d

34.what is the most common cause of a fractured mandible


A) road traffic accidents
B) inter personal violence
C) sporting injury
D) industrial accidents
Answer is B,fighting is the most common cause to fracture in the western world while in the
middle east road traffic accidents in the most cause of these injuries

35.what is the correct treatment for an asymptomatic torus palatinus


A) antibiotics
B) incisional biopsy
C) excisional biopsy
D) none of the above
Answer is d

36. Which is more severe classification of maxillary fracture


A)LeFort I
B) leFort II
C) LeFort III
D) LeFort IV
Answer is c

37.a 21 year old has a mandibular fracture. Which statement is false?


A) the most common x Ray obtained would be a panoramic radiograph
B) the most likely area for mandibular fracture is the mandibular alveolus
C) at least two x Ray should be obtained
D) point tenderness, occlusion changes, step deformities, and gingival lacerations are
observed
Answer is b and c

38.what is a simple fracture


A) lack of connection between fracture and oral cavity
B)fracture through the cortical portion of the bone
C) most common in children
D) multiple fracture of a single bone
Answer is a

39.which is correct acronym for a common treatment of a fractured mandible


A) OTIF
B) ORIF
C)RIMA
D)OSIM
Answer is b

40-best radiograph for diagnosis of impacted wisdoms?


DPT and PA
For condyler reverse towner’s
For fracture between symphsys and canine fracture rotated PA

270
271

41-best suture for skin?


a-dexon
b-vicryl
c-silk
d-navofil
answer is d

42-what is not an indication for removal of 8?


a-recurrent pericoronitis
b-cystic change
c-crowding on the same segment of the arch
d-external resorption
answer is C

43-best anaesthesia for extraction of bilateral impacted wisdom teeth?


a-inhalation sedation
b-IV sedation
c-GA
d-LA
answer is c

44-cerebrospinal fluid leaking from the nose and ear suggest what type of fracture?
Le Fort 2 and three and nasoethmoidal

45-A type of maxillary fracture which involves the nasal bone and zygomatic-frontal sutures:
A. Le Fort I
B. Le Fort II
C. Le Fort III
Answer is C

46-It is also considered as a 'blow out' fracture.


A. Zygomatic Fractures
B. Mandibular Fractures
C. Orbital Fractures
D. Maxillary Fractures
Answer is orbital

47-What is the minimum splinting time for Dento-alveolar fractures?


A. 2 weeks
B. 3 weeks
C. 4 weeks
D. 6 weeks
Answer is c

48-Where is inferior alveolar canal located?


A. It lies below the roots of the third molar.
B. It lies below the roots of the second molar.
C. It lies distal to the roots of the third molar.
D. It lies distal to the roots of the second molar.

271
272

49-The following radiological signs are associated with an increased risk of nerve injury
during third molar surgery except:
A. Interruption of the lamina dura of the interdental canal overlying the tooth.
B. Darkening of the root where it is crossed by the interdental canal.
C. Periradicular bone sclerosis
D. Diversion of the interdental canal
Answer is c

50-Which part of the mandible is the commonest site for fracture?


Condyler

51- What are the two most common features of a failed implant?
Mobility and bone resorpition

52-Xerostomia does not:


A. Occur after radiotherapy
B. Occur in patients with sjogren's disease
C. Occur during panic attacks
D. Cause an increase in root caries
E. Occur when taking pilocarpine
Answer is E

53-- after repair of an oroantral fistula , which one of the following is unnecessary ?
A. Analgesia
B. Antibiotics
C. Ephedrine nasal spray / drops
D. Steam inhalations
E. Referral to ENT team
Answer is E

54-diagnosis of different fractures according to signs and symptoms?


A)Mandible :1- classification is simple(closed fracture),
2-compound(open fracture to skin or oral cavity),
3-comminuted
4-pathological
Or it can be classified based on site:
1-symphasis
2-parasymphasis
3-angle(in the socket of the wisdom)
4-body(in the socket of canine)
5-condyle
6-coronoid
7-ramus
8-dentoalveolar

Diagnosis :
Patient cant bite on their teeth or dentures
Lingual hematoma is a pathognomanic
Brusing and swelling over the site of fracture

272
273

Trismus
Laceration
Saliva mixed with blood
Patient may complain of parasthesia of inferior dental nerve
Gagging on posterior teeth and mouth hangs open

Radiology used is:DPT,PA if these not available then Land R lateral oblique, for fractures
between symphyseal and and body by rotational PA ,condylar fracture by reverse townes

B)Le Fort 1:mobility of the teeth baring area


Bilateral brusing on the buccal mucosa
Gagging on the posterior teeth
Percusion of upper teeth has a cup crack sound
C)Le Fort 2 and 3:gross oedema of soft tissue
Extensive soft palat brusing
Black eyes (panda facies)
Subconjuctival haemorrhage
Mobile mid face
Dish like face
CSF leakage
Le Fort 2 is associated with infra-orbital never parasthesia
Le Fort 3 is associated with entire seperation of the face and bilateral seperation
of the frontozygomatic suture
Radiograph for diagnosis is :occiptomental 10-30,submento-vertex,lateral skull,PA and CT

D)Malar or zygoma fracture:brusing around the eye


Subconjuctival haemorrhage
Diplopia
Unlateral epistaxis
Parasthesia of the infraorbita nerve
Step deformity of the zygoma
Limitation of lateral extrusion of the mandile

E)Orbital floor fracture:enophthalamos


Diplopia
Upward gaze
Brusing
Subconjtival haemorrhage
Fat and mucsles harinates through the thin wall as a tear drop on the
radiograph
F)White eye blowout:seen in children
NO subconjuctival haemorrhage
Intractable vomiting

G)Nasal:deviation and crumbling of the septa


Obvious nasal deformity
Epixtasis
H)Nasoethmoidal:bilateral blacked eyes
Nasal deformity
Septal deviation

273
274

Epixtasis
CSF leak
55-management of dry socket?
Dry socket appears as severe pain that develop 2-4 days after extraxction due to ostitis of the
socket,it looks inflamed with exposed bone. Management is by gental cleaning of the socket
with normal saline and dressing the alvogyl,BIPPor ZOE pack it is a nightmare to remove so
topical metronidazole is an alternative ,presicribe a analagesia, hot saline mouth wash or
chlorohexidine.

56-What does fracture of zygomatic arch cause?


Trismus,diplopia ,infraorbital never pasathesia,limitation in lateral extrusion of the
mandible.epistaxis ,subconjunctival haemorrhage

57-What does le fort 1 fracture cause?


causes mobility of the upper tooth bearing area

58- What does fracture of 2 condyles cause?


Causes a class 3 occlusion and trismus

59- extraction of 3rd molar with dentigerous cyst in the angle of the mandible , sup and Inf
border of mandible very thin. What can happen during the extraction?
Fracture of the mandible

60- All maxillary teeth moving together?


Le Fort 1

61-Zygomatic arch #?
causes anterior open-bite

62- Fracture of skull more likely to cause meningitis?


a- orbit,
b- b-nasal,
c- zygomatic complex,
d- le fort iii

answer is d,scully page 340, antibiotic is given to patients with middle third maxillofacial
injuries because of the risk of maningitis

63- type of suture that is not used in the uk anymore?


Cat gut

64- A 23 year old male patient has been assaulted and received a blow to his lower jaw.
What are the most likely fractures he has sustained?
A. Parasymphysis and zygomatic butress
B. Angle of mandible and hyoid
C. Parasymphysis and coronoid process
D. Angle of mandible and zyomatic process
E. Parasymphysis and condylar process
I think the answer is e

274
275

65- A patient attends your surgery complaining of a swelling at the angle of the mandible.
There is no obvious dental cause for this swelling.
Which of the following investigations is essential in moving from provisional to definitive
diagnosis?
A. Radiographs
B. CT scan
C. Aspiration
D. Biopsy
E. Full Blood Count
Answer isA, CT scan is second in line diagnosis

66-what are the suture materials used for biopsy,lip trauma,intra-oral fistula?
For biopsy 3/0 black silk suture pink book page 386
Lip truama vicryl resorbable for deep laceration overlying with prolene 6/0 non resorbable
Intra oral fistula vicryl

67- Most important in reimplantation?


PDL and cementum preserved

68- Which fracture which lead to emphysema?


Zygomatic fracture: which lead to destruction of the lateral wall of maxillary antrum causing
emphysemia of the maxillary sinus and Orbital fracture causing emphysemia of the frontral
sinus

69- what are the following :bucket handle,gaurdman’s fracture,cracked cup sound,Guerin
fracture,hanging drop?
Bucket handle is Bilateral parasympheal fracture.
Guradman’s Fracture is bilateral condyler neck along with the mandibular symphsis due to a
fall on the chin
Cracked cup sound on percussion of teeth in Lefort I
Lefort I is also called Guerin fracture.
Hanging drop and fracture leading the enopthalmas is orbit fracture.

70- Periosteal elevator and lingual flap during surgery will damage which nerve ?
The lingual nerve and buccal artery branch of the facial artery

71-who would you manage a patient who comes back after two hours of extraction with
bleeding what is the first step you will be taking?
Reassure the patient they will not bleed to death
In good light suction and clean patients face and mouth
If bleeding is from the socket squeeze the ginigva and ask patient to bite on a swab until
bleeding stops then LA and suture with tight intermatress and swab bite
If it doesn’t stop its from larger vessels LA remove clot from the socket and place a package
of oxidased celleous and or tranexamic acid or aminocarpoic acid this will delay the healing
process so use of BIPP or Whiteheads varnish and suture
If its from under a flap remove the sutures repostion and place new sutures around and follow
the same steps above

72-a patient with black eye and eye ball can not loop upwards ?what muscle is effected?
Orbital floor fracture affecting the inferior rectus muscle(check this question)

275
276

73-what is the anasthetic block technique used for apicectomy of the incisors?
For upper incisor the nasopalatine block which anasthetise the nasopalatine nerve it gives a
deeper anasthesia from canine to canine
For lower incisors mental block which ansethetise from second permolar to the contralateral
side

74-what anasthetic technique to do for canine extraction?


Infilteration

75-how to anasthetise the buccal nerve ?


Inferior dental block

76-radiograph for different type of fractures?


a-mandible :DPT and PA
b-condylar: reversed towen’s
c-fracture between symphysis and angle by rotary PA
d-middle facial :OM,Submentovertex, PA,Lateral skull,

78-what fracture of the jaws will affect the the growth of an 8 year old child?
Intercaspular condylar fracture

79-how do you treat the intra-oral fistula?


Small fistula are asymptomatic and heal spontaneously
If found during extraction closure with suture , decongestant and anti biotic
If found later after extraction do buccal advancement flap to cover the fistula and suture
with vertical mattress
Or palatal flap rotation with same suture
Or buccal fat pad flap
All plus decongastant and antibiotic 500mg erythromycin *4
times or doxycycline 100mg once a day

80- Principle of Cryer's elevator and its uses?


Used to remove interseptal bone and loosen root tip principle used is wheel and axle

81- which infection is a combination of cellulitis and abscess affecting both submandibular
and sublingual spaces bilaterally, the floor of mouth is raised, and the tongue pushed up and
back??
Ludwings angina

82- what is the intra and intercellular oedema generated by inflammatory exudates, the
overlying surface is red and hot and it allows rapid migration of bacteria to surrounding
tissues??
Cellulitis

83- Most common needle used for aspiration biopsy?


a)27gauge
b)30 gauge
c)18gauge
d)16gauge

276
277

answer is c

84- what is the choice of treatment for malpositioned incisors and canines if the apices are in
good position for eruption?
orthodontic and surgical expoure

85- what is the treatment for antral fistula??


A)Antibiotics
B)Buccal advancement flap
C)Palatal rotation flap
D)Buccal fat pad flap
E)all the above
Answer is E

86-expansile lesions full of vascular spongy bone present as symptom less swelling , unless
traumatised ,when bleeding causes pain & rapid expansion.. what is this lesion???
Giant cell granuloma

87- Cotton wool appearance is seen in?


Paget disease

88-what is brown tumor??


Lesion of bone affecting the cortical bone making it very thin and action of osetoclast lead to
radiolucency causing the bone marrow to be repalced by fibrovascular tissue it is secondary
to hyperparathyrodism where bone biochemestry shows decreased in Ca and PO4 ,increase in
alkaline phosphate and PTH,resolves after treatment of hyperparathyrodism

89-what is the risk in doing incisional biopsy?


Can cause shredding of the malignant cells into the circulation

90- the correct material for arteriovenous anastomoses or vascular repair is :


A-9/0 prolene. B-5/0 prolene. C-4/0 vicryl. D-catgut
Answer is a

91-a tumour compromised completely of schwann cells :


A-neurolemmoma. B-ossifying fibroma. C-lipoma. D-granular cell myoblastoma
Answer is a

92-what is the most common symptom of # orbital floor:


A-diplopia. B-subcon.haemorrhage. C-enophthalmos. D-b&c
Answer is A

93-classical x-ray sign of orbital floor fracture is:


A-step deformity. B-hang drop. C- swing sign
Answer is b

94-bilateral parasymphysial fracture is called:


A-pan handle. B-bucket handle. C-guarder handle. D-none of the above.
Answer is B

277
278

95- .a patient retruns after a difficult lower wisdom tooth extraction. He has a lateral open
bite on the operative side and is dribbling. He complains of pain. On examination he is tender
on the operative side, but there is no movement about the operative side. What is diagnosis
A) buccal wall fracture
B) mandibular fracture
C) damage to adjacent teeth
D) temporomandibular joint dislocation
Answer is d

96.what is definition of fistula?


A) an endothelium lined connection between two endothelial surfaces
B) an epthelium lined connection between two epthelial surfaces
Answer is b

97. A cyst of the floor of the mouth arising from sublingual gland. Tends to recur unless
marsupialised. It can pass deep to mylohyoid and appears as a swelling in the neck and floor
of the mouth.what is diagnosis
A) epidermoid cyst
B) fibroepithelial polyp
C) mucocele
D) ranula
Answer is d

98.in the TMN classification system


A) a 1.5 cm tumour on the lateral border of the tongue with no palpable neck nodes would be
classified as stage 1
B) stages are based solely on histopathological grades
C) the N classification relates only to lymph nodes on the ipsilateral side to the tumour
D) Nx means that the patient has undergone a previous dissection
Answer is a

99. To which of the following spaces can infection directly speed from a lower wisdom tooth
A) submasseteric space
B) pterygomaxillary space
C) submandibular space
D)cavernous sinus
Answer is a

100- The procedure of creating a surgical window in the wall of the cyst and evacuvation of
cyst contents is called as?
Marsipulization

101- A pt presents with open bite on left side and with tenderness at nasal bones,it could be
fracture
a- unilateral lefort 1 on right side
b-subcondylar on left side and zygoma on right side
c-lefort 2 on right side
answer is c

102- Glasgow coma scale is used

278
279

a-to ascertain motor responsiveness


b-verbal responsiveness
c-eye response
d-to ascertain the level of consciousness
answer is d

103- failure of primary suturing occurs in facial wounds when


a-fine silk had not been used
b-catgut has been used
c-dead space develops
d-continuous suturing is done

Mixed questions

1. What is recommended dose of Duraphat varnish for children aged 2-5 years
A) 0.25 ml
B) 0.40 ml
C) 0.10 ml
D) 0.50 ml
Answer is A

2. Patient with no probing depths > 3.5 mm, no calculus/ overhangs, but bleeding after
probing. What is BPE score for this patient?
A) 1
B) 3
C)2
D)0
Answer is A

3. The first tooth you look at in a sextant has a furcation involvement. What do you do next?
A) check its pocket depths with the BPE probe
B) move to the next tooth
C) move to the next sextant
D) the BPE is finished as soon as a *( star) score is recorded
Answer is A

4.how long does gingivitis take to develop if toothbrushing is stopped?


A) 1-2 days
B)4-5 days
C)7-10 days
D) 10-12 days

279
280

Answer is b

5. After resuming oral hygiene measures, bleeding on probing typically stops


A) after four hours
B) after two days
C) after one week
D) after one month
Answer is C

6-Can Miconazole be prescribed for pt taking warfarin or statins?


No micronazole increases the effect of wafarin

7-The longest interval btween oral health reviews for pt younger than 18 years should be
A-6mnths
B-3 mnths
C-12 mnths
D-18mnths
E-24 mnths
Answer is C

8-The longest interval betwn oral health reviews for pt aged 18 years and older should be
A-6mnths
B-9mnths
C-12 mnths
D-18mnths
E-24 mnths
Answer is E

9-)what is the medication to be given for a child 12 years old after extraction???
A)Paracetamol
B)Aspirin
C)Ibuprofen
D)Diclofenac
Answer is A

10- 1.what is the MAF( master apical file


A) the file used to measure the length of the tooth
B) the file to used to clear debris from the apical region when recapitulating
C)the first file to bind at working length
D) the file used to set the final diameter of the apical region preparation
Answer is D

11.what size is file ( for endodontics therapy) purple color(ISO)


A) 08
B) 10
C) 15
D)25
Answer is B

12-.stainless steel instruments are

280
281

A) K- type- files
B) headstrong file
C) grater taper(GT)
D) hand protaper
ANSWER IS A and B

13.CPITN-E probe is used for


A) recording BPE's
B) epidemiological screening
C) detailed pocket depth charting
D) calculus detection
ANSWER IS B

14-.when using a Gracey Curette, which direction should you move it?
A) vertically
B) horizontally
C) obliquely
D) all of these
Answer is D

15- % of 12 year olds with caries------[ 38 %]


26. % of children with caries less than 5 years old-------[ 45% ]
what is the percentage of children getting cavities after application of sealants?---- [85% of
the fissure sealents fail after 1 year and 55% after 5 years ]

In a family father is having disease and mother is normal,the dse is inherited to only
daughters and not to the sons,what type of dse is this?

a-sex linked dominant

b-sex linked recessive

c-autosomal dominant

d-autosomal recessive

answer is a

A lower motor neurone facial palsy with vesicles on the same side in the pharynx , external
auditory canal and on the face. May lead to deafness. This is characteristic of

A) Von Recklinghausen syndrome

B) Hurler syndrome

C) Apert syndrome

D) Ramsay- Hunt syndrome

281
282

Answer is d

3.This syndrome consists of facial paralysis, facial oedema and a fissured tongue. It is a
variant of orofacial granulomatosis. This is characteristic of

A) Melkersson- Rosenthal syndrome

B) Behcet syndrome

C) Frey syndrome

D) Graves' disease

Answer is a

4.It tends to affect young adults, especially males, and there is an association with HLA-B5.
Classically picture of oral ulceration,genital ulceration and uveitis. This is characteristic of

A) Larsen syndrome

B) Albright syndrome

C) Behcet syndrome

D) Horner syndrome

Answer is c

5.patient with Down's syndrome have

A) macroglossia

B) periodontal disease

C) delayed tooth eruption

D) large pulp chambers

Answer is a,b,c

bird facies is seen in

a-treacher collins syndrome

b-pierre robin syndrome

c-both of the above

d-none of the above

amswer is b

282
283

Syndromes questions

1.consist of multiple basal cell naevi, multiple odontogenic keratocysts, calcified falx
cerebri, cleidocranial dysostosis. This is characteristic of which syndrome

A) Apert syndrome

B) Gorlin-Goltz syndrome

C) Ramsay- Hunt syndrome

D)Chediak- Higashi syndrome

Answer is b

Which of the following is the main neurological birth syndrome caused by anoxia?

a) Down Syndrome

b) Fragile X syndrome

c) Cerebral palsy

d) Cerebral Vascular accident

answer is c

which of the following is known as royal dse?

a-haemophilia

b-sickle cell anaemia

c-alzheimer's dse

d-colour blindness

answer is a

Individuals with Down Syndrome often have moderate to severe intellectual impairment
with a measurable IQ usually between:

a) 45-50

283
284

b) 55-60

c) 35-55.

d) 25-35

answer is c

For the decontamination of devices like dental instruments, what level of hand hygiene is
appropriate?

a-Social Hand-wash

b-disinfection handwash

c-surgical scrub

answer is a

When should staff be trained in hand hygiene?

a-every six months

b-periodically throughout the year

c-every 3 mnths

answer is b

Fingernails should be kept clean, short and smooth. When viewed from the palm side, how
much nail should be visible?

Should not be visible at all

. How does the GDC protect patients from registered Dental Professionals? True/false

A) by setting standards of patient care

B) by setting standards of behaviour in the practice

C) by routinely inspecting dental practices

D) by approving CPD courses

abc

2.how many verifiable CPD hours need to have DCPs

A)150 hours

B) 75 hours

284
285

C) 50 hours

D) 250 hours

Anser is c

3. If collimator intra- oral x Ray tube is placed further away from the patient

A) they will receive less radiation dose

B) they will receive the same radiation dose

C) they will receive more. Radiation dose

Answer is c

4. The optimal film speed for intra oral radiography is

A) c speed

B) d speed

C) e speed

D) f speed

5.which of the following protocols must be included in the" written practice protocols"?

A) disposal of hazardous waste

B) disposal of sharps

C) annual leave entitlements

D) radiation protection

E) autoclaving

A,b,d

I have listed things from todays exam whatever i can remembr:

La calculation

Gingival hyperplaia pic

Surveyor pic

Wts role of NICE

emq on local anesthetics

285
286

Emq on filling material to be used in diffrnt cases

Emq on filling gaps with diffrnt bridges

Emq on trt under sedation or local or GA

a lot of oral patho/medicine related questions(scenarios)

Lot from law ethics

Question on fracture: zygomatic,orbital

Herpes labials pic

Another pic which i think was frictional kratosis

Lichen planus

Ground glass apernce in wt condition

How to diagnose caries .scenario bases

Wht xrays to take

A pic of old guy having swelling under ear

Pic of boy have swelling over sternocledomastoid muscle

Had to identift these

Pic of pt with recurrnt blister on gums

Pic of pt with splinted tooth-asked 2 question with pic

Pt came to u has down syndrome.had to identify a feature

Opg given .had to tell wt you can see in the pic-it was supernumry teeth

286

S-ar putea să vă placă și